TEST MODE - Test 8 Flashcards

1
Q

Elaborative rehearsal is most useful for:
Select one:

A.
encoding information in sensory memory.

B.
holding information in working memory for an extended period of time.

C.
transferring information from sensory memory to short-term memory.

D.
transferring information from short-term memory to long-term memory.

A

Elaborative rehearsal refers to making new information more meaningful by associating it with previously acquired information.
a. Incorrect Elaborative rehearsal is a method for improving long-term memory and not for encoding information in sensory memory, which is responsible for the temporary storage of incoming sensory signals.

b. Incorrect Rote repetition (e.g., repeating a phone number) is useful for maintaining information in working memory.
c. Incorrect Sensory input is transferred from sensory memory to short-term memory by focusing attention on that input.
d. CORRECT The research has confirmed that elaborative rehearsal is an effective way for ensuring that material is transferred from short-term memory to long-term memory.

The correct answer is: transferring information from short-term memory to long-term memory.

How well did you know this?
1
Not at all
2
3
4
5
Perfectly
2
Q

The use of shaping to establish a complex behavior depends on which of the following?
Select one:

A.
latent learning

B.
positive reinforcement

C.
higher-order conditioning

D.
successive discrimination

A

Shaping involves reinforcing successive approximations to the target (desired) behavior.
a. Incorrect Latent learning occurs without direct reinforcement.

b. CORRECT When using shaping to establish a new behavior, responses that come closer and closer to the desired behavior are successively reinforced.
c. Incorrect Higher-order conditioning is a type of classical (not operant) conditioning.
d. Incorrect Successive discrimination teaches the learner to respond differently to two or more stimuli that are presented successively.

The correct answer is: positive reinforcement

How well did you know this?
1
Not at all
2
3
4
5
Perfectly
3
Q

You are working in a community mental health center that employs two other psychologists, a social worker, and a psychiatrist. One Saturday, as you’re talking to your neighbor over the backyard fence, she tells you that the wife of one of your co-workers (one of the psychologists) is a close friend of hers and has told her that her husband (the psychologist) has started to physically abuse her. As an ethical psychologist, your best course of action would be to:
Select one:

A.
tell the neighbor to have your co-worker’s wife call your office to set up an appointment.

B.
call the wife of the co-worker yourself to confirm the charge before filing a complaint with the Ethics Committee.

C.
discuss the situation with the accused co-worker as soon as possible.

D.
report the matter to the director of the mental health center (the psychiatrist).

A

As a psychologist, one of your duties is to ensure that your colleagues are acting ethically in their professional roles. In this case, the man’s alleged behavior is related to his personal life. However, his personal problems may be interfering with his ability to provide services to his clients. Moreover, his behavior could have adverse consequences on the public trust in psychologists or in the profession of psychology.
a. Incorrect See explanation for response c.

b. Incorrect See explanation for response c.
c. CORRECT Of the responses given, this one is most in line with “common sense” and with the spirit of the Ethics Code. Although the psychologist’s behavior does not involve a client, it is possible that his personal problems are having an indirect impact on his work. Of the responses given, this one is the best answer because it represents the most direct approach to resolving the problem.
d. Incorrect See explanation for response c.

The correct answer is: discuss the situation with the accused co-worker as soon as possible.

How well did you know this?
1
Not at all
2
3
4
5
Perfectly
4
Q

Research looking at the relationship between “expressed emotion” and Schizophrenia has found that:
Select one:

A.
negative expressed emotion is often an early sign of the disorder.

B.
expressed emotion is related more to the negative than the positive symptoms of the disorder.

C.
a high degree of negative expressed emotion by family members is associated with a high-risk for relapse for a person with this disorder.

D.
an extremely high or extremely low level of expressed emotion by family members is an environmental factor that increases the risk for the development of this disorder.

A

In the context of Schizophrenia, negative expressed emotion refers to excessive criticism or hostility or, at the other extreme, emotional overinvolvement and overprotectiveness.

a. Incorrect See explanation for response c.
b. Incorrect See explanation for response c.
c. CORRECT The research has consistently linked negative expressed emotion by family members to a high risk of relapse and rehospitalization for a family member with Schizophrenia.

d. Incorrect See explanation for response c.
The correct answer is: a high degree of negative expressed emotion by family members is associated with a high-risk for relapse for a person with this disorder.

How well did you know this?
1
Not at all
2
3
4
5
Perfectly
5
Q

During Piaget’s heteronomous stage, children’s moral judgments are based on:
Select one:

A.
the intentions behind the act.

B.
the seriousness of the consequences of the act.

C.
the status or authority of the actor.

D.
the events leading up to the act.

A

Piaget proposed a sequence of moral development involving three stages: premoral, moral realism, and moral relativism.

a. Incorrect This is more characteristic of moral relativism (which is also known as the autonomous stage).
b. CORRECT Rules and the consequences of violating a rule are the basis of moral judgment during the stage of moral realism (which is also called the heteronomous stage).
c. Incorrect See explanation for response b.
d. Incorrect See explanation for response b.

The correct answer is: the seriousness of the consequences of the act.

How well did you know this?
1
Not at all
2
3
4
5
Perfectly
6
Q

Statistical regression threatens a study’s internal validity when:
Select one:

A.
individuals are included in a study because of their extremely high (or low) scores on a pretest.

B.
individuals are included in a study because of their moderate (mid-range) scores on a pretest.

C.
the measure of the dependent variable is modified during the course of a study.

D.
ratings on the measure of the dependent variable are affected by “criterion contamination.”

A

Answer A is correct. Statistical regression is the tendency of extreme scores to “regress” (move) toward the mean on retesting.

Statistical regression is a threat to a study’s internal validity whenever participants are selected on the basis of their extreme scores on the pretest. In this situation, it may not be possible to determine if a change in post-test scores is due to the effects of the independent variable or to statistical regression.

The correct answer is: individuals are included in a study because of their extremely high (or low) scores on a pretest.

How well did you know this?
1
Not at all
2
3
4
5
Perfectly
7
Q

The aspect of implicit memory that is referred to as procedural memory is mediated primarily by the:
Select one:

A.
hippocampus.

B.
thalamus.

C.
medulla.

D.
striatum.

A

Implicit memories are retrieved or recalled without conscious awareness. Several types of implicit memories are distinguished including procedural memories (memories for skills and procedures) and associative memories (memories that result from classical conditioning). Implicit memory has been linked to several areas of the brain including the striatum, amygdala, and cerebellum.
a. Incorrect The hippocampus is involved in the processing and consolidation of explicit memories.
b. Incorrect The thalamus also play a role in explicit memory.
c. Incorrect The medulla coordinates swallowing, coughing, and sneezing and regulates a number of vital functions including breathing, heartbeat, and blood pressure.
d. CORRECT The striatum (also referred to as the corpus striatum) is part of the basal ganglia and consists of the caudate nucleus and putamen. It plays an important role in procedural memory.
The correct answer is: striatum.

How well did you know this?
1
Not at all
2
3
4
5
Perfectly
8
Q

A colleague of yours has just attended a seminar on Culture and Ethics where several experts argued that it is inappropriate for psychologists to attempt to obtain a signed written informed consent from members of certain cultural groups prior to their participation in therapy. As an ethical psychologist, you tell your colleague that it:
Select one:

A.
is always necessary to obtain a signed written consent from therapy clients regardless of their ethnic/cultural background.

B.
is always necessary to obtain either a signed written consent or a signed waiver of consent from all therapy clients regardless of their ethnic/cultural background.

C.
is acceptable to document an informed consent from therapy clients in an alternative way only when those clients are illiterate.

D.
may be acceptable to document an informed consent from therapy clients in an alternative way, especially when a signed consent would be inappropriate for the client.

A

An ongoing discussion in the literature is related to the need to consider modifying the informed consent process for members of ethnic/cultural groups who may find written consents objectionable or may want to include family members in the consent process. See, e.g., L. O. Gostin, Informed consent, cultural sensitivity, and respect for persons, Journal of the American Medical Association, 274(10), 844-845, 1995.
a. Incorrect See explanation for response d.

b. Incorrect See explanation for response d.
c. Incorrect See explanation for response d.
d. CORRECT Standard 3.10(c) of the APA’s Ethics Code and Standard I.22 of the Canadian Code of Ethics apply to this situation: Standard 3.10(c) states that “Psychologists appropriately document written or oral consent, permission, and assent”; and Standard I.22 states that psychologists “accept and document oral consent, in situations in which signed consent forms are not acceptable culturally or in which there are other good reasons for not using them.” (Note, however, that legal requirements or institutional regulations may differ, and psychologists should check applicable laws and regulations before deciding to forego a written consent.)

The correct answer is: may be acceptable to document an informed consent from therapy clients in an alternative way, especially when a signed consent would be inappropriate for the client.

How well did you know this?
1
Not at all
2
3
4
5
Perfectly
9
Q

Research on Baddeley’s (2000) multi-component model of working memory suggests that the ____________ plays an essential role in mental arithmetic and is responsible for accessing and executing computational algorithms and heuristics.
Select one:

A.
sensory register

B.
central executive

C.
phonological loop

D.
visuo-spatial sketchpad

A

A. D. Baddeley’s multi-component model of working memory describes working memory as being comprised of a central executive that controls and regulates three specialized “slave” systems - the phonological loop, the visuo-spatial sketchpad, and the episodic buffer (The episodic buffer: A new component of working memory, Trends in Cognitive Science, 4, 417-423, 2000).
a. Incorrect Sensory register is not one of the systems of working memory described by Baddeley.

b. CORRECT The central executive serves a number of supervisory functions including focusing and switching attention, controlling encoding and retrieval strategies, and mentally manipulating information held in the slave systems. For example, while performing mental arithmetic, the central executive is responsible for selecting and executing calculation heuristics.
c. Incorrect The phonological loop is responsible for processing and temporarily storing verbally encoded information. It also plays a role in mental arithmetic but, in contrast to the central executive, is responsible for the temporary storage of partial solutions and subvocal rehearsal of running totals.
d. Incorrect The visuo-spatial sketchpad is responsible for processing and temporarily storing visually encoded information.

The correct answer is: central executive

How well did you know this?
1
Not at all
2
3
4
5
Perfectly
10
Q

Roe and Holland share in common an emphasis on the role of ___________ in career choice.
Select one:

A.
aptitude

B.
personality

C.
social pressure

D.
vocational interests

A

Familiarity with Roe and/or Holland would have helped you choose the correct response to this question. Their approaches to career development and choice are described in the Industrial-Organizational Psychology chapter of the written study materials.
a. Incorrect See explanation for response b.

b. CORRECT Holland proposes that the best career choice is the career that provides a match between the individual’s personality characteristics (e.g., conventional, artistic) and the characteristics of the work environment. Roe links career choice to personality and basic needs.
c. Incorrect See explanation for response b.
d. Incorrect See explanation for response b.

The correct answer is: personality

How well did you know this?
1
Not at all
2
3
4
5
Perfectly
11
Q

Two studies are conducted to test the effects of a cognitive-behavioral technique on attention span in children with ADHD. The studies are identical in procedures, number of participants, and so on, but in Study #1, p.10. Based on this information, you can conclude that:
Select one:

A.
the results of Study #1 indicate an effect opposite of the effect produced in Study #2.

B.
the effect size in Study #1 was larger than the effect size in Study #2.

C.
the researcher has made a Type II error in Study #2.

D.
a one-tailed test was used in Study #1; a two-tailed test in Study #2.

A

Answer B is correct: The difference between the two studies is that one achieved statistical significance (p.10). Significance is achieved when the effect size (effect of the IV) is sufficiently large to be detected by the statistical test. There is enough information given in this question to draw this conclusion.

Answer A: The results may or may not be in the same direction. There is insufficient information in the question to determine which is the case.

Answer C: Study #2 may involve a Type II error, or, alternatively, Study #1 may involve a Type I error. Whether either of these is true cannot be determined from the information provided in the question.

Answer D: There is nothing in the question that implies that this is true.
The correct answer is: the effect size in Study #1 was larger than the effect size in Study #2.

How well did you know this?
1
Not at all
2
3
4
5
Perfectly
12
Q

The ApoE4 gene on chromosome 19 has been linked to an increased risk for:
Select one:

A.
Alzheimer’s disease.

B.
Huntington’s disease.

C.
diabetes mellitus.

D.
multiple sclerosis.

A

Research has identified a link between Alzheimer’s disease and specific genes on several chromosomes including chromosome 19. a. CORRECT The ApoE4 gene on chromosome 19 has been associated with an increased risk for both heart disease and late-onset Alzheimer’s disease. Note that, even if you were not familiar with the ApoE4 gene, knowing that Alzheimer’s disease has been linked to abnormalities on chromosome 19 would have helped you identify this as the correct answer. b. Incorrect Huntington’s disease is an autosomal dominant gene disorder. It has not been linked to the ApoE4 gene. c. Incorrect Diabetes mellitus is due to insufficient insulin or the inability of cells to use insulin. It has not been linked to the ApoE4 gene. d. Incorrect The cause of multiple sclerosis is unknown but may entail a combination of biological and environmental factors.
The correct answer is: Alzheimer’s disease.

How well did you know this?
1
Not at all
2
3
4
5
Perfectly
13
Q

To calculate an effect size (Cohen’s d), you need:
Select one:

A.
the standard error of estimate of the criterion.

B.
the median scores for the pre- and post-tests.

C.
the means of the experimental and control groups.

D.
the actual and predicted scores for the outcome measure.

A

Cohen’s d is one of several methods for calculating an effect size, which is an index of treatment effects.
a. Incorrect See explanation for response c.

b. Incorrect See explanation for response c.
c. CORRECT Cohen’s d indicates the magnitude of the effect of a treatment in terms of the difference between the means of the experimental (treatment) and control (no treatment) groups. It is calculated by subtracting the mean of the control group from the mean of the experimental group and dividing the result by a pooled standard deviation or by the control group standard deviation.
d. Incorrect See explanation for response c.

The correct answer is: the means of the experimental and control groups.

How well did you know this?
1
Not at all
2
3
4
5
Perfectly
14
Q

As defined by Minuchin, triangulation is a:
Select one:

A.
healthy adaptation by family members to unexpected change.

B.
way for family members to avoid dealing with stress or conflict.

C.
method used by the therapist to diagnose the family structure.

D.
method used by the therapist to reduce stress in the family.

A

Minuchin described triangulation as a type of rigid triad.
a. Incorrect See explanation for response b.

b. CORRECT Triangulation is occurring when the parents are in conflict and both try to recruit a child to their side as a means of avoiding or reducing the conflict between them.
c. Incorrect See explanation for response b.
d. Incorrect See explanation for response b.

The correct answer is: way for family members to avoid dealing with stress or conflict.

How well did you know this?
1
Not at all
2
3
4
5
Perfectly
15
Q

Tova and Thom are in their early 30s, have been married for four years, and are expecting their first child. The only vehicle they have had since their marriage is a two-seater sports car, which they both love; but they realize they will have to trade it for something more “sensible” when the baby is born. In terms of Piaget’s theory of cognitive development, the couple’s realization about their car can be best viewed as an example of:
Select one:

A.
assimilation.

B.
accommodation.

C.
centration.

D.
decalage.

A

This is an example of a question that takes a concept that may be familiar but places it in an unexpected context, which can be confusing. Fortunately, you’re likely to encounter only a few questions on the exam like this one.
a. Incorrect Assimilation involves interpreting new information in terms of existing cognitive structures.

b. CORRECT Accommodation involves modifying existing cognitive structures or creating new ones to meet the demands of the environment. In this situation, the couple has realized that they will have to alter their schema for “car.”
c. Incorrect Centration is the inability to focus on more than one aspect of an object or situation at a time and underlies the inability to conserve.
d. Incorrect A horizontal decalage is the name for the gradual mastery of logical concepts.

The correct answer is: accommodation.

How well did you know this?
1
Not at all
2
3
4
5
Perfectly
16
Q

About 10 to 15% of new mothers develop postpartum major depression in the first few days or weeks following delivery. In addition, up to _____% of new mothers exhibit “baby blues,” a less severe condition that involves mood swings and sleep disturbances.
Select one:

A.
30

B.
40

C.
50

D.
80

A

The reported rates for post-partum (baby) blues and depression vary in the literature. However, most authorities agree that the majority of new mothers do experience some degree of post-partum “blues.”

a. Incorrect See explanation for response d.
b. Incorrect See explanation for response d.
c. Incorrect See explanation for response d.
d. CORRECT Up to 80% is the percent commonly cited in the literature.

The correct answer is: 80

How well did you know this?
1
Not at all
2
3
4
5
Perfectly
17
Q

You are a psychodynamic psychotherapist and are seeing a client who has several problems. One of these problems has been found to respond best to a behavioral technique that you are unfamiliar with. You should:
Select one:

A.
seek supervision while applying the technique.

B.
take a class on the technique.

C.
make a referral to a behavioral therapist.

D.
discuss the issue with the client and decide what she wants to do.

A

There really isn’t enough information given in this question to choose the best course of action, but keep in mind that you always want to be somewhat conservative when answering ethics questions on the exam.
a. Incorrect This might be adequate in some situations. The question states, however, that you’re a “psychodynamic psychotherapist,” so you may not be interested in adding a behavioral technique to your repertoire. This may also be inadequate if the problem is a very serious one.

b. Incorrect Obviously, in most situations, this would not be adequate.
c. CORRECT Given the limited information; this is the best answer. Since you don’t have the training or skills needed to provide the optimal treatment to the client, referral would be the best course of action. (Of course, you can continue treating the client for her other problems.)
d. Incorrect This would not be a good course of action.

The correct answer is: make a referral to a behavioral therapist.

How well did you know this?
1
Not at all
2
3
4
5
Perfectly
18
Q

Which of the following is not true of mediation?
Select one:

A.
A mediator can impose a settlement.

B.
A mediator does not serve as an advocate for either party.

C.
Participation in mediation is voluntary.

D.
Arbitration provides a more final resolution than mediation.

A

Mediation is the most used third-party option. A neutral third party (mediator) uses various tactics to facilitate voluntary agreement between the disputants. Remember you are looking for a statement that is not true.
a. CORRECT Because the mediator has no formal powers, he or she cannot impose a settlement.

b. Incorrect A mediator serves as a neutral facilitator, and thus does not advocate for either participant.
c. Incorrect Mediation is voluntary.
d. Incorrect Arbitration is the final and most formal settlement technique.

The correct answer is: A mediator can impose a settlement.

How well did you know this?
1
Not at all
2
3
4
5
Perfectly
19
Q

When calculating the F-ratio for an analysis of variance, which of the following provides an estimate of variability due to the effects of error only?
Select one:

A.
MST

B.
MSW

C.
MSB

D.
MSA

A

The F-ratio is calculated by dividing a measure of treatment effects plus error (MSB) by a measure of error only (MSW).
a. Incorrect MST is “mean square total” and is a measure of both treatment effects and error (MSB plus MSW).

b. CORRECT MSW stands for “mean square within” and is a pooled measure of variability within each of the treatment groups. It provides an estimate of variability that is due to error only.
c. Incorrect MSB stands for “mean square between” and is a measure of variability between treatment groups. It provides an estimate of variability due to treatment effects plus error.
d. Incorrect MSA is not one of the terms used to calculate the F-ratio.

The correct answer is: MSW

How well did you know this?
1
Not at all
2
3
4
5
Perfectly
20
Q

Taylor’s (1911) scientific management is based on the premise that which of the following is most useful for maximizing employee motivation?
Select one:

A.
opportunities for advancement

B.
opportunities to satisfy prepotent needs

C.
satisfying relationships with co-workers

D.
pay that is directly linked to performance

A

As its name implies, scientific management involves applying the principles of science to the field of management in order to determine the most effective way for workers to perform their jobs.
a. Incorrect See explanation for response d.

b. Incorrect See explanation for response d.
c. Incorrect See explanation for response d.
d. CORRECT A premise of scientific management is that workers are motivated primarily by economic self-interest, and Taylor proposed that productivity is maximized when pay is clearly linked to performance.

The correct answer is: pay that is directly linked to performance

How well did you know this?
1
Not at all
2
3
4
5
Perfectly
21
Q

When attempting to expand your private practice, it is important to keep in mind that in-person solicitations:
Select one:

A.
are always unethical.

B.
are unethical only when they target individuals receiving similar services from another professional.

C.
are unethical only when they include deceptive or misleading information.

D.
are unethical when they include deceptive information or when the person solicited is susceptible to undue influence.

A

Although the APA’s prohibitions against solicitation of clients have been limited by the requirements of the Federal Trade Commission, APA still considers certain types of business solicitations unethical.
a. Incorrect In-person solicitations might be ethically acceptable in some situations (e.g., when they are “invited” solicitations).

b. Incorrect This type of solicitation might also be acceptable in some situations.
c. Incorrect This is not the most complete response of those given.
d. CORRECT This is most consistent with Standard 5.06 of the Ethic Code: Solicitations are unethical when they include deceptive or false information or when their targets are individuals who are vulnerable to undue influence.

The correct answer is: are unethical when they include deceptive information or when the person solicited is susceptible to undue influence.

How well did you know this?
1
Not at all
2
3
4
5
Perfectly
22
Q

In the Seattle Longitudinal Study, K. W. Schaie (1996) used which of the following research designs to investigate the relationship between age and intelligence?
Select one:

A.
counterbalanced

B.
crossover

C.
cross-sectional

D.
cross-sequential

A

For the exam, you want to be familiar with the methodology and major results of the Seattle Longitudinal Study. This information is provided in the Psychological Assessment chapter of the written study materials.
a. Incorrect See explanation for response d.

b. Incorrect See explanation for response d.
c. Incorrect See explanation for response d.
d. CORRECT To reduce problems associated with longitudinal and cross-sectional research (e.g., dropouts, cohort effects), Schaie used a cross-sequential design that combined the two strategies. The results of his research identified predictable changes (increases or decreases) in specific cognitive abilities during adulthood and indicated that the range of change for each ability is fairly narrow until age 60 or later.

The correct answer is: cross-sequential

How well did you know this?
1
Not at all
2
3
4
5
Perfectly
23
Q

According to Gyorgy Gergely (1994), visual feature representation is a necessary precondition for:
Select one:

A.
gender identity development.

B.
mirror self-recognition.

C.
attachment.

D.
social referencing.

A

Mirror self-recognition in children, dolphins, and elephants has generated a considerable amount of research in the last decade.

a. Incorrect See explanation for response b.
b. CORRECT According to G. Gergely, the ability to recognize oneself in the mirror (which occurs between 1-1/2 and 2 years in humans) requires a certain level of cognitive development. One cognitive prerequisite is the construction of “a visual feature representation of the typical physical appearance of the not-directly-visible parts of … [the] body” (p. 55). [From self-recognition to theory of mind~ in S. T. Parker et al., (Eds.), Self-awareness in animals and humans, Boston, Cambridge University Press, 1994.]
c. Incorrect See explanation for response b.
d. Incorrect See explanation for response b.

The correct answer is: mirror self-recognition.

How well did you know this?
1
Not at all
2
3
4
5
Perfectly
24
Q

When a person’s hypertension is not adequately alleviated through lifestyle changes, a hypertension drug may be prescribed. These drugs include all of the following except:
Select one:

A.
NSAIDs.

B.
beta blockers.

C.
ACE inhibitors.

D.
calcium channel blockers.

A

The choice of a hypertension drug for an individual depends on his/her medical history. However, the first drug is usually a diuretic and, if that does not reduce blood pressure to an acceptable level, a beta blocker, ACE inhibitor, or calcium channel blocker may be prescribed. Note that this question is asking for the drug that is NOT prescribed for hypertension.
a. CORRECT NSAIDs (non-steroidal anti-inflammatory drugs) increase blood pressure and interfere with the effects of antihypertensive medication. Therefore, they should usually be avoided by people with high blood pressure.

b. Incorrect Beta blockers decrease blood pressure through their action on the sympathetic nervous system.
c. Incorrect Angiotensin converting enzyme (ACE) inhibitors decrease blood pressure by dilating blood vessels.
d. Incorrect Calcium channel blockers alter blood pressure by slowing the movement of calcium in the cells of the heart and blood vessel walls.

The correct answer is: NSAIDs.

How well did you know this?
1
Not at all
2
3
4
5
Perfectly
25
Q

In children, Generalized Anxiety Disorder:
Select one:

A.
usually first develops after a recent loss such as parental divorce or death of a family member.

B.
is usually manifested as autonomic hyperarousal and other prominent physical symptoms.

C.
is often manifested as excessive concern about performance or competence at school.

D.
typically involves crying, tantrums, freezing, and clinging to family members, especially among those under 7 years of age.

A

The symptoms of Generalized Anxiety Disorder (GAD) are somewhat age-related.

a. Incorrect This is not characteristic of GAD.
b. Incorrect Autonomic hyperarousal is actually somewhat less in GAD than in other Anxiety Disorders.
c. CORRECT Children with GAD often worry excessively about their competence or the quality of their performance and this worry is frequently related to schoolwork and performance in sports activities.
d. Incorrect This is characteristic of children with Social Phobia.

The correct answer is: is often manifested as excessive concern about performance or competence at school.

How well did you know this?
1
Not at all
2
3
4
5
Perfectly
26
Q

(phenylketonuria) is an inherited disorder that can cause Intellectual Disability:
Select one:

A.
only in individuals who are homozygous for the condition.

B.
only in individuals who are heterozygous for the condition.

C.
in individuals who are either homozygous or heterozygous for the condition.

D.
in an individual who has at least one parent who is homozygous for the condition.

A

PKU is an autosomal recessive condition, which means that an individual must have two recessive genes (i.e., be homozygous) to have the disorder.
a. CORRECT PKU is carried on a recessive gene. Consequently, to have the disorder, an individual must have inherited one recessive gene from each parent.

b. Incorrect See explanation above.
c. Incorrect See explanation above.
d. Incorrect See explanation above.

The correct answer is: only in individuals who are homozygous for the condition.

How well did you know this?
1
Not at all
2
3
4
5
Perfectly
27
Q

Dr. X has been asked to administer a psychological test to an individual who requires special accommodations because of a physical disability. The test was not developed or normed for individuals with this disability. The APA’s Guidelines for Test User Qualifications recommend that, in this situation, psychologists:
Select one:

A.
refuse to administer the test.

B.
refuse to administer the test unless there are no alternative methods of assessment.

C.
exercise “special care” when administering and scoring the test and interpreting its results.

D.
obtain legal advice on relevant legal requirements regarding the administration and use of the test.

A

The APA’s position on this issue is summarized by S. M. Turner, S. T. DeMers, H. R. Fox, and G. M. Reed in APA’s Guidelines for Test User Qualifications: An executive summary (American Psychologist, 56, 1099-1113, 2001).
a. Incorrect See explanation for response d.

b. Incorrect See explanation for response d.
c. Incorrect See explanation for response d.
d. CORRECT In their summary, Turner et al. note that there may be legal requirements that apply to the administration and use of tests when an individual requires special accommodations. To ensure that these requirements are met, psychologists should consider them and, when appropriate, “obtain legal advice on legal and regulatory requirements regarding appropriate administration of tests and use of test data when assessing individuals with disabilities” (p. 1103). Note that Standard 10.2 of the Standards for Educational and Psychological Testing also addresses this issue and requires test users who do not have adequate knowledge or experience in modifying tests for individuals with disabilities to seek guidance from a consultant or supervisor who has expertise.

The correct answer is: obtain legal advice on relevant legal requirements regarding the administration and use of the test.

How well did you know this?
1
Not at all
2
3
4
5
Perfectly
28
Q

Ebbinghaus was one of the first investigators to systematically study memory. In his studies, Ebbinghaus used himself as a subject and memorized lists of nonsense syllables. Results of his research indicated that:
Select one:

A.
syllables in the middle of a list are better remembered than those at the beginning and end of the list.

B.
when memorizing syllables, new learning tends to interfere with previous learning.

C.
overlearning improves memory for syllables up to a point but thereafter has no effect.

D.
rote learning of syllables tends to lead to rapid forgetting.

A

Ebbinghaus was interested in assessing the effects of time on memory and, based on the results of his studies, constructed a “curve of forgetting,” which shows that 50% of memorized syllables are forgotten within less than one hour.
a. Incorrect This is the opposite of what is true. In addition, this was not a phenomenon discussed by Ebbinghaus.

b. Incorrect This phenomenon is referred to as retroactive inhibition but was not discussed by Ebbinghaus.
c. Incorrect This is not true; the greater the overlearning, the greater the retention.
d. CORRECT Ebbinghaus memorized syllables using rote learning (verbatim learning through repetition) and found that rote learning of nonsense syllables leads to rapid forgetting.

The correct answer is: rote learning of syllables tends to lead to rapid forgetting.

How well did you know this?
1
Not at all
2
3
4
5
Perfectly
29
Q

A drive for order that involves the testing of patterns and structures against the real world is referred to as:
Select one:

A.
elaboration.

B.
deduction.

C.
equilibration.

D.
accommodation.

A

This is a difficult question because while it may ask about a familiar concept it uses language that is unfamiliar. Using the “process of elimination” may have helped you choose the correct answer.

a. Incorrect Elaboration refers to the process of increasing the number of associations between items of information which enhances the meaningfulness of the information and is associated with improvements in retention and retrieval.
b. Incorrect Deduction refers to reasoning from the general to the specific.
c. CORRECT As defined by Piaget equilibration involves a combination of assimilation and accommodation and is motivated by a drive for balance or order.
d. Incorrect Accommodation is only one aspect of equilibration.

The correct answer is: equilibration.

How well did you know this?
1
Not at all
2
3
4
5
Perfectly
30
Q

Yerkes and Dodson’s (1908) “inverted-U” hypotheses predicts that:
Select one:

A.
moderate levels of group cohesiveness are associated with the highest levels of performance.

B.
moderate levels of arousal are associated with the highest levels of performance.

C.
performance is a function of task complexity, level of arousal, and learned habits.

D.
performance is the result of an interaction between motivation, ability, and task difficulty.

A

The inverted-U theory hypothesis is also known as the Yerkes-Dodson Law and has been applied to a number of activities including academic learning and sports performance. The Yerkes-Dodson 1908 study is a classic in the field and is likely to be asked about on the exam.
a. Incorrect See explanation for response b.

b. CORRECT The inverted-U hypothesis predicts that, for every activity, there is an optimum level of arousal and that arousal above or below that level has a negative impact on performance. Of the answers given, this one best describes this hypothesis.
c. Incorrect This answer describes an assumption underlying Hull’s (1943) drive theory.
d. Incorrect This answer does not accurately describe the inverted-U hypothesis.

The correct answer is: moderate levels of arousal are associated with the highest levels of performance.

How well did you know this?
1
Not at all
2
3
4
5
Perfectly
31
Q

When using flooding (in vivo exposure) to treat a client’s Specific Phobia, which of the following instruments would be most useful for monitoring the client’s progress in treatment and determining when to terminate treatment?
Select one:

A.
SUDS

B.
SCID

C.
MMSE

D.
BDI

A

Flooding involves exposing the client to high anxiety-arousing situations. When using this technique, the client’s anxiety level is regularly monitored to determine the progress of the exposure during a single session and over multiple sessions. Only one of the instruments listed in the answers is useful for this purpose.
a. CORRECT When using SUDS (Subjective Units of Distress Scale), the client rates his/her level of anxiety on a scale ranging from 1 to 100 (or, alternatively, from 1 to 10) at regular intervals during exposure. In general, a single exposure session should be continued until a 50% reduction in anxiety level occurs. See, e.g., J. Rosqvist, Exposure treatments for anxiety disorders, Boca Raton, FL, CRC Press, 2005. Note that being familiar with SUDS would have helped you identify this as the correct answer even if you’re unfamiliar with its use in conjunction with flooding.

b. Incorrect The SCID (Structured Clinical Interview for DSM) is a semi-structured diagnostic interview that is useful for deriving a client’s diagnosis.
c. Incorrect The MMSE (Mini Mental State Exam) is a screening test for cognitive impairment in older adults.
d. Incorrect The BDI (Beck Depression Inventory) is useful for assessing and monitoring severity of depressive symptoms.

The correct answer is: SUDS

How well did you know this?
1
Not at all
2
3
4
5
Perfectly
32
Q

Which of the following is considered a cornerstone of Bandura’s social-cognitive theory?
Select one:

A.
cognitive load

B.
equilibration

C.
scaffolding

D.
vicarious learning

A

Knowing that Bandura’s theory is also known as social learning theory and observational learning theory would have helped you identify the correct answer to this question.
a. Incorrect Cognitive load is a key concept in cognitive theories that address the amount of information that can be held in short-term memory before information is lost.

b. Incorrect Equilibration is a central concept in Piaget’s constructivism and refers to the drive to achieve an optimal state of balance between one’s cognitive structures and the demands of the environment.
c. Incorrect Scaffolding is associated with Vygotsky’s sociocultural theory and refers to the assistance given to a learner by a parent, teacher, or other person within the learner’s zone of proximal development.
d. CORRECT Bandura proposed that human behavior is influenced by vicarious learning - i.e., by watching another person engage in that behavior.

The correct answer is: vicarious learning

How well did you know this?
1
Not at all
2
3
4
5
Perfectly
33
Q

What is a unique challenge that comes from using Evidence-Based Therapies in public mental health services settings?
Select one:

A.
No universal guidelines provided by the government.

B.
No standardized supervision or training literature.

C.
No procedures for selecting supervisors or trainers.

D.
No testing applications to verify licensing authenticity.

A

The correct answer is B. A significant portion of the training and supervision literature within the field of clinical psychology does not often address the unique challenges that arise in training in the public mental health service settings. A comprehensive model for providing training in EBTs that can apprise local and larger scale implementation and facilitate training partnerships with publicly funded or community-based agencies and practitioners of EBTs has been demonstrated to be most effective. Answers A, C and D are incorrect as government guidelines, lack of procedures and lack of testing applications are not related to any unique challenges with employing evidence-based therapies in public mental health based settings.

The correct answer is: No standardized supervision or training literature.

How well did you know this?
1
Not at all
2
3
4
5
Perfectly
34
Q

During the alarm stage of the general adaptation syndrome (GAS), the release of _______ results in an increase in heart rate, blood pressure, and respiration rate.
Select one:

A.
thyroxine

B.
melatonin

C.
epinephrine

D.
insulin

A

During the initial alarm stage of the GAS, the hypothalamus activates the adrenal medulla to increase its release of epinephrine.
a. Incorrect Thyroxine is secreted by the thyroid gland and controls the rate of metabolism and plays a role in protein synthesis.

b. Incorrect Melatonin is secreted by the pineal gland and plays a role in the sleep-wake cycle.
c. CORRECT Epinephrine (which is also referred to as adrenaline) is a hormone and neurotransmitter. It is released by the adrenal medulla during the alarm stage of the GAS and is responsible for the body’s “fight-or-flight” response.
d. Incorrect Insulin is secreted by the pancreas and is responsible for the uptake and use of glucose.

The correct answer is: epinephrine

How well did you know this?
1
Not at all
2
3
4
5
Perfectly
35
Q

During the second therapy session with a client, she admits that she has beaten her 7-year-old son on two occasions. She is very remorseful, promises never to do it again, and says she will continue coming to therapy to get the help she needs. You should:
Select one:

A.
continue seeing the woman in therapy and contact the authorities only if you believe she is capable of hitting her son again.

B.
continue seeing the woman and make a report to the authorities only if she stops coming to therapy.

C.
explain that you are obligated to make a report and call the authorities while she is in your office.

D.
say nothing to the client about making a report but call the authorities as soon as possible after the therapy session ends.

A

In this situation, you would be legally required to make a report to the appropriate authorities.
a. Incorrect See explanation for response c.

b. Incorrect See explanation for response c.
c. CORRECT The experts generally agree that it is often preferable to involve the client in the reporting process (e.g., to make the report when the client is present). Consequently, this is the best answer of those given.
d. Incorrect See explanation for response c.

The correct answer is: explain that you are obligated to make a report and call the authorities while she is in your office.

How well did you know this?
1
Not at all
2
3
4
5
Perfectly
36
Q

From the perspective of operant conditioning, which of the following techniques is most useful for combining component responses into a particular sequence?
Select one:

A.
stimulus generalization

B.
response priming

C.
shaping

D.
chaining

A

Translating this question into a less awkward statement might have made it easier to answer; e.g., “Which operant technique is used to put together individual responses in a particular sequence?”
a. Incorrect In classical conditioning, stimulus generalization occurs when an organism responds with the target response to stimuli similar to the conditioned stimulus.

b. Incorrect Response priming refers to any procedure used to initiate the early steps in a sequence of responses. Although response priming can be used to initiate a behavioral chain, this is not the best answer of those given.
c. Incorrect People often confuse shaping and chaining. However, in shaping, the goal is the end behavior; in chaining, the whole string (chain) of behaviors is important.
d. CORRECT Chaining is used to combine a sequence of responses in order to end up with a “behavior chain.” Chaining is considered by advocates of operant conditioning as the mechanism underlying the acquisition of at least some complex behaviors.

The correct answer is: chaining

How well did you know this?
1
Not at all
2
3
4
5
Perfectly
37
Q

Creating an acronym involves:
Select one:

A.
forming a word or pronounceable sequence of letters from the first letter of the words included in a list of words.

B.
creating a sentence or rhyme using the first letter of the words included in a list of words.

C.
creating an image that combines the images of two or more words.

D.
chunking items together to make a smaller number of items.

A

For the exam, you want to be familiar with acronyms and other mnemonic devices that are described in the Learning Theory chapter of the written study materials.
a. CORRECT This is the method used to form an acronym. For example, students sometimes use the acronym “PEMDAS” to remember the order of operations in mathematics - i.e., parentheses, exponents, multiplication, division, addition, and subtraction.

b. Incorrect This is the procedure for creating an acrostic. “Please excuse my dear Aunt Sally” is an acrostic for the order of operations in mathematics.
c. Incorrect This describes the keyword method.
d. Incorrect Chunking is useful for maintaining information in short-term memory.

The correct answer is: forming a word or pronounceable sequence of letters from the first letter of the words included in a list of words.

How well did you know this?
1
Not at all
2
3
4
5
Perfectly
38
Q

The kappa statistic is used as a measure of reliability when data are:
Select one:

A.
nominal or ordinal (discontinuous).

B.
interval or ratio (continuous).

C.
nonlinear.

D.
metric.

A

The kappa statistic is used to evaluate inter-rater reliability.
a. CORRECT The kappa statistic is used to measure the consistency of ratings assigned by two raters when data are nominal or ordinal. (Note that some authors use the term “discontinuous” to refer to nominal and other discrete data - i.e., data that represents noncontinuous categories.)

b. Incorrect See explanation above.
c. Incorrect See explanation above.
d. Incorrect See explanation above. (Note that the term “metric” is sometimes used to refer to interval and ratio data.)

The correct answer is: nominal or ordinal (discontinuous).

How well did you know this?
1
Not at all
2
3
4
5
Perfectly
39
Q

Which of the following best describes the relationship between REM sleep and dreaming?
Select one:

A.
All dreaming occurs during REM sleep.

B.
Most dreaming occurs during REM sleep and dreams during non-REM sleep are less vivid and elaborate and more realistic.

C.
Most dreaming occurs during REM sleep but dreams during non-REM sleep are more vivid, elaborate, and bizarre in nature.

D.
Most dreaming occurs during REM sleep and dreams during non-REM sleep occur only when the individual has been sleep-deprived.

A

Studies on sleep and dreaming have consistently confirmed that most dreams occur during REM (rapid eye movement) sleep.
a. Incorrect See explanation for response b.

b. CORRECT Although some dreaming occurs during non-REM (Stages 1 through 4) sleep, these dreams consist of fleeting images and are less vivid, detailed, and storylike than the dreams that occur during REM sleep.
c. Incorrect See explanation for response b.
d. Incorrect See explanation for response b.

The correct answer is: Most dreaming occurs during REM sleep and dreams during non-REM sleep are less vivid and elaborate and more realistic.

How well did you know this?
1
Not at all
2
3
4
5
Perfectly
40
Q

Compared to decentralized communication networks, centralized networks:

Select one:

A.
are associated with fewer errors and greater productivity when the task is simple.

B.
are associated with fewer errors and greater productivity when the task is complex.

C.
are associated with fewer errors and greater productivity regardless of whether the task is simple or complex.

D.
are associated with more errors and poorer productivity regardless of whether the task is simple or complex.

A

Communication networks are often described as centralized or decentralized. The most effective network depends on several factors, including the characteristics of the task.<p>a. CORRECT As its name implies, a centralized network requires all communication to pass through a central person. It is most effective for simple tasks but is less efficient when the task is complex.</p><p>b. Incorrect A decentralized network is more efficient when the task is complex.</p><p>c. Incorrect See explanation for response a.</p><p>d. Incorrect See explanation for response a.

The correct answer is: are associated with fewer errors and greater productivity when the task is simple.</p>

How well did you know this?
1
Not at all
2
3
4
5
Perfectly
41
Q

Research conducted by Mamie and Kenneth Clark in the 1940s led to their conclusion that:
Select one:

A.
for young Black children, recognition of their race is linked to low self-esteem.

B.
distrust of Whites by adolescent and adult Blacks is a healthy, adaptive response to racism.

C.
young Black children do not begin to form a racial identity until they enter school.

D.
when compared to White children, Black children exhibit accelerated motor development during the first three years of life.

A

Sociologists Kenneth and Mamie Clark studied the identity and self-esteem of African American children in the 1930s and 1940s.
a. CORRECT The results of the Clarks’ research helped the NAACP win the Brown v. Board of Education case, which led to the banning of school segregation.

b. Incorrect See explanation above.
c. Incorrect See explanation above.
d. Incorrect See explanation above.

The correct answer is: for young Black children, recognition of their race is linked to low self-esteem.

How well did you know this?
1
Not at all
2
3
4
5
Perfectly
42
Q

A solution-focused therapist is working with a family whose conversations frequently escalate to intense arguments. The therapist will most likely:
Select one:

A.
reduce tension during therapy sessions by having family members talk directly to her rather than to each other.

B.
appoint one family member to act as “referee” whenever the family fights at home.

C.
ask the family members if they can remember a time when they were able to talk to each other for an extended period without arguing.

D.
instruct the family members to argue with each other for at least one hour each evening.

A

Not surprisingly, solution-focused therapists focus on solutions.
a. Incorrect See explanation for response c.

b. Incorrect See explanation for response c.
c. CORRECT Solution-focused therapists use “exceptions” as one way of identifying possible solutions to the family’s problem.
d. Incorrect See explanation for response c.

The correct answer is: ask the family members if they can remember a time when they were able to talk to each other for an extended period without arguing.

How well did you know this?
1
Not at all
2
3
4
5
Perfectly
43
Q

“Consensual observer drift” tends to __________ the size of the inter-rater reliability coefficient.
Select one:

A.
artificially deflate

B.
artificially inflate

C.
accurately deflate

D.
accurately inflate

A

Consensual observer drift occurs when two or more raters work together and influence each other’s ratings. Consensual observer drift tends to make the raters’ ratings more similar, which artificially inflates the inter-rater reliability coefficient.

The correct answer is: artificially inflate

How well did you know this?
1
Not at all
2
3
4
5
Perfectly
44
Q

Malnutrition during fetal development is likely to have which effect on the brain?
Select one:

A.
scarring due to the death of brain cells

B.
synaptic enlargement

C.
malformation of the brain cells

D.
a reduced number of brain cells

A

Most of the research on the effects of malnutrition on the developing brain have been conducted on animals. Studies with humans have, however, suggested that the effects are very similar.
a. Incorrect See explanation for response d.

b. Incorrect See explanation for response d.
c. Incorrect See explanation for response d.
d. CORRECT The most consistent finding of the research is that malnutrition during fetal development results in a reduced number of brain cells. There is also some evidence that there are problems in myelination.

The correct answer is: a reduced number of brain cells

How well did you know this?
1
Not at all
2
3
4
5
Perfectly
45
Q

Which of the following is one of the initial symptoms of renal failure?
Select one:

A.
a change in the amount and frequency of urination

B.
a salty or sour taste in the mouth

C.
feeling hot even in a cool room

D.
increased appetite with weight loss

A

Renal failure is also known as kidney failure and occurs when the kidneys are no longer able to remove waste from the body and maintain a normal level of electrolytes in the bloodstream.
a. CORRECT The symptoms of renal failure depend on its severity, rate of progression, and cause. However, in many cases, the initial symptoms are a change in the amount and frequency of urination and discomfort or pain when urinating. The change can involve an increase in urination (polyuria) with urine being paler than usual or a decrease in urination (oliguria) with urine being darker than usual.

b. Incorrect A metallic taste (not a salty or sour taste) in the mouth is one of the symptoms of renal failure.
c. Incorrect Feeling cold (even in a hot room) is one of the symptoms of renal failure.
d. Incorrect People with renal failure may experience a decrease (not an increase) in appetite.

The correct answer is: a change in the amount and frequency of urination

How well did you know this?
1
Not at all
2
3
4
5
Perfectly
46
Q

Gray and McNaughton’s (2000) revised version of reinforcement sensitivity theory proposes that there are three systems of neural circuitry that explain differences in personality. Which of the following is not one of these systems?
Select one:

A.
fight-flight-freeze system

B.
behavioral activation system

C.
attachment-detachment system

D.
behavioral inhibition system

A

J. A. Gray and N. McNaughton’s (2000) revised version of reinforcement sensitivity theory (RST) distinguishes between three brain systems (FFFS, BAS, and BIS) that are responsible for individual differences in responses to punishing and rewarding events and help explain differences in personality and psychopathology (The neuropsychology of anxiety: An enquiry into the functions of the septo-hippocampal system, Oxford, Oxford University Press, 2000).
a. Incorrect The fight-flight-freeze system (FFFS) is one of the systems identified by RST. It mediates reactions to conditioned and unconditioned aversive stimuli and, in terms of personality, is associated with fear proneness and avoidance which can lead to phobic reactions and panic.

b. Incorrect According to Gray and McNaughton, the behavioral activation system (BAS) mediates reactions to conditioned and unconditioned appetitive (rewarding) stimuli. With regard to personality, the BAS is associated with optimism, impulsivity, and a reward orientation which can lead to addictive behaviors and high risk, impulsive behaviors (such as those associated with mania).
c. CORRECT An attachment-detachment system is not one of the three systems identified by RST.
d. Incorrect The behavioral inhibition system (BIS) is the third system identified by RST. It’s responsible for resolving goal conflicts, including conflicts between BAS-mediated approach behaviors and FFFS-mediated avoidance behaviors. Associated personality characteristics are worry proneness and anxious rumination which can lead to Generalized Anxiety Disorder and OCD.

The correct answer is: attachment-detachment system

How well did you know this?
1
Not at all
2
3
4
5
Perfectly
47
Q

Traumatic brain injury (TBI) is likely to have the most detrimental effect on which of the following WAIS-IV Indexes?
Select one:

A.
Working Memory

B.
Processing Speed

C.
Perceptual Reasoning

D.
Verbal Comprehension

A

Answer B is correct: The WAIS-IV is often used to assist in the diagnosis of traumatic brain injury (TBI). The Processing Speed score is most adversely affected by TBI, with the negative impact being most pronounced for individuals with moderate to severe TBI.

Answer A: Although memory is affected by TBI, the Working Memory score tends to be less adversely affected than the Processing Speed score.

Answer C: Perceptual Reasoning is less adversely affected than Processing Speed.

Answer D: Verbal Comprehension is less adversely affected than Processing Speed.

The correct answer is: Processing Speed

How well did you know this?
1
Not at all
2
3
4
5
Perfectly
48
Q

Recently, the United States Supreme Court ruled that those who were Gay and Lesbian could legally be married. Until then, those who were heterosexual were part of a ______________social group than those who were male or female homosexuals.
Select one:

A.
Dominant

B.
Stagnant

C.
Privileged

D.
Narrowing

A

The correct answer is C. Privilege is an aspect of the American power system that occurs when a right or benefit belongs to one group. Answers A, B and D are incorrect as none of these answers properly describes the previous dynamic between opposite and same-sex marriage and the legal and social laws that governed it.

The correct answer is: Privileged

How well did you know this?
1
Not at all
2
3
4
5
Perfectly
49
Q

According to Gerald Patterson and his colleagues, which of the following is a key contributor to Conduct Disorder?

Select one:

A.
the child’s temperament

B.
the child’s cognitive skills

C.
the strength of the parent-child attachment

D.
the parent’s childrearing skills

A

The approach of Gerald Patterson and his colleagues to aggressive behavior in children and adolescents is classified as a social learning approach.<p>a. Incorrect See explanation for response d.</p><p>b. Incorrect See explanation for response d.</p><p>c. Incorrect See explanation for response d.</p><p>d. <b>CORRECT</b> Patterson et al.’s “coercion hypothesis” focuses on interactions between parents and their children and identifies the parents’ childrearing skills as a key determinant of the nature of these interactions.

The correct answer is: the parent’s childrearing skills</p>

How well did you know this?
1
Not at all
2
3
4
5
Perfectly
50
Q

Anna, age 34, tells you that she is convinced that she is extremely overweight even though her body mass index is within normal limits. She says that she’s mostly concerned about her “fat thighs and stomach” which make her feel ugly and that she’s afraid to go out in public because she thinks people will ridicule her. If a DSM-5 diagnosis of Body Dysmorphic Disorder is the appropriate diagnosis for Anna, further questioning of Anna will confirm that:
Select one:

A.
her self-evaluations are unduly influenced by her body weight and shape.

B.
she performs repetitive behaviors or mental acts related to her concerns about her weight.

C.
she is preoccupied with having or acquiring a serious illness related to her weight.

D.
she exhibits impairments in cognitive functioning.

A

Answer B is correct: The performance of repetitive behaviors or mental acts related to the individual’s concerns was added to the DSM-5 as a diagnostic criterion for Body Dysmorphic Disorder.

Answer A: Self-evaluations that are unduly influenced by body weight and shape is a diagnostic criterion for Bulimia Nervosa and would not confirm a diagnosis of Body Dysmorphic Disorder.

Answer C: A preoccupation with having or acquiring a serious illness is characteristic of Illness Anxiety Disorder and would not confirm a diagnosis of Body Dysmorphic Disorder.

Answer D: The DSM-5 notes that executive dysfunction (impaired higher-order cognitive skills) is an associated symptom of Body Dysmorphic Disorder. Because it is an associated symptom rather than a diagnostic criterion, it would not be as useful as performing repetitive behaviors or mental acts for confirming a diagnosis of Body Dysmorphic Disorder.

The correct answer is: she performs repetitive behaviors or mental acts related to her concerns about her weight.

How well did you know this?
1
Not at all
2
3
4
5
Perfectly
51
Q

To determine the degree of association between number of hours studied and score on the EPPP for a sample of 50 licensure candidates, you would use which of the following bivariate correlation coefficients?
Select one:

A.
Spearman rho

B.
Pearson r

C.
phi coefficient

D.
point biserial coefficient

A

In this situation, you will correlate two continuous variables.
a. Incorrect Spearman rho is used to correlate two variables that are measured in terms of ranks.

b. CORRECT The Pearson product moment correlation coefficient (Pearson r) is the appropriate correlation coefficient when both variables are measured on a continuous scale.
c. Incorrect The phi coefficient is used to determine the correlation between two dichotomous variables.
d. Incorrect The point biserial correlation coefficient is appropriate when one variable is continuous and the other is a true dichotomy.

The correct answer is: Pearson r

How well did you know this?
1
Not at all
2
3
4
5
Perfectly
52
Q

Which of the following is true about the difference between Obsessive-Compulsive Personality Disorder and Obsessive-Compulsive Disorder (OCD)?
Select one:

A.
Obsessions and compulsions in Obsessive-Compulsive Personality Disorder are less severe than those in OCD but began in early adulthood and are present in a variety of contexts.

B.
Obsessions and compulsions in Obsessive-Compulsive Personality Disorder (but not in OCD) are experienced by the individual as ego-syntonic.

C.
The diagnosis of Obsessive-Compulsive Personality Disorder (but not OCD) requires the presence of obsessions and compulsions prior to age 21.

D.
The diagnosis of OCD (but not Obsessive-Compulsive Personality Disorder) requires the presence of obsessions and/or compulsions.

A

The name “Obsessive-Compulsive Personality Disorder” is misleading because the disorder is NOT characterized by obsessions or compulsions.

a. Incorrect See explanation for response d.
b. Incorrect See explanation for response d.
c. Incorrect See explanation for response d.
d. CORRECT Obsessive-Compulsive Personality Disorder is characterized by a preoccupation with orderliness, perfectionism, and control. The diagnosis does not require the presence of obsessions and/or compulsions.

The correct answer is: The diagnosis of OCD (but not Obsessive-Compulsive Personality Disorder) requires the presence of obsessions and/or compulsions.

How well did you know this?
1
Not at all
2
3
4
5
Perfectly
53
Q

One prediction of goal-setting theory is that difficult goals result in greater productivity than easy or ambiguous goals. With regard to task difficulty, the studies have shown that:
Select one:

A.
the more complex the task, the stronger the link between goal difficulty and productivity.

B.
the less complex the task, the stronger the link between goal difficulty and productivity.

C.
the relationship between task complexity and productivity is curvilinear with the strongest link being between moderate goal complexity and productivity.

D.
the relationship between task complexity and productivity is curvilinear with the strongest links being between low and high goal complexity and productivity.

A

Even if you’re not familiar with the research, it makes sense that goal-setting theory would apply better to simple tasks than difficult tasks, since performance on the latter would also depend on other factors in addition to the acceptance of difficult goals.
a. Incorrect This is the opposite of what is true.

b. CORRECT This is the conclusion reached by the developers of goal-setting theory. See R. F. Wood, E. A. Locke, and A. J. Mento, Task complexity as a moderator of goal effects: A meta-analysis, Journal of Applied Psychology, 72, 416-425, 1987.
c. Incorrect See explanation above.
d. Incorrect See explanation above.

The correct answer is: the less complex the task, the stronger the link between goal difficulty and productivity.

How well did you know this?
1
Not at all
2
3
4
5
Perfectly
54
Q

Interpreting the defenses against anxiety that underlie a client’s current maladaptive behavior patterns would be of most interest to a practitioner of:
Select one:

A.
solution-focused therapy.

B.
object relations therapy.

C.
reality therapy.

D.
personal construct therapy.

A

Defenses are of interest to psychodynamic psychotherapists.
a. Incorrect Solution-focused therapy adopts a here-and-now orientation and focuses on identifying solutions to problems.

b. CORRECT As a psychodynamic therapy, object relations therapy utilizes many traditional psychodynamic strategies. It focuses on the impact of unconscious processes that impact the client’s current relationships and involves identifying and interpreting the client’s defenses and transferences.
c. Incorrect Reality therapy focuses on current issues and current problems.
d. Incorrect Personal construct therapy combines cognitive, behavioral, and humanistic concepts and focuses on the impact of the individual’s perspective on his/her experience of the world.

The correct answer is: object relations therapy.

How well did you know this?
1
Not at all
2
3
4
5
Perfectly
55
Q

A DSM diagnosis of Bipolar II Disorder requires the presence or history of:
Select one:

A.
at least one major depressive episode and one hypomanic episode.

B.
at least one major depressive episode and one mixed episode.

C.
two or more mixed episodes without a manic, hypomanic, or major depressive episode.

D.
two or more hypomanic episodes.

A

For the exam, you want to be familiar with the diagnostic criteria for both Bipolar Disorders.

a. CORRECT The diagnosis of Bipolar II Disorder requires the presence or history of major depressive and hypomanic episodes.
b. Incorrect See explanation above.
c. Incorrect See explanation above.
d. Incorrect See explanation above.

The correct answer is: at least one major depressive episode and one hypomanic episode.

How well did you know this?
1
Not at all
2
3
4
5
Perfectly
56
Q

A 22-month old refers to all vehicles - e.g., cars, buses, and trucks - as “cars.” However, when asked to point to a picture of a bus or truck, she is able to do so. This discrepancy:
Select one:

A.
represents normal language development.

B.
suggests giftedness in the area of language development.

C.
is an early sign of autism.

D.
is an early sign of a language production disorder.

A

Children’s production and comprehension of language develop at different rates.
a. CORRECT Overextension (applying a word to a wider collection of objects or events than is appropriate) is a normal part of language acquisition. In addition, young children exhibit greater understanding (comprehension) of language than is suggested by their production of language.

b. Incorrect See explanation above.
c. Incorrect See explanation above.
d. Incorrect See explanation above.

The correct answer is: represents normal language development.

How well did you know this?
1
Not at all
2
3
4
5
Perfectly
57
Q

A university freshman would be best advised to rely primarily on which of the following when studying for his Psychology 101 final exam?
Select one:

A.
maintenance rehearsal

B.
stimulus coding

C.
imagery

D.
elaborative rehearsal

A

In this situation, the student will have to memorize and understand fairly complex information.
a. Incorrect Maintenance rehearsal involves simply repeating information with little or no processing and is less effective than elaborative rehearsal.

b. Incorrect Stimulus coding refers to connecting an image with a part of a word (e.g., the first syllable). It is especially good for paired-associate tasks such as foreign language learning (matching an English word to its foreign-language equivalent).
c. Incorrect Imagery is a useful technique but, for a large amount of complex material, elaboration is better.
d. CORRECT Elaborative rehearsal involves actively analyzing information and relating it to previously stored data. It is the best way to encode information into long-term memory.

The correct answer is: elaborative rehearsal

How well did you know this?
1
Not at all
2
3
4
5
Perfectly
58
Q

Neuropsychiatric symptoms are common sequelae of a stroke (cerebrovascular accident); and, of these symptoms, ________ is/are the most common.
Select one:

A.
anxiety

B.
delusions and hallucinations

C.
depressed mood

D.
disinhibition and euphoria

A

Individuals who have had a stroke are at risk for a variety of neuropsychiatric symptoms.
a. Incorrect Anxiety is fairly common but not as common as depression. Also, when anxiety occurs, it is often comorbid with depression.

b. Incorrect Delusions and hallucinations occur following a stroke but are uncommon.
c. CORRECT The experts generally agree that, of the behavioral and emotional symptoms associated with stroke, depression is most common, although estimates of the occurrence of depression vary. With regard to depressed mood, a recent study reports a prevalence rate of 61% in post-stroke patients [J. A. Bourgeois et al., Poststroke neuropsychiatric illness: An integrated approach to diagnosis and management, Current Treatment Options in Neurology, 6(5), 403-4200, 2004]. However, other studies report lower prevalence rates ranging from about 30 to 40%.
d. Incorrect Disinhibition and euphoria occur following a stroke but are uncommon.

The correct answer is: depressed mood

How well did you know this?
1
Not at all
2
3
4
5
Perfectly
59
Q

Research by Sue and his colleagues (1991) found that matching therapist and client in terms of race/ethnicity:
Select one:

A.
improves treatment outcomes and reduces premature termination for Asian, Hispanic, African American, and Caucasian clients.

B.
reduces premature termination for Asian, Hispanic, African American, and Caucasian clients but has no effect for members of these groups on other treatment outcomes.

C.
improves treatment outcomes and reduces premature termination for members of some groups but not for others.

D.
has no effect on treatment outcomes or premature termination for Asian, Hispanic, African American, or Caucasian clients.

A

Research by Sue et al. (1991) and others has produced inconsistent results with regard to the impact of therapist-client matching and treatment outcomes.
a. Incorrect See explanation for response c.

b. Incorrect See explanation for response c.
c. CORRECT For example, Sue et al. found that therapist-client matching in terms of race/ethnicity improved outcomes for Asian and Hispanic clients and reduced premature termination rates for Caucasian clients.
d. Incorrect See explanation for response c.

The correct answer is: improves treatment outcomes and reduces premature termination for members of some groups but not for others.

How well did you know this?
1
Not at all
2
3
4
5
Perfectly
60
Q

Major Neurocognitive Disorder due to HIV infection has been linked to abnormalities in which of the following structures of the brain?
Select one:

A.
putamen and ventral striatum

B.
suprachiasmatic nucleus

C.
superior and inferior colliculi

D.
entorhinal cortex

A

Knowing that Major Neurocognitive Disorder due to HIV infection includes impairments in motor functioning (e.g., tremors and clumsiness) would have helped you identify the correct answer to this question, even if you are unfamiliar with the research on this issue.
a. CORRECT Wang et al. found that Major Neurocognitive Disorder due to HIV infection is associated with decreased dopamine transporters in the putamen and ventral striatum. These structures are part of the basal ganglia, which is involved in voluntary motor activity. [G. J. Wang et al., Decreased brain dopaminergic transporters in HIV-associated dementia patients, Brain, 127(11), 2452-2458, 2004]

b. Incorrect The suprachiasmatic nucleus (SCN) is part of the hypothalamus and mediates the sleep-wake cycle and other circadian rhythms.
c. Incorrect The superior and inferior colliculi are components of the midbrain. The inferior colliculus plays a role in audition, while the superior colliculus is involved in vision.
d. Incorrect Neuron loss and other abnormalities in the entorhinal cortex have been linked to Alzheimer’s disease.

The correct answer is: putamen and ventral striatum

How well did you know this?
1
Not at all
2
3
4
5
Perfectly
61
Q

Dr. Watson is one of three organizational psychologists who have been hired by a large corporation to conduct research on the selection of top-level managers. The psychologists’ research has provided some evidence that “anal retentive” managers are the least effective managers in the company. Prior to submitting the project report, which recommends that the company screen candidates for top management positions for “anal retentiveness,” Dr. Watson approaches the Project Director, who is also an industrial psychologist, and argues that, although there is some evidence that there is a relationship between anal retentiveness and management effectiveness, the evidence is insufficient to justify eliminating potential managers on the basis of this characteristic alone. Despite Dr. Watson’s protest, the Project Director refuses to review the report and submits it to the company. Dr. Watson decides to go to the company’s management to point out the possible limitations of the report’s recommendations. Which of the following statements best describes the ethical issue in this situation?
Select one:

A.
Watson clearly acted unethically by directly contacting management and thereby compromising the Project Director’s relationship with the company.

B.
Watson acted unethically by not consulting with the Ethics Committee before taking further action against the Project Director after his refusal to review the report.

C.
Watson acted unethically by directly contacting management and the Project Director acted unethically by not reviewing the report.

D.
Watson acted ethically but the Project Director acted unethically by not reviewing the report.

A

Watson acted ethically and responsibly by attempting to prevent distortion of psychological findings, while the Project Director acted unethically by not reviewing the data contained in the report in light of Watson’s protest and thereby possibly providing the company with misleading information.
a. Incorrect Although direct contact with company mananagement may seem extreme, this is not the best answer of those given.

b. Incorrect Watson’s responsibility did not stop after discussing the matter with the Project Director.
c. Incorrect Although the Project Director acted unethically, Watson’s behavior was ethical.
d. CORRECT This response is most consistent with the requirements of Standards 1.04 and 1.05 of the Ethics Code (which apply to resolving ethical violations by colleagues) and Standard 5.01 (which requires psychologists to avoid making false, deceptive, or misleading statements).

The correct answer is: Watson acted ethically but the Project Director acted unethically by not reviewing the report.

How well did you know this?
1
Not at all
2
3
4
5
Perfectly
62
Q

If you believe that, during a full moon, people tend to act “crazier,” you will be likely to notice incidents of bizarre behavior during a full moon and ignore similar incidents when the moon is not full. This tendency is referred to as the:
Select one:

A.
confirmation bias.

B.
saliency bias.

C.
self-fulfilling prophecy bias.

D.
correspondence bias.

A

In this situation, you are noticing events that confirm your beliefs.
a. CORRECT This tendency is referred to as the confirmation bias.

b. Incorrect The saliency bias occurs when a vivid (salient) stimulus in a situation is viewed as the cause of behavior in that situation.
c. Incorrect The self-fulfilling prophecy effect occurs when one’s expectations alter the behaviors of the targets of those expectations.
d. Incorrect The correspondence bias is another name for the fundamental attribution bias, which is the tendency to attribute the behaviors of others to dispositional factors.

The correct answer is: confirmation bias.

How well did you know this?
1
Not at all
2
3
4
5
Perfectly
63
Q

When providing assessment services via video chat, what is one of the primary concerns?
Select one:

A.
Quality of therapeutic services being delivered

B.
Privacy and security as the assessment is being administered

C.
Potential technical difficulties that could arise during the assessment

D.
Client knowledge of the technology, or lack thereof, that could impact the quality of the services being provided

A

The correct answer is B. With assessments and evaluations being performed online, there needs to be greater security and privacy. Access can be severely limited to a select few if necessary, but also the drawback to this has been an increase to cyber infiltration threatening individuals privacy and personal records.

Answers A, C and D are incorrect as quality of service can still be maintained using long distance technology, and although technical difficulties and user understanding are potential difficulties, security and privacy are a greater concern in this new computer technological age.

The correct answer is: Privacy and security as the assessment is being administered

How well did you know this?
1
Not at all
2
3
4
5
Perfectly
64
Q

Dr. Rafael, a licensed psychologist, requires his Psychology 101 students to participate in two research studies conducted by graduate students or professors at the university as part of the course requirements. This requirement:
Select one:

A.
is unethical under any circumstances.

B.
is ethical only if students are aware of the requirement prior to enrolling in the class.

C.
is ethical only if students are given a choice of alternative assignments.

D.
is ethical as long as the research has been approved by the university’s Institutional Review Board.

A

This issue is addressed in Standard 8.04(b) of the APA’s Ethics Code.
a. Incorrect See explanation for response c.

b. Incorrect See explanation for response c.
c. CORRECT This answer is consistent with Standard 8.04(b), which states that, “when research participation is a course requirement or an opportunity for extra credit, the prospective participant is given the choice of equitable alternative activities.”
d. Incorrect See explanation for response c.

The correct answer is: is ethical only if students are given a choice of alternative assignments.

How well did you know this?
1
Not at all
2
3
4
5
Perfectly
65
Q

Which of the following statements illustrates animistic thinking?
Select one:

A.
Step on a crack, break your mother’s back.

B.
My knees are shaking, so I must be scared.

C.
At night the sun takes a nap.

D.
The man on the TV is talking to me.

A

Animistic thinking is characteristic of the preoperational stage of cognitive development.
a. Incorrect This illustrates precausal reasoning, which is also characteristic of the preoperational stage and which underlies animism.

b. Incorrect This illustrates the James-Lange theory of emotion.
c. CORRECT Animistic thinking is the tendency to attribute life (and life-like qualities) to inanimate objects.
d. Incorrect This sounds like a delusion of reference.

The correct answer is: At night the sun takes a nap.

How well did you know this?
1
Not at all
2
3
4
5
Perfectly
66
Q

Pavlov produced “experimental neurosis” in experimental dogs by using which of the following procedures?
Select one:

A.
applying negative reinforcement each time the dogs ate

B.
pairing eating with an aversive tone

C.
making it impossible for the dogs to escape electrical shock

D.
requiring the dogs to make difficult discriminations

A

Experimental neurosis, a phenomenon described by Pavlov, is manifested by restless, aggressive, and fearful responses.
a. Incorrect Negative reinforcement increases the occurrence of the response it follows. Negative reinforcement is an operant conditioning technique and was not used by Pavlov.

b. Incorrect Pairing a response with an aversive stimulus (e.g., alcohol consumption with electric shock) is referred to as aversive counterconditioning and results in a decrease of the response.
c. Incorrect Unavoidable shock was administered to animals by Seligman in his studies on learned helplessness.
d. CORRECT Stimulus discrimination refers to an organism’s ability to discriminate between stimuli and thereby respond only to the conditioned stimulus with a conditioned response. While conducting studies on stimulus discrimination, Pavlov discovered that extremely difficult discriminations elicited behaviors characteristic of human neurosis.

The correct answer is: requiring the dogs to make difficult discriminations

How well did you know this?
1
Not at all
2
3
4
5
Perfectly
67
Q

A patient with Alzheimer’s dementia would be expected to perform BEST on which of the following WAIS-IV Indexes?
Select one:

A.
Perceptual Reasoning

B.
Processing Speed

C.
Working Memory

D.
Verbal Comprehension

A

Answer D is correct: The WAIS-IV provides scores on the four Indexes that are given as responses to this question. The Verbal Comprehension Index provides information on verbal conceptualization, knowledge, and experience. Knowing that patients with Alzheimer’s dementia do better on verbal subtests than on performance subtests would have helped you identify this as the correct response.

Answer A: Perceptual Reasoning is a measure of nonverbal reasoning.

Answer B: Processing Speed is a measure of response speed. Patients with Alzheimer’s dementia do most poorly on this Index.

Answer C: Working Memory provides information on number ability and sequence processing.

The correct answer is: Verbal Comprehension

How well did you know this?
1
Not at all
2
3
4
5
Perfectly
68
Q

Morphine, an opioid agonist, produces euphoria and other pleasurable feelings primarily by __________ activity in the “reward center” of the brain.
Select one:

A.
enhancing dopamine

B.
inhibiting dopamine

C.
enhancing glutamate

D.
inhibiting glutamate

A

Agonists are drugs that mimic or stimulate the activity of an endogenous neurotransmitter; and opioid agonists are drugs that stimulate activity at receptors stimulated by endorphins and other naturally occurring opioids.
a. CORRECT Naturally occurring opioids and certain opioid agonists such as morphine exert their effects by increasing dopamine activity at opioid receptors in the ventral tegmental area and nucleus accumbens, which have been identified as the primary components of the brain’s reward system.

b. Incorrect See explanation for response a.
c. Incorrect See explanation for response a.
d. Incorrect See explanation for response a.

The correct answer is: enhancing dopamine

How well did you know this?
1
Not at all
2
3
4
5
Perfectly
69
Q

The reduction of parasuicidal behaviors is a primary and explicit target of which of the following?
Select one:

A.
stress inoculation training

B.
self-control therapy

C.
eye movement desensitization and reprocessing

D.
dialectical behavior therapy

A

Parasuicide refers to deliberate burning, cutting, and other self-injurious behaviors and suicidal gestures that do not cause death or are done without the intent to cause death.

a. Incorrect As its name implies, stress inoculation training focuses on helping individuals acquire the skills they need to deal more effectively with stress. Although some of the skills addressed in stress inoculation training might be useful for reducing parasuicidal behaviors, these behaviors are not “a primary and explicit target” of treatment.
b. Incorrect Self-control therapy is a behavioral intervention that is used primarily to reverse undesirable habits (e.g., overeating, smoking) and has also been found to be an effective treatment for depression. However, parasuicidal behaviors are not a primary target of self-control therapy.
c. Incorrect EMDR was initially developed as a treatment for PTSD. Parasuicidal behaviors are not a primary target of EMDR.
d. CORRECT DBT was developed as a treatment for Borderline Personality Disorder and explicitly targets parasuicidal behaviors, a common feature of this disorder. For example, skills training is one component of DBT and includes teaching the client strategies to help regulate his/her emotions and better tolerate distress, which decreases the likelihood of parasuicidal behaviors.

The correct answer is: dialectical behavior therapy

How well did you know this?
1
Not at all
2
3
4
5
Perfectly
70
Q

When using the multiple hurdles approach in personnel selection, a critical decision is:
Select one:

A.
the order in which the predictors will be administered.

B.
how each predictor should be weighted.

C.
whether or not the person administering the criterion should also evaluate predictor performance.

D.
which statistical test to use to determine if results are statistically significant.

A

As its name implies, the multiple hurdles technique involves administering the predictors one at a time.
a. CORRECT To maximize the usefulness of the multiple hurdles technique, predictors must be administered in a logical order (e.g., from least to most difficult or least to most expensive).

b. Incorrect See explanation for response a.
c. Incorrect See explanation for response a.
d. Incorrect See explanation for response a.

The correct answer is: the order in which the predictors will be administered.

How well did you know this?
1
Not at all
2
3
4
5
Perfectly
71
Q

A potential problem with the Premack Principle is that:
Select one:

A.
it is often difficult to identify an alternative behavior that serves the same function as the target behavior.

B.
it may not be possible to apply the reinforcer immediately following the target behavior.

C.
for some people, secondary reinforcers have little value.

D.
it is difficult to extinguish a behavior without simultaneously reinforcing an alternative one.

A

The Premack Principle entails reinforcing a low-frequency behavior with a high-frequency behavior or activity.
a. Incorrect See explanation for response b.

b. CORRECT The high-frequency behavior or activity may not be immediately available, which can reduce the effectiveness of the intervention since positive reinforcement is most effective when the reinforcer can be delivered immediately following the target behavior.
c. Incorrect See explanation for response b.
d. Incorrect See explanation for response b.

The correct answer is: it may not be possible to apply the reinforcer immediately following the target behavior.

How well did you know this?
1
Not at all
2
3
4
5
Perfectly
72
Q

Higher scores on the Gender Role Conflict Scale (GRCS) have been linked to a number of negative outcomes including lower self-esteem, more symptoms of depression, and more somatic complaints. For example, a study by Good et al. (1995) found that, for male college students, scores on which of the following subscales of the GRSC are most predictive of scores on a measure of psychological distress?
Select one:

A.
conflicts between work and family relations

B.
restrictive affectionate behavior between men

C.
success, power, and competition

D.
restrictive emotionality

A

This is a very difficult question because it requires you to be familiar with the results of a specific research study. Fortunately, you won’t encounter too many questions like this one on the EPPP. The GRCS was designed as a measure of men’s gender role conflict and provides scores on the four subscales listed in the answers to this question. For their sample of male undergraduates, Good et al. found that scores on the restrictive emotionality subscale were most predictive of scores on the Global Severity Index of the Symptom Checklist-90-Revised. Based on these results, the researchers conclude that men may experience psychological distress as a result of “their beliefs about the need for restrictive emotionality as part of male gender role expectations” (p. 8). [G. E. Good et al., Male gender role conflict: Psychometric issues and relations to psychological distress, Journal of Counseling Psychology, 42(1), 3-10, 1995.]

The correct answer is: restrictive emotionality

How well did you know this?
1
Not at all
2
3
4
5
Perfectly
73
Q

The DSM-5 diagnoses of Bipolar I and Bipolar II Disorder both require that the person has had at least one episode involving an abnormally persistent elevated, expansive, or irritable mood:
Select one:

A.
and persistently increased goal-directed activity or energy.

B.
with a lack of awareness of his/her illness and the need for treatment.

C.
that is sufficiently severe to cause a marked impairment in functioning or require hospitalization.

D.
and at least one episode involving a depressed mood or a loss of interest or pleasure in all or most activities.

A

Answer A is correct: The DSM-5 diagnostic criteria for Bipolar I and Bipolar II disorder include at least one episode involving an abnormally persistent elevated, expansive, or irritable mood with the presence of persistently increased goal-directed activity or energy.

Answer B: A lack of awareness of the illness and the need for treatment is an associated symptom of Bipolar I disorder but is not a diagnostic criterion for either disorder.

Answer C: For a diagnosis of Bipolar I Disorder, symptoms must be sufficiently severe to cause marked impairment in functioning or hospitalization to prevent harm to self or others or include psychotic features. For a diagnosis of Bipolar II Disorder, symptoms must not be severe enough to cause marked impairment in functioning or necessitate hospitalization.

Answer D: The diagnosis of Bipolar II Disorder requires the presence of both hypomanic and major depressive episodes, but the diagnosis of Bipolar I Disorders requires only the presence of one or more manic episodes (although it may include hypomanic and/or major depressive episodes).

The correct answer is: and persistently increased goal-directed activity or energy.

How well did you know this?
1
Not at all
2
3
4
5
Perfectly
74
Q

Dr. Colombu, a licensed psychologist, is working in a community mental health center. The parents of a 15-year old girl bring their daughter for counseling. The parents will, of course, be paying Dr. Colombu’s fee. As an ethical psychologist, Dr. Colombu should:
Select one:

A.
caution the girl that, since she is a minor, anything she says may be told to her parents.

B.
advise the parents that he is required to maintain confidentiality and that he will not be able to advise them of anything the girl says during therapy sessions without her consent.

C.
clearly establish with the girl and her parents prior to therapy what the policy will be with regard to sharing information with the parents.

D.
use his discretion with regard to what information to reveal to the girl’s parents and what information to keep confidential.

A

The therapeutic relationship would be best served if the policy regarding confidentiality is established with both the parents and the child at the onset of therapy.
a. Incorrect Although this may not violate ethical or legal guidelines, answer c describes a better course of action.

b. Incorrect See explanation for response c.
c. CORRECT Parents have a legal right to information about their child in most situations but providing them with all information is not necessarily the best course of action from

a clinical standpoint. Therefore, this is the best course of action since it takes the welfare of the child as well as the legal rights of the parents into consideration.

d. Incorrect See explanation given above.

The correct answer is: clearly establish with the girl and her parents prior to therapy what the policy will be with regard to sharing information with the parents.

How well did you know this?
1
Not at all
2
3
4
5
Perfectly
75
Q

According to Fiedler’s contingency model, leadership is most effective when the leader’s style (task- or person-oriented) matches which of the following?

Select one:

A.
level of employee “maturity”

B.
organizational goals

C.
degree of situational control

D.
role expectations

A

Fiedler’s contingency model distinguishes between two types of leaders - high-LPC and low-LPC. High LPC leaders are person-oriented and are concerned primarily with maintaining good interpersonal relationships, while low LPC leaders are task-oriented and are most concerned about successful task performance.

Answer C is correct: Fiedler’s contingency model predicts that task-oriented (low-LPC) leaders are most effective when the situation is extreme - that is, when the leader has either low or high situational control, while person-oriented (high-LPC leaders) are most effective when situational control is moderate. As defined by Fiedler, situational control is determined by three factors: leader-member relations, task structure, and leader position power.

The correct answer is: degree of situational control

How well did you know this?
1
Not at all
2
3
4
5
Perfectly
76
Q

Long-term stress can result in excessive production of ___________ which, in turn, can suppress the body’s immune system.
Select one:

A.
antigens

B.
interferons

C.
corticosteroids

D.
anabolic steroids

A

Elevated levels of cortisol and other stress hormones lower resistance to disease by compromising the body’s immune system.
a. Incorrect Antigens are substances (e.g., bacteria, viruses) that stimulate the body’s production of antibodies.

b. Incorrect Interferons are antiviral proteins that are produced by certain cells in the body in response to exposure to a virus.
c. CORRECT Chronic stress leads to the release of corticosteroids (cortisol and corticosterone) by the adrenal cortex. Continually high levels of corticosteroids cause suppression of the immune system, apparently by interfering with the production of antibodies and T-cells.
d. Incorrect Anabolic steroids are manufactured substances that mimic male sex hormones.

The correct answer is: corticosteroids

How well did you know this?
1
Not at all
2
3
4
5
Perfectly
77
Q

In the 1950s, researchers at Ohio State University developed an influential theory of leadership that focused on a leader’s:
Select one:

A.
personality traits.

B.
behaviors.

C.
attitudes toward subordinates.

D.
power.

A

Results of the Ohio State leadership studies indicated that leaders can be described in terms of two behavioral dimensions - consideration and initiating structure.
a. Incorrect See explanation for response b.

b. CORRECT The Ohio State group focused on the behaviors exhibited by supervisors and other organizational leaders.
c. Incorrect See explanation for response b.
d. Incorrect See explanation for response b.

The correct answer is: behaviors.

How well did you know this?
1
Not at all
2
3
4
5
Perfectly
78
Q

The use of functional brain imaging techniques to study ADHD has linked its symptoms to lower-than-normal levels of metabolic activity in the:
Select one:

A.
prefrontal cortex and basal ganglia.

B.
prefrontal cortex and ARAS.

C.
entorhinal cortex and SCN.

D.
entorhinal cortex and thalamus.

A

Knowing the functions of the brain areas listed in the responses and the core symptoms of ADHD would have helped you choose the correct answer to this question even if you are unfamiliar with the brain-imaging research.

a. CORRECT The prefrontal cortex is involved in executive cognitive functions (e.g., planning, decision-making, attention), while the basal ganglia are involved in the regulation of movement and consist of several structures including the caudate nucleus and globus pallidus. Abnormalities in these structures have been linked to ADHD.
b. Incorrect See explanation above.
c. Incorrect See explanation above.
d. Incorrect See explanation above.

The correct answer is: prefrontal cortex and basal ganglia.

How well did you know this?
1
Not at all
2
3
4
5
Perfectly
79
Q

A drug that ____________ would be most useful for treating Bulimia Nervosa.
Select one:

A.
increases sensitivity to cortisol

B.
reduces glutamate levels

C.
decreases sensitivity to acetylcholine

D.
increases serotonin levels

A

Bulimia Nervosa has been linked to lower-than-normal levels of serotonin.

a. Incorrect See explanation for response d.
b. Incorrect See explanation for response d.
c. Incorrect See explanation for response d.
d. CORRECT Bulimia has been found to be responsive to drugs that increase serotonin levels (e.g., an SSRI).

The correct answer is: increases serotonin levels

How well did you know this?
1
Not at all
2
3
4
5
Perfectly
80
Q

Research evaluating the impact of child sexual abuse on the adjustment of males and females has:
Select one:

A.
consistently found that negative outcomes are essentially the same for males and females in terms of both nature and severity.

B.
consistently found that negative outcomes are worse for males than for females in terms of both nature and severity.

C.
consistently found that negative outcomes are worse for females than for males in terms of both nature and severity.

D.
produced inconsistent results, with some studies showing that females have more negative outcomes and others finding that the outcomes are similar for males and females.

A

Like much research in psychology, studies on the outcomes of child sexual abuse have not produced consistent results.
a. Incorrect See explanation for response d.

b. Incorrect See explanation for response d.
c. Incorrect See explanation for response d.
d. CORRECT Currently, there is no clear consensus about the outcomes of child sexual abuse for male and female victims. A number of studies have found no consistent gender differences; but, when differences are found, the outcomes are worse for females than for males.

The correct answer is: produced inconsistent results, with some studies showing that females have more negative outcomes and others finding that the outcomes are similar for males and females.

How well did you know this?
1
Not at all
2
3
4
5
Perfectly
81
Q

Why does the most pervasive modern form of supervision tend to be the triangular model?
Select one:

A.
Because organizational policies and professional knowledge form the foundation, while the relationship with the supervisor forms the core.

B.
Because the relationship with the supervisor and professional experience form the foundation while care of clients forms the core.

C.
Because the relationship with the clients and counseling theory form the foundation and the organizational policies form the core.

D.
Because the relationship with the organization and the supervisor literature form the foundation and personal experience form the core.

A

The correct answer is A. One of the common modern forms of supervision is a triangular model where the organizational policies and professional knowledge form the foundation, the supervisor relationship is at the core, and the ultimate emphasis on providing service to the clients. Answers B, C and D are incorrect as these various answer models are not models of supervision based on the triangular model.

The correct answer is: Because organizational policies and professional knowledge form the foundation, while the relationship with the supervisor forms the core.

How well did you know this?
1
Not at all
2
3
4
5
Perfectly
82
Q

According to Kohlberg, the moral judgments of preschool children are ordinarily based on:
Select one:

A.
the dictates of the superego.

B.
role conformity.

C.
an inner sense of right and wrong.

D.
the consequences of the act.

A

Kohlberg’s theory of moral development distinguishes between three stages, with each stage including two substages. The three stages are preconventional, conventional, and postconventional. Preschool children are in the preconventional stage of moral development and base their moral judgments on the consequences of actions.

a. Incorrect Kohlberg did not refer to the superego.
b. Incorrect This is more characteristic of individuals in the conventional stage.
c. Incorrect Kohlberg did not describe moral development in terms of an “inner sense.”

The correct answer is: the consequences of the act.

How well did you know this?
1
Not at all
2
3
4
5
Perfectly
83
Q

Research on the universality of the “Big Five” personality traits has generally found that the traits are cross-culturally valid. However, of the five traits, empirical evidence for the universality of __________ is least compelling.
Select one:

A.
openness to experience

B.
neuroticism

C.
conscientiousness

D.
extraversion

A

The Big Five personality traits are openness to experience, conscientiousness, extraversion, agreeableness, and neuroticism. (A useful way for remembering the Big Five is with the acronym “OCEAN.”)
a. CORRECT The best general conclusion about the Big Five is that they are universal. However, factor analyses have not been entirely consistent, with evidence for openness to experience showing the weakest replicability in some Asian and other non-Western countries.

b. Incorrect See explanation for response a.
c. Incorrect See explanation for response a.
d. Incorrect See explanation for response a.

The correct answer is: openness to experience

How well did you know this?
1
Not at all
2
3
4
5
Perfectly
84
Q

A child with Autism Spectrum Disorder would most likely obtain a higher score than a “normal” peer on which of the following?
Select one:

A.
Embedded Figures Test

B.
Wisconsin Card Sorting Test

C.
Tower of London Test

D.
WISC-V Arithmetic subtest

A

A number of tests have been found useful for evaluating the abilities of individuals with Autism Spectrum Disorder.
a. CORRECT Children with autism often show superior performance on the Embedded Figures Test, which measures field dependence/independence.

b. Incorrect Children with autism tend to do poorly on the WCST.
c. Incorrect Children with autism also receive low scores on the Tower of London Test.
d. Incorrect Children with autism tend to receive a low score on the Arithmetic subtest of the WISC-V.

The correct answer is: Embedded Figures Test

How well did you know this?
1
Not at all
2
3
4
5
Perfectly
85
Q

Self-in-relation therapy is an approach to:
Select one:

A.
family therapy that emphasizes the multiple transferences that affect current relationships among family members.

B.
group therapy that focuses on interpersonal interpretations and misinterpretations that affect individual functioning.

C.
feminist therapy that emphasizes the role of the mother-son versus mother-daughter relationship in creating gender differences in behavior.

D.
biopsychosocial therapy that recognizes the interacting impact of biological and sociocultural factors on male and female development.

A

Self-in-relation therapy was developed at the Stone Center for Developmental Services and Studies at Wellesley College. See, e.g., J. V. Jordan et al., Women’s growth in connection: Writings from the Stone Center, Guilford Press, NY, 1991.
a. Incorrect See explanation for response c.

b. Incorrect See explanation for response c.
c. CORRECT Object relations theory was an important influence on self-in-relation theory. However, self-in-relation theory extends the object relations approach by considering the impact of same versus opposite gender in caregiver-infant relationships on development, especially on development of the relational self.
d. Incorrect See explanation for response c.

The correct answer is: feminist therapy that emphasizes the role of the mother-son versus mother-daughter relationship in creating gender differences in behavior.

How well did you know this?
1
Not at all
2
3
4
5
Perfectly
86
Q

During the course of a mental status exam, a 36-year old man takes a great deal of time to answer the examiner’s questions because he often focuses on unnecessary details and makes parenthetical remarks. When he is not interrupted, the man usually does get around to answering the examiner’s question. The man’s speech illustrates which of the following?
Select one:

A.
circumstantiality

B.
loosening of associations

C.
confabulation

D.
derailment

A

The important thing to note in the description of the man’s speech is that he does eventually get back to the point.
a. CORRECT Circumstantiality refers to speech that is indirect and delayed in reaching the point because of unnecessary, tedious details and parenthetical remarks.

b. Incorrect Loosening of associations involves a complete lack of connectedness between utterances and the loss of the original point.
c. Incorrect Confabulation involves a fabrication of facts or events to compensate for memory loss.
d. Incorrect Derailment is another term for loosening of associations.

The correct answer is: circumstantiality

How well did you know this?
1
Not at all
2
3
4
5
Perfectly
87
Q

According to Hamburger et al. (1996), psychopathic personality traits interact with biological sex to produce Antisocial Personality Disorder in males and __________ Personality Disorder in females.
Select one:

A.
Histrionic

B.
Narcissistic

C.
Borderline

D.
Schizoid

A

The DSM Cluster B personality disorders share a number of characteristics and sometimes co-occur but differ in terms of gender ratios.

a. CORRECT Hamburger et al. proposed that Antisocial Personality Disorder (APD) and Histrionic Personality Disorder (HPD) represent gender-related manifestations of the same biological predisposition toward psychopathy. Note, however, that the research has not been entirely supportive of this hypothesis. [M. E. Hamburger, S. O. Lilienfeld, and M. Hogben, Psychopathy, gender, and gender roles: Implications for antisocial and histrionic personality disorders, Journal of Personality Disorders, 10(1), 41-55, 1996.]
b. Incorrect See explanation for response a.
c. Incorrect APD and Borderline Personality Disorder (BPD) also share a number of symptoms (e.g., impulsivity, manipulativeness, and problems with relationships) and are viewed by some experts as gender-related manifestations of a similar pathology. However, this question is asking about Hamburger et al.’s hypothesis, which describes APD and HPD (not BPD) as sharing underlying psychopathic personality traits.
d. Incorrect See explanation for response a.

The correct answer is: Histrionic

How well did you know this?
1
Not at all
2
3
4
5
Perfectly
88
Q

A new selection test is apparently having an adverse impact on females. To determine if the adverse impact is being caused by “unfairness,” you would check:

Select one:

A.
the criterion (performance) scores obtained by males and females.

B.
the slopes of the regression lines for males and females.

C.
to see if the language used in the test is biased.

D.
to see if the measure of job performance is biased.

A

As described in the Industrial-Organizational Psychology chapter of the written study materials. adverse impact can be the result of several factors including differential validity and unfairness.<p>a. CORRECT As defined by the EEOC, unfairness occurs when the validity coefficients and criterion performance for two groups are similar but members of the groups perform differently on the predictor.</p><p>b. Incorrect The slopes of the regression lines would be useful for determining if adverse impact is due to differential validity.</p><p>c. Incorrect See explanation above.</p><p>d. Incorrect See explanation above.

The correct answer is: the criterion (performance) scores obtained by males and females.
</p>

How well did you know this?
1
Not at all
2
3
4
5
Perfectly
89
Q

A 42-year-old woman who has just begun taking lithium carbonate as a treatment for Bipolar Disorder will most likely experience which of the following side effects?
Select one:

A.
constipation, abdominal pain, and excessive salivation

B.
nausea, vomiting, diarrhea, and abdominal pain

C.
increased appetite, carbohydrate craving, and weight gain

D.
nausea, vomiting, constipation, and salt craving

A

Lithium may produce a number of unpleasant gastrointestinal side effects.
a. Incorrect See explanation for response b.

b. CORRECT Nausea, vomiting, diarrhea, abdominal pain, anorexia, and dry mouth are common early gastrointestinal side effects of lithium. These symptoms usually appear within a few weeks and, if they occur late in therapy, may be a sign of lithium toxicity.
c. Incorrect See explanation for response b.
d. Incorrect See explanation for response b.

The correct answer is: nausea, vomiting, diarrhea, and abdominal pain

How well did you know this?
1
Not at all
2
3
4
5
Perfectly
90
Q

A 33-year-old man tells his therapist that, since his involvement in a serious car accident two months ago, he has been feeling sad every day; has had no appetite; feels tired all of the time and sleeps for 10 to 12 hours each night; isn’t interested in spending time with his friends and family; and feels guilty about the accident, even though it wasn’t his fault. He also says that, because he’s having trouble concentrating at work and getting to work on time, he’s afraid he’s going to be fired. Based on these symptoms, the most likely diagnosis is:
Select one:

A.
Bipolar II Disorder.

B.
Major Depressive Disorder.

C.
Adjustment Disorder.

D.
Acute Stress Disorder.

A

The nature, number, and duration of the man’s symptoms suggest that the best diagnosis is Major Depressive Disorder.

a. Incorrect See explanation for response b.
b. CORRECT The nature, number, and duration of the man’s symptoms are consistent with a diagnosis of Major Depressive Disorder: He is experiencing a depressed mood and has at least five other characteristic symptoms of this disorder; his symptoms have lasted for more than two weeks; and he is experiencing impaired occupational functioning as the result of his symptoms.
c. Incorrect See explanation for response b.
d. Incorrect See explanation for response b.

The correct answer is: Major Depressive Disorder.

How well did you know this?
1
Not at all
2
3
4
5
Perfectly
91
Q

A former therapy client calls you and expresses an interest in becoming friends. The former client suggests that you meet at a local restaurant for lunch. You remember that the client shared many of your interests, and that, at the time of therapy, you were aware that he/she was someone you would like to know better. As an ethical psychologist, you:
Select one:

A.
meet the former client for lunch only if the therapeutic relationship was terminated more than two years ago.

B.
meet the former client for lunch only if the therapeutic relationship was terminated more than two years ago and you believe that a relationship is not clinically contraindicated and is not likely to be exploitative.

C.
meet the client for lunch to clarify and discuss any potential conflicts and then proceed with caution.

D.
do not meet the client for lunch and explain why a friendship would not be in the client’s best interests.

A

Friendships with former clients are covered by the multiple relationship clause in the Ethics Code. In most situations, a psychologist would want to avoid friendships with former clients.
a. Incorrect See explanation for responses b and d.

b. Incorrect The two-year limit applies to sexual relationships, which this question is not addressing.
c. Incorrect See explanation for response d.
d. CORRECT Most experts agree that friendships with former clients should be avoided since this sets up an unequal situation and precludes the possibility that the former client can return for therapy in the future.

The correct answer is: do not meet the client for lunch and explain why a friendship would not be in the client’s best interests.

92
Q

Providing social skills training to children whose aggressive behaviors have already had a substantial adverse impact on their family and peer relations and school performance is an example of:
Select one:

A.
primary prevention.

B.
secondary prevention.

C.
tertiary prevention.

D.
behavioral prevention.

A

Three types of prevention are distinguished in the literature - primary, secondary, and tertiary.
a. Incorrect Primary preventions are aimed at reducing the development and incidence of mental disorders.

b. Incorrect Secondary preventions target individuals who have just begun to exhibit symptoms of a disorder in order to prevent the disorder from becoming more serious.
c. CORRECT Tertiary preventions are aimed at individuals who have already developed a disorder. Their goal is to reduce the risk for chronicity and relapse and any other problems related to the disorder.
d. Incorrect This is a “made up” term.

The correct answer is: tertiary prevention.

93
Q

Research has shown that membership in a cohesive (versus noncohesive) group is associated with:
Select one:

A.
greater productivity, greater satisfaction, and reduced turnover.

B.
greater satisfaction, reduced turnover and, in some situations, greater productivity.

C.
greater productivity but no significant impact on satisfaction or turnover.

D.
in the long run, reduced productivity, lower satisfaction, and increased turnover.

A

Group cohesiveness has been linked to a number of benefits and liabilities.
a. Incorrect See explanation for response b.

b. CORRECT Group cohesiveness has been consistently linked to reduced absenteeism and turnover and higher levels of satisfaction and commitment by the research, but its association with productivity is moderated by a number of factors (e.g., the group’s norms regarding productivity).
c. Incorrect See explanation for response b.
d. Incorrect See explanation for response b.

The correct answer is: greater satisfaction, reduced turnover and, in some situations, greater productivity.

94
Q

Global aphasia involves:
Select one:

A.
intact comprehension, nonfluent speech, and deficits in naming and repetition.

B.
impaired comprehension, nonfluent speech, and deficits in naming and repetition.

C.
impaired comprehension, fluent speech, and paraphasia.

D.
intact comprehension, fluent speech, and paraphasia.

A

The symptoms listed in the four answers to this question are characteristic of four types of aphasia that you want to be familiar with for the licensing exam.
a. Incorrect These symptoms are characteristic of Broca’s aphasia.

b. CORRECT As its name suggests, global aphasia is caused by widespread brain injury and is characterized by extensive disruption in the ability to produce and understand language. Impaired comprehension, impaired speech, anomia (inability to name a common or familiar object, attribute, or action), and paraphasia (substitution of a word with a sound or incorrect word) are symptoms of this type of aphasia.
c. Incorrect These symptoms are characteristic of Wernicke’s aphasia.
d. Incorrect These symptoms are characteristic of conduction aphasia.

The correct answer is: impaired comprehension, nonfluent speech, and deficits in naming and repetition.

95
Q

Borderline Personality Disorder is most commonly diagnosed in individuals ages:
Select one:

A.
19 through 34.

B.
29 through 44.

C.
39 through 54.

D.
49 through 64.

A

Knowing that the symptoms of Borderline Personality Disorder tend to dissipate with increasing age would have helped you identify the correct answer to this question.

a. CORRECT The rates of BPD are highest during the young adult years. Although individuals with this disorder often continue to exhibit some symptoms throughout their life, by age 40, many no longer meet the diagnostic criteria for BPD.
b. Incorrect See explanation for response a.
c. Incorrect See explanation for response a.
d. Incorrect See explanation for response a.

The correct answer is: 19 through 34.

96
Q

A child living in the United States is exposed to both English and a second language between the ages of six months and three years and, as result, becomes fluent in both languages. If the child had not been exposed to the second language until after the age of 4 or 5, she would have had more trouble acquiring the second language. This provides evidence for:
Select one:

A.
the concept of critical periods

B.
the concept of sensitive periods

C.
the Whorfian hypothesis

D.
the notion of imprinting

A

Although some authors use the terms “critical period” and “sensitive period” interchangeably, they actually mean two different things. a. Incorrect A critical period refers to a time when, if certain developmental events don’t happen, later aspects of development will not occur. A sensitive period refers to an optimal period rather than a necessary period. The situation described in this question suggests there is a sensitive period for dual language learning.

The correct answer is: the concept of sensitive periods

97
Q

When using punishment to reduce or eliminate an undesirable behavior, it is important to keep in mind that gradually increasing the intensity or severity of a punishment will likely reduce its effectiveness. This reduction in the effectiveness of punishment is referred to as:
Select one:

A.
satiation.

B.
habituation.

C.
sensitization.

D.
extinction.

A

For the exam, you want to be familiar with the definitions of the terms listed in the answers to this question.
a. Incorrect Satiation occurs when a reinforcer loses its reinforcing value because the individual has received too much reinforcement.

b. CORRECT The question describes habituation which, in the context of punishment, occurs when the punisher loses its effectiveness. Gradually increasing the intensity of a punishment over time is usually ineffective because it leads to habituation.
c. Incorrect In the context of punishment, sensitization occurs when an individual’s response to a neutral (innocuous) stimulus increases after the individual has been exposed to a punishing (aversive) stimulus.
d. Incorrect In operant conditioning, extinction occurs when a previously reinforced response is no longer reinforced and, as a result, the response decreases or no longer occurs.

The correct answer is: habituation.

98
Q

Howard et al.’s (1986) phase model proposes that a client’s progress in therapy occurs in three predictable phases. These are:
Select one:

A.
unfreezing, changing, and refreezing.

B.
remoralization, remediation, and rehabilitation.

C.
engagement, exploration, and evaluation.

D.
contemplation, action, and termination.

A

For the exam, you want to be familiar with Howard et al.’s (1986) phase model and dose dependent effect, which are both described in the Clinical Psychology chapter of the AATBS written study materials.
a. Incorrect These are the three stages in Lewin’s force field model of organizational change.

b. CORRECT As described by Howard et al., remoralization is an improvement in the subjective sense of well-being, remediation is a reduction in symptoms, and rehabilitation involves improvements in overall functioning.
c. Incorrect These are not the phases identified by Howard and colleagues.
d. Incorrect These are three of the six phases of the transtheoretical model of change.

The correct answer is: remoralization, remediation, and rehabilitation.

99
Q

In recent years, psychologists have attempted to become more sensitive to the uniqueness of each culture. This sensitivity is most consistent with:
Select one:

A.
an emic approach.

B.
an etic approach.

C.
an etic-emic synthesis.

D.
neither an etic nor an emic approach.

A

An emic approach is characterized by an awareness that each culture possesses unique characteristics.
a. CORRECT An emic approach recognizes cultural differences and the impact of those differences on behaviors, attitudes, etc.

b. Incorrect An etic approach applies universal principles that are derived from one culture to all cultures.
c. Incorrect See explanation above.
d. Incorrect See explanation above.

The correct answer is: an emic approach.

100
Q

As defined by Bandura, self-efficacy beliefs have four sources. These are:
Select one:

A.
prior accomplishments, observations of others, verbal persuasion, and emotional and physiological states.

B.
external reinforcement, internal (self) reinforcement, vicarious reinforcement, and logical verification.

C.
observation of others, imitation of others, reinforcement by others, and self-reinforcement.

D.
observation, cognitive mediation, rehearsal, and successful performance.

A

Bandura’s (1982, 1986) theory of self-efficacy proposes that people undertake activities they believe they are capable of performing.
a. CORRECT These are the four sources of self-efficacy that Bandura has identified. For additional information on Bandura’s theory, see the Learning Theory chapter of the written study materials.

b. Incorrect See explanation above.
c. Incorrect See explanation above.
d. Incorrect See explanation above.

The correct answer is: prior accomplishments, observations of others, verbal persuasion, and emotional and physiological states.

101
Q

Which of the following best describes the requirements of the ethics codes published by the American and Canadian Psychological Associations?
Select one:

A.
Subjects do not have to be debriefed about the nature of a research study following their participation unless doing so is required by the institutional review board.

B.
Subjects do not have to be debriefed about the nature of a research study following their participation unless the study involved the use of deception.

C.
Subjects must be debriefed about the nature of a research study following their participation in a way that minimizes harm.

D.
Subjects must always be debriefed about the nature of a research study immediately following their participation.

A

The debriefing of research subjects is addressed in Standards 8.07 and 8.08 of the APA’s Ethics Code and Principles II and III of the Canadian Code of Ethics.
a. Incorrect See explanation for answer c.

b. Incorrect See explanation for answer c.
c. CORRECT This answer is most consistent with the requirements of the American and Canadian codes: Standard 8.08(c) of the APA’s Ethics Code states that, “when psychologists become aware [during the debriefing] that research procedures have harmed a participant, they take reasonable steps to minimize the harm.” And Principle II.44 of the Canadian Code of Ethics states that psychologists must “debrief research subjects in such a way that any harm caused can be discerned, and act to correct any resultant harm.”
d. Incorrect Neither code states that debriefing must be provided to subjects immediately following their participation. The Canadian Code of Ethics does not address the timing of debriefing, and Standard 8.07(c) of the APA’s Ethics Code requires psychologists to debrief subjects “as early as is feasible” when the study involved deception, which may be at the conclusion of their participation or at the end of data collection. In addition, Standard 8.08(b) allows psychologists to delay or withhold information that is normally provided during debriefing when there is a scientific or humane justification for doing so.

The correct answer is: Subjects must be debriefed about the nature of a research study following their participation in a way that minimizes harm.

102
Q

The minimum and maximum values of the standard error of estimate are:
Select one:

A.
-1 and +1.

B.
0 and 1.

C.
0 and the standard deviation of the criterion.

D.
0 and the standard deviation of the predictor.

A

Knowing the formula for the standard error of estimate may have helped you identify the correct answer to this question. The formula is given in the Test Construction chapter of the written study materials.
a. Incorrect See explanation for response c.

b. Incorrect See explanation for response c.
c. CORRECT The standard error of estimate equals the standard deviation of Y (the criterion) times the square root of 1 minus the validity coefficient squared. When the validity coefficient is 1, the standard error equals 0 (there is no error), and when the validity coefficient is 0, the standard error equals the standard deviation of the criterion scores.
d. Incorrect See explanation for response c.

The correct answer is: 0 and the standard deviation of the criterion.

103
Q

As defined by Gerald Caplan, the target of consultee-centered case consultation is the consultee’s skills, knowledge, abilities, and/or objectivity. Caplan describes which of the following as a potential cause of a consultee’s lack of objectivity?
Select one:

A.
projective identification

B.
diagnostic overshadowing

C.
parallel process

D.
theme interference

A

Caplan distinguishes between four types of mental health consultation: client-centered case consultation, consultee-centered case consultation, program-centered administrative consultation, and consultee-centered administrative consultation.
a. Incorrect See explanation for response d.

b. Incorrect See explanation for response d.
c. Incorrect See explanation for response d.
d. CORRECT Caplan defines theme interference as a type of transference that occurs when a consultee’s unresolved conflict related to a particular type of client or situation interferes with his/her current performance with similar clients or in similar situations.

The correct answer is: theme interference

104
Q

The ___________ refers to the tendency to ignore relevant statistical data when making a probability judgment and to rely, instead, on irrelevant information.
Select one:

A.
Von Restorff effect

B.
Forer effect

C.
availability heuristic

D.
base rate fallacy

A

Researchers have identified a number of cognitive biases that distort our ability to accurately perceive or interpret reality.
a. Incorrect The Von Restorff effect is the tendency to remember very unusual or unique objects or events (i.e., objects or events that “stick out like a sore thumb”).

b. Incorrect The Forer effect is another name for the Barnum effect and refers to the tendency to accept a vague or very general description of oneself (e.g., a horoscope) as an accurate description.
c. Incorrect The availability heuristic is the tendency to focus on the most salient or affectively charged aspects of a stimulus or situation.
d. CORRECT This question accurately describes the base rate fallacy, which is also known as base rate neglect. In one study on this phenomenon, participants were given data about the distribution of GPA in the population and then asked to estimate the GPA of hypothetical students. Rather than relying on the statistical information, participants usually used irrelevant descriptive information about each hypothetical student to estimate the student’s GPA.

The correct answer is: base rate fallacy

105
Q

Why do animal models of genetic research provide more accurate research than using human participants?
Select one:

A.
It is due to the lack of quality of research that occurs in human trials in the field.

B.
Researchers can manipulate both the environment and the genes in a controlled laboratory setting.

C.
In animal models there are unique chemical agents that more efficaciously interact with many potential experimental methods than human testing models.

D.
There are international rules governing and stating that it is illegal to do any kind of research using human participants.

A

The correct answer is B. Studies using animal models tend to provide more accurate data than studies in humans due to the fact that both genes and environment can be manipulated and controlled in the laboratory setting. Because genes and environments cannot be manipulated in the human species, two quasi-experimental methods are used most often to screen for genetic influence on individual differences in complex traits such as behavior. Answers A, C, and D are incorrect as there is no direct correlation between quality of research and using human participants, there are no unique chemical agents that occur within animal research models, and there are no international rules governing or stating that any kind of research involving human participants is illegal.

The correct answer is: Researchers can manipulate both the environment and the genes in a controlled laboratory setting.

106
Q

When 11-year old Samantha plays with her 5-year old brother Sam, she often assumes the role of “mother” and teaches Sam how to do things. Sam cooperates by doing what Samantha instructs him to do. The interaction between Samantha and her brother is best described as:
Select one:

A.
complementary.

B.
symmetrical.

C.
parallel.

D.
disengaged.

A

In this situation, Samantha is the “teacher” while her younger brother is the “student.”
a. CORRECT Complementary transactions are transactions between unequals - e.g., between a dominant and a subordinate person.

b. Incorrect Symmetrical transactions occur between equals.
c. Incorrect Parallel transactions are a combination of complementary and symmetrical transactions.
d. Incorrect The term “disengaged” is not used to describe the type of transaction presented in this question.

The correct answer is: complementary.

107
Q

When treating a client with a paraphilia, which of the following techniques would be most useful for replacing the inappropriate stimulus that currently elicits sexual arousal with a more appropriate stimulus?
Select one:

A.
covert sensitization

B.
flooding (in vivo exposure)

C.
habit reversal training

D.
orgasmic reconditioning

A

This question is asking about the technique that would change the cue (stimulus) that produces sexual arousal for a client with a paraphilia.

a. Incorrect When using covert sensitization to eliminate an undesirable behavior, the individual repeatedly imagines engaging in that behavior (or a stimulus associated with it) and then imagines an aversive consequence of engaging in the behavior. Eventually, as a result of this pairing, thinking about the undesirable behavior will produce an aversive response. For example, if a person is sexually aroused by exposing himself to women in public places, covert sensitization might involve having him imagine exposing himself to a woman on the subway and then imagine that he is arrested by the police for doing so, convicted of a sex crime, and sent to jail.
b. Incorrect Flooding is used primarily to reduce anxiety responses and has not been found to be an effective treatment for paraphilias.
c. Incorrect Habit reversal training is used to reduce habits such as nail-biting, thumb-sucking, and hair pulling and has not been found to be an effective treatment for paraphilias. Habit reversal training is used to increase the individual’s awareness of his/her habit and replace the habit with alternative behaviors.
d. CORRECT When using orgasmic reconditioning, the client is “reconditioned” to masturbate to a more appropriate stimulus. This is accomplished by having the client begin to masturbate while fantasizing about the inappropriate stimulus and then switch his fantasy to the more appropriate stimulus.

The correct answer is: orgasmic reconditioning

108
Q

As defined by Donald Super, “career maturity”:

Select one:

A.
is achieved by most people by the time they reach their late 20s or early 30s.

B.
is not fully achieved until mid-life or later.

C.
refers to the individual’s mastery of tasks at each developmental stage.

D.
refers to the degree of match between the individual’s “self-concept” and current occupational aspirations.

A

Super’s life space/life span theory includes a stage theory that encompasses the entire lifespan. The five stages are growth, exploration, establishment, maintenance, and disengagement.<p>a. Incorrect See explanation for response c.</p><p>b. Incorrect See explanation for response c.</p><p>c. CORRECT Each stage involves a set of developmental tasks, and mastery of those tasks represents career maturity. Career maturity prepares the individual to fulfill the life roles characteristic of the stage the person is in as well as for the tasks of the subsequent stage.</p><p>d. Incorrect See explanation for response c.

The correct answer is: refers to the individual’s mastery of tasks at each developmental stage.
</p>

109
Q

The fee for a 23-year-old woman suffering from depression is being paid by her parents. In terms of confidentiality, in this situation:
Select one:

A.
the parents are the holders of the privilege since they are paying the therapist’s fee.

B.
the parents should be supplied with general information about their daughter’s progress since they are paying the therapist’s fee and, therefore, are the “users” of the therapist’s services.

C.
the therapist should relate only the information she believes the parents will understand.

D.
the therapist should, at the onset of treatment, establish a policy about confidentiality jointly with the daughter and her parents.

A

In a case similar to the one described in this question (APA Casebook, 1987), the parents filed a complaint against the psychologist when he would not release information to them about their daughter. The Ethics Committee decided that the psychologist acted ethically by not releasing the information since he had clarified this policy in advance with the parents and the daughter at the onset of treatment.
a. Incorrect Payment of fees does not give a party the right to information about a client’s progress in therapy.

b. Incorrect Once again, payment of fees does not mean that the parents need to be given information (specific or general) unless this has been explicitly decided upon by all parties.
c. Incorrect This response does not address the issue of confidentiality presented by the question.
d. CORRECT This is the most ethical alternative and, as noted above, was decided by the Ethics Committee to be an ethical course of action.

The correct answer is: the therapist should, at the onset of treatment, establish a policy about confidentiality jointly with the daughter and her parents.

110
Q

A practitioner of Minuchin’s structural family therapy uses which of the following to alter the hierarchical relationships within a family system or subsystem?
Select one:

A.
unbalancing

B.
tracking

C.
reframing

D.
mimesis

A

For the exam, you want to be familiar with the four techniques listed in the answers to this question.

a. CORRECT Unbalancing is a restructuring technique that is used by structural family therapists to change hierarchical relationships between family members. It may involve affiliating with a family member, ignoring a family member, or entering into a coalition with a family member against another member.
b. Incorrect Tracking is a joining technique and involves following the verbal and nonverbal content of the family’s communications by, for example, asking clarifying questions, showing interest in what they’re saying, and connecting things they’ve said in the past to current interactions.
c. Incorrect Reframing is a paradoxical technique that involves offering an alternative interpretation of the meaning of a behavior or event.
d. Incorrect Mimesis is another joining technique and involves adopting the family’s affective and communication style.

The correct answer is: unbalancing

111
Q

If a psychologist is requested by a defense attorney to evaluate and testify about a defendant for the purpose of establishing the defense of insanity, the psychologist should:
Select one:

A.
obtain an order from the court before agreeing to evaluate the defendant.

B.
evaluate the defendant and testify as requested but inform the defendant of the purpose of the evaluation and of the limits of confidentiality before doing so.

C.
evaluate the defendant as requested and not be concerned about informing the defendant of the limits of confidentiality since privilege is waived in this situation.

D.
refuse to evaluate the defendant because psychologists are not qualified to determine if a person is “insane.”

A

Claiming insanity as a defense is one of the times when privilege is waived. However, precautions regarding confidential information must still be taken.
a. Incorrect This isn’t necessary.

b. CORRECT Although the psychologist would want to refrain from concluding whether or not the defendant is “insane,” he/she can testify about the client’s mental state at the time of the crime. Also, while privilege is waived when a defendant claims insanity, the defendant should be informed of the limits of confidentiality prior to the evaluation.
c. Incorrect See explanation for response b.
d. Incorrect Psychologists should not conclude whether or not a person is “insane,” but they can draw conclusions about a person’s mental status, which the court then uses to make a determination of insanity.

The correct answer is: evaluate the defendant and testify as requested but inform the defendant of the purpose of the evaluation and of the limits of confidentiality before doing so.

112
Q

Results of Pope’s (1992) survey of APA members indicated that which of the following issues was cited most frequently as being ethically troubling or challenging?
Select one:

A.
fees

B.
sexual matters

C.
confidentiality

D.
the conduct of colleagues

A

This is a straightforward question. Unfortunately, it requires you to be familiar with the results of a specific survey, although you might have been able to rely on your own experience and those of your colleagues.
a. Incorrect Fees were mentioned by 14% of respondents as being ethically troublesome in a recent study conducted by S. Pope (Ethical dilemmas as encountered by members of the APA: A national survey, American Psychologist, 47(3), 397-411, 1992).

b. Incorrect Only 4% of respondents listed sexual issues as a problem.
c. CORRECT Confidentiality was listed by 18% of respondents as an ethically troubling issue.
d. Incorrect The conduct of colleagues was listed as a problem by 4% of respondents.

The correct answer is: confidentiality

113
Q

Although the nature-nurture controversy has not been resolved, many theorists now view development as involving both factors. More specifically, theorists who take a middle ground with regard to this issue:
Select one:

A.
view some traits as being due primarily to nature and others as being due primarily to nurture.

B.
believe that genetic potentials (nature) cannot be realized unless the appropriate environmental experiences (nurture) occur.

C.
believe that most traits reflect about equal contributions of nature and nurture.

D.
view individual differences as due to genetic factors (nature) and group differences as due to environmental factors (nurture).

A

Recent theories of development typically view nature and nature as contingent factors; i.e., development cannot occur without the ongoing contributions of each factor.
a. Incorrect Although this is true, this answer is not the best one since it does not address the contingent relationship between nature and nurture.

b. CORRECT This is the only response that suggests a contingent relationship between the factors and, therefore, is the best answer.
c. Incorrect This is not a good description of current thinking about the nature-nurture controversy.
d. Incorrect This is not an accurate description of the current status of the nature-nurture controversy.

The correct answer is: believe that genetic potentials (nature) cannot be realized unless the appropriate environmental experiences (nurture) occur.

114
Q

Amitriptyline is often used as a treatment for neuropathic pain, and it exerts its effects by:
Select one:

A.
increasing norepinephrine and serotonin levels.

B.
increasing norepinephrine levels and decreasing serotonin levels.

C.
decreasing serotonin and dopamine levels.

D.
increasing serotonin levels and decreasing dopamine levels.

A

Knowing that amitriptyline is a tricyclic antidepressant may have helped you identify the correct answer to this question.
a. CORRECT Amitriptyline increases the levels of both norepinephrine and serotonin by inhibiting their reuptake.

b. Incorrect See explanation for response a.
c. Incorrect See explanation for response a.
d. Incorrect See explanation for response a.

The correct answer is: increasing norepinephrine and serotonin levels.

115
Q

Which of the following is most useful for explaining why so many people believe that their horoscopes accurately describe them?
Select one:

A.
Barnum effect

B.
false consensus bias

C.
Zeigarnik effect

D.
illusory correlation

A

Researchers have found that individuals are often willing to agree that vague descriptions of them - such as those presented in horoscopes - accurately describe their personal characteristics.
a. CORRECT The Barnum effect (also known as the Forer effect) predicts that people tend to accept vague or general descriptions of themselves as accurate. The Barnum effect has been attributed to a number of factors including gullibility, wishful thinking, and a confirmation bias.

b. Incorrect The false consensus bias is the tendency to overestimate the degree to which others agree with you.
c. Incorrect The Zeigarnik effect is the tendency to remember interrupted or uncompleted tasks better than completed ones.
d. Incorrect Illusory correlation is the belief that there is a relationship between events when there actually isn’t one.

The correct answer is: Barnum effect

116
Q

When using Atkinson, Morten, and Sue’s (1993) Racial/Cultural Identity Development Model, a person’s stage of identity development is determined by:
Select one:

A.
identifying his/her degree of acceptance of the minority and majority cultures.

B.
assessing the amount of contact he/she normally has with members of the minority and majority cultures.

C.
assessing his/her attitudes toward the minority and majority cultures.

D.
evaluating his/her reaction to racial oppression.

A

Atkinson et al.’s model distinguishes between five stages: conformity, dissonance, resistance/immersion, introspection, and integrative awareness. Additional information about these stages is provided in the Clinical Psychology chapter of the written study materials.
a. Incorrect See explanation for response c.

b. Incorrect See explanation for response c.
c. CORRECT Each stage in this model is characterized by a different combination of attitudes toward one’s own minority group, other minority groups, and the majority (dominant) group.
d. Incorrect See explanation for response c.

The correct answer is: assessing his/her attitudes toward the minority and majority cultures.

117
Q

Polly Pooch teaches her son to feed the dog by first showing him how to put the dog food into the dog dish. Once the boy has mastered that task, she teaches him to open a can of dog food and then put it into the dish. Finally, she teaches her son to open the cupboard, take out a can of dog food, open it, and put the dog food in the dog dish. The procedure that Mrs. Pooch has used is best described as:
Select one:

A.
backward chaining.

B.
forward chaining.

C.
stimulus control training.

D.
sequential training.

A

Polly Pooch’s son has been taught a complex behavior by teaching him the individual responses in the “behavior chain.”
a. CORRECT Since Mrs. Pooch has started with the last behavior and “worked backward” from there, this is best described as an example of backward chaining.

b. Incorrect See explanation above.
c. Incorrect See explanation above.
d. Incorrect See explanation above.

The correct answer is: backward chaining.

118
Q

A transformational leader defines a goal to employees in a way that imbues the goal with meaning and purpose. This is referred to as:

Select one:

A.
priming.

B.
escalating commitment.

C.
framing.

D.
positioning.

A

Transformational leaders differ from transactional leaders in several ways. One way is that they motivate employees by giving goals added meaning by placing them in a larger context (e.g., the “good of society”).<p>a. Incorrect The term priming is used in several contexts. For example, in the field of learning and memory, it refers to unconscious cues that activate related associations (e.g., seeing the word “test” reminds you of the licensing exam even if you are not aware that you saw the word “test”).</p><p>b. Incorrect Escalating commitment refers to the tendency to continue with a previously chosen course of action even when it is not working.</p><p>c. CORRECT Giving goals added meaning is referred to as framing. It is one of the characteristics of transformational leaders.</p><p>d. Incorrect Positioning is a paradoxical technique used in family therapy. It involves helping the family see a symptom from a new perspective by exaggerating its severity.

The correct answer is: framing.</p>

119
Q

During the first session with an American Indian family consisting of a grandmother, mother, father, and three children, an Anglo-American therapist would be best advised to:
Select one:

A.
refer the family to an American Indian therapist.

B.
ensure that she establishes a “value-free” environment.

C.
assess each family member’s level of acculturation before developing a treatment plan.

D.
treat the family like any other family unless there is a clear reason to do otherwise.

A

You may have been able to identify the correct response to this question through the process of elimination.
a. Incorrect This may be unnecessary if the therapist has appropriate training or seeks consultation.

b. Incorrect American Indian clients are likely to distrust a therapist who attempts to establish a “value-free” environment.
c. CORRECT An initial step when working with clients from culturally diverse groups is to identify the client’s stage of racial/ethnic identity development, degree of acculturation, and worldview (beliefs, values, attitudes, etc.).
d. Incorrect APA guidelines discourage adopting a “colorblind” approach and, instead, require psychologists to always consider the impact of culture on the therapeutic process.

The correct answer is: assess each family member’s level of acculturation before developing a treatment plan.

120
Q

When using the DSM-5, the diagnosis of Delusional Disorder requires the presence of:
Select one:

A.
one or more delusions for at least one month.

B.
one or more non-bizarre delusions for at least one month.

C.
one or more bizarre delusions for at least one month.

D.
one or more bizarre or non-bizarre delusions for at least three months.

A

Answer A is correct: The DSM-5 diagnosis of Delusional Disorder requires the presence of one or more delusions that last at least one month.

The correct answer is: one or more delusions for at least one month.

121
Q

The purpose of “rotation” in factor analysis is to:
Select one:

A.
obtain a pattern of factor loadings that is easier to interpret.

B.
reduce the impact of measurement error on the factor loadings.

C.
reduce the magnitude of the communalities.

D.
obtain a clearer pattern of communalities.

A

For the exam, you don’t need to understand what rotation involves, but you do want to be familiar with its purpose.
a. CORRECT The pattern of factor loadings in the initial factor matrix is often difficult to interpret, so the factors are rotated to obtain a pattern that’s easier to interpret.

b. Incorrect Rotation does not reduce the effects of measurement error.
c. Incorrect Rotation alters the magnitude of the factor loadings but not the magnitude of the communalities.
d. Incorrect See explanation for responses a and c.

The correct answer is: obtain a pattern of factor loadings that is easier to interpret.

122
Q

Escape and avoidance conditioning both involve:
Select one:

A.
positive reinforcement.

B.
negative reinforcement.

C.
positive punishment.

D.
negative punishment.

A

Escape and avoidance conditioning both establish a response that allows the individual to elude an aversive stimulus.
a. Incorrect See explanation for response b.

b. CORRECT In escape and avoidance conditioning, the individual’s behavior increases because it enables him/her to escape or avoid a stimulus (i.e., a stimulus is taken away following the response). Escape conditioning is simply a straightforward application of negative reinforcement, while avoidance conditioning is more complex and involves a combination of negative reinforcement and classical conditioning. See the Learning Theory chapter for additional information about escape and avoidance conditioning.
c. Incorrect See explanation for response b.
d. Incorrect See explanation for response b.

The correct answer is: negative reinforcement.

123
Q

For most children, expressions of pride, shame, and guilt are first evident by ________ months of age.
Select one:

A.
12

B.
20

C.
30

D.
42

A

Self-conscious emotions become apparent during the second and third years of life. Research on the ages at which the self-conscious emotions are first evident has provided somewhat inconsistent results. However, there is evidence that empathy, jealousy, and embarrassment emerge at about 18 to 24 months of age and are followed by pride, shame, and guilt at about 30 months. See, e.g., M. Lewis, Early emotional development, in A. Slatter and M. Lewis (Eds.), Introduction to infant development, New York, Oxford University Press, 2002.

The correct answer is: 30

124
Q

Applicants for a job at the Acme Corporation are very upset by some of the items included in the company’s Biographical Information Blank. They feel that the items are not work-related and therefore violate their privacy. Apparently, these items:

Select one:

A.
lack content validity.

B.
lack face validity.

C.
are “unfair”

D.
are unreliable

A

In the situation described in this question, some of the items on the Biographical Information Blank appear to be unrelated to the requirements of the job. Face validity refers to the extent to which a measure appears to measure what it is supposed to measure.

The correct answer is: lack face validity.

125
Q

Which of the following work shifts is likely to be associated with the least amount of job-related stress?
Select one:

A.
permanent day shift

B.
permanent night shift

C.
rotating shifts

D.
compressed workweek with rotating shifts

A

Unfortunately, the research on work schedules is very inconsistent with different studies comparing different types of schedules and producing different results with regard to the outcomes associated with flextime, compressed workweek, rotating shifts, etc.
a. CORRECT In one study, workers who were switched from rotating shifts to permanent day shifts showed significant increases in several measures of well-being [T. Akerstedt and L. Torsvall, Experimental changes in shift schedules: The effect on well-being, Ergonomics, 21(10), 849-856, 1978]. In a more recent study, workers on fixed and rotating shifts were compared and those on the fixed shift reported less stress and better work attitudes [M. Jamal and V. Baba, Shiftwork and department-type related to job stress, work attitudes, and behavioral intentions: A study of nurses, Journal of Organizational Behavior, 13(5), 449-464, 1992].

b. Incorrect Studies comparing day and night shifts have usually found the day shift to be associated with better outcomes. Apparently, this is because, for most people, the night shift is inconsistent with the normal sleep-wake cycle.
c. Incorrect See explanation above.
d. Incorrect See explanation above.

The correct answer is: permanent day shift

126
Q

A cigarette smoker says that she has decided to quit smoking in the next three to four weeks and has started developing a plan for doing so. According to the transtheoretical (stages-of-change) model, this person is in which of the following stages?
Select one:

A.
preparation

B.
contemplation

C.
precontemplation

D.
action

A

The transtheoretical model predicts that an intervention is most effective when it matches the individual’s stage of change.
a. CORRECT From the perspective of the transtheoretical model, a person is in the preparation stage when he/she has a clear intent to take action in the immediate future (usually defined as within the next month).

b. Incorrect A person in the contemplation stage is planning to take steps to change in the more distant future (usually defined as within the next six months).
c. Incorrect A person in the precontemplation stage has no intention of changing in the foreseeable future.
d. Incorrect People in the action phase have already started making changes in their lives.

The correct answer is: preparation

127
Q

Studies investigating stepparent-stepchild relationships indicate that:
Select one:

A.
stepparent-stepchild relationships are less likely to be conflictual when the child is in early adolescence when the remarriage occurs.

B.
stepfathers tend to be distant and disengaged from their stepchildren.

C.
stepmothers tend to be more authoritative in parenting style than stepfathers.

D.
stepfather-stepchild relationships tend to be more difficult for boys than for girls.

A

Relatively little empirical research has been conducted on the stepparent-stepchild relationship, so this question is hard to answer. Unfortunately, the exam does occasionally include questions on topics that have received little research attention.
a. Incorrect In most cases, this is the opposite of what is true – relationships with stepparents are most difficult when remarriage occurs when the child is in early adolescence.

b. CORRECT This finding is often cited in the literature and is particularly true for stepfathers who do not have biological children. See, e.g., J. W. Santrock, K. A. Sitterle, and R. A. Warshak, Parent-child relationships in stepfather families, in P. Bronstein and C. P. Cowan (Eds.), Fatherhood today, New York, Wiley, 1988.
c. Incorrect In fact, there is some evidence that stepparents tend to be more authoritarian (not authoritative) than biological parents are.
d. Incorrect This is the opposite of what has been found to be true.

The correct answer is: stepfathers tend to be distant and disengaged from their stepchildren.

128
Q

To compare the effectiveness of three different doses of a new drug for generalized anxiety disorder (GAD), an investigator obtains a sample of 16 adults who have just received a diagnosis of GAD. All participants in the study will receive all three doses of the drug but in different order: Four participants will first receive the low dose for two weeks, then the moderate dose for two weeks, and then the high dose for two weeks; four other participants will first receive the moderate dose for two weeks, then the low dose for two weeks, then the high dose for two weeks; and so on. The investigator is using which of the following research designs?
Select one:

A.
analogue

B.
dismantling

C.
counterbalanced

D.
Solomon four-group

A

The investigator is using a within-subjects design that involves having each participant receive all levels of the independent variable.
a. Incorrect The term “analogue” is used in the context of research design to refer to the investigation of a well-defined research question in well-controlled conditions.

b. Incorrect The dismantling strategy is used to determine the effects of each component of a multi-component treatment and involves administering different components to different groups of participants.
c. CORRECT Counterbalanced designs are within-subjects designs that help control order or practice effects by administering the levels of the independent variable to different subjects (or groups of subjects) in a different order.
d. Incorrect The Solomon four-group design is used to evaluate the impact of pretesting on a study’s internal and external validity.

The correct answer is: counterbalanced

129
Q

Research on sexual and nonsexual dual relationships between therapists and their clients has found that:
Select one:

A.
male therapists engage in more sexual dual relationships, but female therapists engage in more nonsexual dual relationships.

B.
the rates of sexual dual relationships are higher among those with less professional experience.

C.
in a sexual dual relationship, the client is, on the average, 10 years younger than the therapist.

D.
therapists who undergo personal therapy are less likely to become sexually involved with their clients.

A

As noted in the chapter on ethics and professional issues in the written study materials, several surveys have been conducted to determine factors related to sexual misconduct by mental health professionals.
a. Incorrect Male therapists are involved in substantially more sexual and nonsexual dual relationships than female therapists.

b. Incorrect This is not necessarily true. Some research indicates higher rates among the more experienced.
c. CORRECT The average age of therapists who become involved in sexual dual relationships is 42 or 43 (depending on which study you read), while the average age of the client is 32 or 33. See K. S. Pope, Sexual feelings in psychotherapy: Explorations for therapists and therapists-in-training, American Psychological Association, Washington, DC, 1993.
d. Incorrect The research is not entirely consistent on this issue. Most studies have found either that there is no effect of therapy or that therapy is associated with an increased risk for sexual involvement.

The correct answer is: in a sexual dual relationship, the client is, on the average, 10 years younger than the therapist.

130
Q

The primary advantage of manual-guided treatment is that the use of manuals:
Select one:

A.
standardizes treatment delivery so that, in research, treatment effects can be more clearly ascertained.

B.
substantially improves treatment outcomes across different clients, diagnoses, and contexts.

C.
reduces the legal liability of clinicians for “treatment errors.”

D.
ensures that the procedures used to evaluate treatment effectiveness are valid.

A

Manual-guided treatments provide manuals that specify the theoretical underpinnings of the treatment and treatment goals and present specific therapeutic guidelines and strategies.
a. CORRECT Treatment manuals were initially developed to standardize treatments so that their effects could be empirically evaluated and to provide training guidelines for therapists.

b. Incorrect The research on this issue is inconsistent and but has generally not found manual guided treatments to have substantially better outcomes than treatments that are not guided by the use of a manual.
c. Incorrect This has not been identified as an outcome of the use of treatment manuals.
d. Incorrect This is not one of the benefits of treatment manuals.

The correct answer is: standardizes treatment delivery so that, in research, treatment effects can be more clearly ascertained.

131
Q

Statistics on the risk for suicide among adolescents from single-parent and dual-parent families show that:
Select one:

A.
there is no relationship between family configuration and risk for suicide.

B.
girls (but not boys) from single-parent families are at higher risk than girls from dual-parent families.

C.
boys (but not girls) from single-parent families are at higher risk than boys from dual-parent families

D.
girls and boys from single-parent families are both at higher risk than girls and boys from dual-parent families.

A

One of the most consistent findings is that, for adolescents, living in a single-parent family increases the risk for suicide.

a. Incorrect See explanation for response d.
b. Incorrect See explanation for response d.
c. Incorrect See explanation for response d.
d. CORRECT The linkage between a single-parent family and risk for a suicide in adolescents was recently confirmed in a large-scale study in Sweden, which compared 65,085 children from single-parent homes with 921,257 children from two-parent homes and found that the rate of suicide attempts was 2 times greater for girls from single-parent homes and 2.3 times greater for boys from single-parent homes [G. R. Weitoft et al., Mortality, severe morbidity, and injury in children living with single parents in Sweden: A population-based study, The Lancet, 361(9354), 289-295, 2003].

The correct answer is: girls and boys from single-parent families are both at higher risk than girls and boys from dual-parent families.

132
Q

The APA’s Ethics Code requires that informed consents be:
Select one:

A.
in writing.

B.
in writing whenever the individual is not a minor.

C.
documented.

D.
signed by the individual or his/her legal guardian.

A

Although written, signed consents are advisable in both therapy and research, they are not required by the Ethics Code.
a. Incorrect See explanation for response c.

b. Incorrect See explanation for response c.
c. CORRECT Paragraph 3.10(d) of the Code states that “Psychologists appropriately document written or oral consent, permission, and assent.”
d. Incorrect See explanation for response c.

The correct answer is: documented.

133
Q

“I have two feets” and “I runned fast” are examples of the grammatical error known as:
Select one:

A.
telegraphic speech

B.
holophrastic speech

C.
overextension

D.
overregularization

A

Young children make a number of predictable errors when acquiring language. Overregularization occurs when children overextend grammatical rules - e.g., add “s” to feet to form the plural or “ed” to run to form the past tense.

The correct answer is: overregularization

134
Q

With regard to terminating the life of animals used in research studies, the APA’s Ethics Code:
Select one:

A.
permits this practice only when an animal is suffering.

B.
permits this practice only in the “most unusual circumstances.”

C.
permits this practice when it is appropriate and done in a way that minimizes pain.

D.
prohibits this practice.

A

This situation is addressed in Standard 8.09(g) of the Ethics Code.
a. Incorrect See explanation for response c.

b. Incorrect See explanation for response c.
c. CORRECT Standard 8.09(g) states that “when it is appropriate that an animal’s life be terminated, psychologists proceed rapidly, with an effort to minimize pain and in accordance with accepted procedures.”
d. Incorrect See explanation for response c.

The correct answer is: permits this practice when it is appropriate and done in a way that minimizes pain.

135
Q

A father puts his son in his room every time the boy hits his sister. To increase the probability that the boy will stop hitting his sister, the father should:
Select one:

A.
give the child 50 cents each time he is nice to his sister.

B.
initially send the boy to his room every time he hits his sister and then send him to his room every three times he hits his sister.

C.
gradually increase the length of time the boy spends in his room after each time he hits his sister.

D.
sometimes send the boy to his room for 10 minutes and other times send him to his room for 20 minutes or 30 minutes after he hits his sister.

A

The parent is using punishment in order to decrease the probability that the behavior will occur again. A variety of factors affect the effectiveness of punishment.
a. CORRECT Punishment, like all forms of behavior therapy used to reduce undesirable behaviors, is most effective when used in conjunction with reinforcement for alternate desirable behaviors. In this case, the 50 cents would represent reinforcement (a stimulus that increases the probability of a behavior) for the alternate desirable behavior of being nice.

b. Incorrect Punishment is more effective when it is applied consistently (after every performance of a behavior) than when it is applied intermittently.
c. Incorrect A gradual increase in the intensity of punishment can produce habituation. In other words, if the child has to spend more and more time in his room, he may get used to it and won’t mind it as much.
d. Incorrect This strategy will do nothing to decrease the occurrence of the behavior and may produce habituation.

The correct answer is: give the child 50 cents each time he is nice to his sister.

136
Q

In organizations, __________ involves training workers to perform different tasks and activities.
Select one:

A.
vestibule training

B.
cross-training

C.
over-training

D.
frame-of-reference training

A

For the exam, you want to be familiar with the off-the-job and on-the-job training techniques that are described in the Industrial-Organizational Psychology chapter of the written study materials so that you can answer questions like this one.
a. Incorrect See explanation for response b.

b. CORRECT As its name suggests, cross-training involves training an employee to perform other tasks (usually tasks related to his/her current job). Cross-training benefits the employer by increasing the flexibility of employees and the employees by increasing their interest and motivation.
c. Incorrect See explanation for response b.
d. Incorrect See explanation for response b.

The correct answer is: cross-training

137
Q

As a treatment for alcohol dependence, naltrexone:
Select one:

A.
causes the person to feel sick when taken in combination with alcohol.

B.
reduces the person’s craving for alcohol.

C.
eliminates symptoms of alcohol withdrawal.

D.
is used to produce rapid detoxification.

A

Naltrexone (ReVia) is an opioid receptor antagonist. a. Incorrect Antabuse reduces alcohol consumption by causing adverse effects when taken in combination with alcohol. b. CORRECT Naltrexone blocks the reinforcing effects and craving (desire) for alcohol. c. Incorrect Naltrexone does not reduce withdrawal symptoms. d. Incorrect Naltrexone is not used to produce rapid detoxification.

The correct answer is: reduces the person’s craving for alcohol.

138
Q

A Bowenian therapist would use a genogram to:
Select one:

A.
examine family relationships over several generations.

B.
identify genetic (biological) contributions to maladaptive behavior.

C.
examine interactions between a family and its environment.

D.
evaluate the progress and outcome of therapy.

A

Being familiar with the major concerns and assumptions of Bowen’s extended family systems therapy would have helped you identify the correct answer to this question even if you’re unfamiliar with a genogram.
a. CORRECT As originally described by Bowen, a genogram is a schematic diagram of a family’s relationships and usually includes at least three generations. A primary purpose of a genogram is to identify behavior patterns that recur in the family.

b. Incorrect See explanation for response a.
c. Incorrect This describes an ecomap. Although some authors use the terms ecomap and genogram interchangeably, this question is asking specifically about the use of a genogram by a Bowenian therapist, so response a is the best answer.
d. Incorrect See explanation for response a.

The correct answer is: examine family relationships over several generations.

139
Q

Research investigating the benefits of psychological treatments for children, adolescents, and adults with cystic fibrosis is very limited. However, the existing studies suggest that which of the following approaches is likely to be most beneficial for treating depression, anxiety, pain, and stress associated with this illness?
Select one:

A.
cognitive-behavioral therapy

B.
client-centered therapy

C.
solution-focused therapy

D.
reality therapy

A

A review of the existing studies clearly indicates that one type of therapy has been the focus of the research on psychological interventions for individuals with cystic fibrosis and several other medial conditions.

a. CORRECT Very few studies have been conducted on psychological interventions for patients with cystic fibrosis, and the majority of these have focused on children. However, most of the existing studies investigated the impact of behavioral therapy or cognitive-behavioral therapy and have found that these interventions are beneficial for alleviating psychological problems associated with the disorder. Therefore, of the answers given, this is the best one. See, e.g., H. Oxley and A. K. Webb, How a clinical psychologist manages the problems of adults with cystic fibrosis, Journal of the Royal Society of Medicine, 98 (Suppl. 45), 37-46, 2005.
b. Incorrect See explanation above.
c. Incorrect See explanation above.
d. Incorrect See explanation above.

The correct answer is: cognitive-behavioral therapy

140
Q

Neo-Piagetian theories of cognitive development differ from Piaget’s theory in terms of their:
Select one:

A.
emphasis on information processing.

B.
emphasis on the role of social influences.

C.
rejection of a stage model of development.

D.
rejection of Piaget’s notion of domain specificity.

A

The neo-Piagetian theories include the theories of Kurt Fischer and Robbie Case.
a. CORRECT The neo-Piagetian theories use information processing concepts to explain proposed shortcomings of Piagetian theory.

b. Incorrect An emphasis on social influences is not a factor that distinguishes Piagetian and neo-Piagetian theories.
c. Incorrect The neo-Piagetians view cognitive development as involving stages.
d. Incorrect This is the opposite of what is true. Neo-Piagetians are interested in cognitive development within specific domains.

The correct answer is: emphasis on information processing.

141
Q

The three main types of genetic testing (cytogenetic, biochemical, molecular) examine what specific areas for any abnormalities?
Select one:

A.
Chromosome structure; protein function; mitochondrial nerves.

B.
DNA sequence; protein structure; cortex neurons.

C.
Chromosome structure; protein function; DNA sequence.

D.
Binomial function; DNA sequence; protein structure.

A

The correct answer is C. Cytogenetics involves the examination of chromosomes and their abnormalities. Biochemical testing utilizes techniques that examine the protein instead of the gene. Molecular or DNA testing tends to be the most efficacious methodology, particularly if the function of the protein is not known and a biochemical test cannot be developed. Answers A, B and D are incorrect as mitochondrial nerves, protein structure, cortex neurons and binomial functions are not part of the genetic material that is tested.

The correct answer is: Chromosome structure; protein function; DNA sequence.

142
Q

High scores on Scales 1 and 3 of the MMPI with a low score on Scale 2 are most suggestive of which of the following?
Select one:

A.
a somatoform disorder

B.
a psychotic disorder

C.
alcohol abuse

D.
schizoid reactions

A

On the MMPI, a combination of high scores on the hypochondriasis and hysteria scales (Scales 1 and 3) and a low score on the depression scale (Scale 2) is referred to as the conversion-V pattern.
a. CORRECT The conversion-V pattern is characteristic of people with Conversion Disorder or other somatoform disorder. (Note that, in the DSM-5, the Somatoform Disorder category has been replaced by the Somatic Symptom and Related Disorders category.)

b. Incorrect Higher scores on Scales 6 and 8 than on Scale 7 is referred to as the psychotic-V pattern and is associated with psychotic symptoms.
c. Incorrect Alcohol and drug abuse and depression are associated with high scores on Scales 2, 4, and 7.
d. Incorrect Elevated scores on Scales 2, 7, and 8 are associated with schizoid reactions and phobic and depressive symptoms.

The correct answer is: a somatoform disorder

143
Q

__________ are mental “rules-of-thumb” that help us make decisions or solve problems more quickly but that do not necessarily lead to the optimal decision or solution.
Select one:

A.
Prototypes

B.
Mnemonics

C.
Algorithms

D.
Heuristics

A

For the exam, you want to know the definition of each of the terms listed in the answers to this question.
a. Incorrect Prototypes are representative examples of a concept or category. For example, some people would consider Freud to be a prototypical psychotherapist.

b. Incorrect Mnemonics are strategies used to improve memory.
c. Incorrect Algorithms are step-by-step problem-solving procedures or strategies that allow individuals to solve problems even if they do not completely understand the problem.
d. CORRECT Heuristics are mental shortcuts (rules-of-thumb) that simplify decision-making or problem-solving. For example, you would be relying on a heuristic if you assume that a higher price is always associated with a better quality product and then use price to make decisions about what products to buy.

The correct answer is: Heuristics

144
Q

What two most common quasi-experimental methods are used to screen for genetic influence on individual differences such as behavior?
Select one:

A.
The Twin Method and Family Method.

B.
The Benson Method and Adoption Method.

C.
The Separate Method and Individual Method.

D.
The Twin Method and Adoption Method.

A

The correct answer is D. The twin method relies on the providence of nature that results in identical (monozygotic, MZ) twins or fraternal (dizygotic, DZ) twins. The adoption method is a quasi-experimental design that relies on a social chance in which children are adopted away from their biological (birth) parents early in life, thus separating effects of genetics versus environmental factors. Answers A, B, and C are incorrect as the Family Method, Benson Method, Separate Method, and Individual Method are irrelevant or non-existent regarding this type of research.

The correct answer is: The Twin Method and Adoption Method.

145
Q

Which of the following are characteristic symptoms of Parkinson’s disease?
Select one:

A.
resting tremor; slowed movement; rigidity; and impaired balance and coordination

B.
jerky, involuntary movements; facial grimaces; restlessness; and general weakness

C.
muscle weakness; impaired balance and coordination; paresthesias; and nystagmus

D.
twitching; cramping or stiffness of the muscles; muscle weakness; and slurred speech

A

The symptoms of Parkinson’s disease vary somewhat from person to person and from day to day. However, most people with this disorder experience the symptoms listed in answer a.
a. CORRECT Resting tremor, slowed movement (bradykinesia), rigidity, and impaired balance and coordination are common symptoms of Parkinson’s disease.

b. Incorrect These are symptoms of Huntington’s disease.
c. Incorrect These are characteristic symptoms of multiple sclerosis.
d. Incorrect These are characteristic symptoms of amyotrophic lateral sclerosis (ALS), which is also known as Lou Gehrig’s disease.

The correct answer is: resting tremor; slowed movement; rigidity; and impaired balance and coordination

146
Q

A DSM-5 diagnosis of Intermittent Explosive Disorder requires the presence of:
Select one:

A.
verbal aggression or physical aggression toward property, animals, or other people that does not cause injury or destruction or behavioral outbursts that do cause injury or destruction.

B.
physical aggression that is injurious to animals or people or destructive to property.

C.
physically aggressive acts that are precipitated by an irritable mood and that cause injury to people or animals or destruction to property.

D.
aggressive acts that cause injury to people or animals or destruction to property and are accompanied by a lack of remorse or guilt.

A

Answer A is correct: The DSM-5 diagnosis of Intermittent Explosive Disorder requires recurrent behavioral outbursts that reflect a failure to control aggressive impulses as manifested by verbal aggression or physical aggression toward property, animals, or other people that does not result in destruction or injury or behavioral outbursts that do result in destruction of property or injury of people or animals.

Answer B: This is not a requirement for a DSM-5 diagnosis of Intermittent Explosive Disorder.

Answer C: The aggressive acts associated with Intermittent Explosive Disorder are due to a failure to control aggressive impulses but are not necessarily precipitated by an irritable mood. Also, as noted above, aggression can be verbal or physical.

Answer D: The diagnosis of Intermittent Explosive Disorder does not require that aggression be accompanied by a lack of remorse or guilt.

The correct answer is: verbal aggression or physical aggression toward property, animals, or other people that does not cause injury or destruction or behavioral outbursts that do cause injury or destruction.

147
Q

Dr. E. Dict has been seeing Jay Bird, a prisoner at the local jail, in therapy for the past five months. The parole board contacts Dr. Dict to request that he evaluate Bird for an upcoming parole hearing. Dr. Dict should:
Select one:

A.
agree to evaluate Bird since he is best qualified to evaluate him fairly.

B.
agree to evaluate Bird only if Bird agrees to the arrangement.

C.
agree to evaluate Bird only if he feels that Bird is eligible for parole.

D.
refuse to evaluate Bird for the parole hearing.

A

The APA’s guidelines require psychologists to avoid dual (multiple) relationships.
a. Incorrect See explanation for response d.

b. Incorrect See explanation for response d.
c. Incorrect See explanation for response d.
d. CORRECT This type of situation is addressed in the Specialty Guidelines for Forensic Psychology and the Ethics Code, which require psychologists to avoid dual relationships whenever objectivity, exploitation, and other issues related to effectiveness are of concern, which would be the case in this situation. The two roles – therapist and forensic evaluator – clearly conflict and should be avoided.

The correct answer is: refuse to evaluate Bird for the parole hearing.

148
Q

____________ theory of socially mediated learning is based on the premise that learning is first socially mediated, then self-mediated.
Select one:

A.
Vygotsky’s

B.
Piaget’s

C.
Dewey’s

D.
Gagne’s

A

For the exam you want to be familiar with the major approaches to learning and cognitive development especially the approaches of Piaget and Vygotsky.

a. CORRECT If you did not know that Vygotsky’s sociocultural theory is also referred to as the theory of socially mediated learning you may have been able to identify this as the correct response by recalling that Vygotsky described learning as first social then individual.
b. Incorrect Piaget’s developmental theory (which is also known as the theory of genetic epistemology) proposes that individuals build cognitive structures in active and adaptive ways (e.g. through assimilation and accommodation).
c. Incorrect Dewey’s theory emphasizes the experiential aspects of learning and it proposes that learning is the result of our experiences and attempts to make sense of those experiences.
d. Incorrect Gagne’s approach to learning focuses on instruction and is sometimes categorized as an information processing approach. It focuses on learning outcomes (cognitive effective or psychomotor) and the conditions that facilitate each type of outcome.

The correct answer is: Vygotsky’s

149
Q

Research evaluating the use of multi-component cognitive-behavioral therapy for rheumatoid arthritis has found that it:
Select one:

A.
improves social functioning but has little or no effect on pain intensity or joint inflammation.

B.
improves comorbid anxiety and depression but has no effect on pain intensity or joint inflammation.

C.
improves psychological functioning and, in some cases, has a beneficial effect on pain intensity and joint inflammation.

D.
reduces pain intensity and joint inflammation but has little or no effect on overall psychological functioning.

A

This is a difficult question since the research on the effects of cognitive-behavior therapy (CBT) on the physical and psychological functioning of patients with rheumatoid arthritis has not produced entirely consistent results.
a. Incorrect See explanation for response c.

b. Incorrect See explanation for response c.
c. CORRECT Of the answers given, this is the best one since the qualifier “in some cases” reflects the inconsistency of the research results. While the studies have generally found that CBT has beneficial effects on feelings of self-efficacy, anxiety, and other aspects of psychological functioning, its effects on the physical symptoms of rheumatoid arthritis vary from study to study. In some studies, however, CBT did reduce pain intensity and joint impairment and inflammation. See, e.g., A. O’Leary et al., A cognitive-behavioral treatment for rheumatoid arthritis, Health Psychology, 7, 527-544, 1988
d. Incorrect See explanation for response c.

The correct answer is: improves psychological functioning and, in some cases, has a beneficial effect on pain intensity and joint inflammation.

150
Q

From the perspective of two-factor theory, the avoidance response in a phobic reaction is maintained by:
Select one:

A.
negative reinforcement.

B.
positive reinforcement.

C.
intermittent reinforcement.

D.
vicarious reinforcement.

A

Two-factor theory describes phobic reactions as involving both classical conditioning and negative reinforcement.
a. CORRECT Negative reinforcement is occurring when a response increases because an aversive stimulus is removed following the response. In phobic reactions, the avoidance response increases because, by avoiding the feared object or event, anxiety is reduced or eliminated.

b. Incorrect See explanation for response a.
c. Incorrect See explanation for response a.
d. Incorrect See explanation for response a.

The correct answer is: negative reinforcement.

151
Q

Wolfgang Kohler’s research on animal learning and animal cognition led to his conclusion that learning is:
Select one:

A.
biologically based.

B.
insightful.

C.
the result of trial-and-error.

D.
the result of reinforcement and punishment.

A

Kohler is a co-founder of Gestalt psychology, which focuses on perception. He is probably best known for his research on animals, especially Sultan the chimpanzee.
a. Incorrect See explanation for response b.

b. CORRECT Kohler’s research revealed that like humans, animals seem to experience an “aha” experience (“insight”) while solving problems.
c. Incorrect See explanation for response b.
d. Incorrect See explanation for response b.

The correct answer is: insightful.

152
Q

Ataxia is a common symptom of advanced multiple sclerosis and is characterized by:
Select one:

A.
a complete or almost complete loss of movement.

B.
an uncomfortable sense of restlessness or agitation.

C.
impaired coordination and balance.

D.
slow writhing involuntary movements of the extremities.

A

Ataxia is a symptom of a number of disorders including multiple sclerosis, cerebral palsy, spinal injury, and stroke. Note that, for the licensing exam, you want to be familiar with all of the conditions described in the answers to this question. Additional information on these conditions is provided in the Physiological Psychology and Psychopharmacology chapter of the written study materials.
a. Incorrect A complete or almost complete loss of movement is referred to as akinesia.

b. Incorrect An uncomfortable sense of restlessness or agitation is referred to as akathisia.
c. CORRECT Ataxia involves a loss of balance and coordination. It is one of the symptoms of multiple sclerosis and can be caused by damage to the cerebellum, dorsal spinal cord, or vestibular system. Although its symptoms depend on the area affected, they often include unsteady gait, difficulty with fine motor tasks, abnormal eye movements, slurred speech, and difficulty swallowing.
d. Incorrect Athetosis involves slow writhing involuntary movements.

The correct answer is: impaired coordination and balance.

153
Q

Jenny and Craig are engaging in intercontinental dating via video chatting and other electronic means since they initially met through a dating website. She lives in Hong Kong, and he lives in Belfast. What does research indicate could be a potential problem as Jenny and Craig try to grow their relationship?
Select one:

A.
The long distance.

B.
Lack of quality and depth in their relationship.

C.
Their cultural differences.

D.
None of the above.

A

The correct answer is B. The use of the internet for interpersonal communication is not the same as traditional face-to-face communication in enriching the quality of life. Online communication does have the potential to have an adverse effect on peoples perceived life quality. Although the relative lack of strong ties or in-depth quality that can result in internet communication cannot be the sole reason for the negative effect of online communication for quality of life. An absence of nonverbal cues: lack of warmth, and less demand for engagement in internet communication results in impersonality, shallow interactions, and difficulty in building social support. These are reasons why there is a negative contribution of online communication to perceived quality of life.

Answers A and C are incorrect and although they may be part of ongoing long distant relationship problems that occur between individuals of various cultures, it is not necessarily tied directly to the use of technology to foster the growth of their relationship.

The correct answer is: Lack of quality and depth in their relationship.

154
Q

The point at which an item characteristic curve intercepts the vertical (Y) axis indicates which of the following?
Select one:

A.
the item’s difficulty level

B.
the probability of answering the item correctly by guessing

C.
the degree to which the item is affected by measurement error

D.
the item’s ability to distinguish between high and low achievers.

A

When using item response theory to construct a test, an item characteristic curve is constructed for each item.
a. Incorrect See explanation for response b.

b. CORRECT The point at which the curve intercepts the Y axis indicates the probability of answering the item correctly by guessing. Other information provided by the curve is described in the Test Construction chapter of the written study materials.
c. Incorrect See explanation for response b.
d. Incorrect See explanation for response b.

The correct answer is: the probability of answering the item correctly by guessing

155
Q

Research suggests that the underutilization of mental health services by Asian Americans results from:
Select one:

A.
cultural values that encourage resolution of problems within the family.

B.
a lower incidence of mental health problems.

C.
cultural values that stress personal independence.

D.
a distaste for the formality involved in therapy relationships.

A

The research has generally found that Asian Americans underutilize traditional mental health services.
a. CORRECT Asian Americans often adhere to traditional cultural values that encourage the resolution of problems within the family. Reasons for preferring assistance from family members include the stigma associated with mental illness and seeking outside help and a lack of culturally competent mental health services.

b. Incorrect See explanation for response a.
c. Incorrect See explanation for response a.
d. Incorrect See explanation for response a.

The correct answer is: cultural values that encourage resolution of problems within the family.

156
Q

You receive an order from the court to conduct a presentencing evaluation of a defendant who has been convicted of murder. You should:
Select one:

A.
obtain a signed release of information from the defendant before conducting the evaluation.

B.
discuss the limits of confidentiality with the defendant even though a release of information is not legally required in this situation.

C.
say nothing about confidentiality since a release of information is not legally required and discussing confidentiality may decrease the defendant’s willingness to be truthful.

D.
proceed with the evaluation since a release of confidentiality will have already been obtained by the court.

A

In general, the results of an evaluation are not revealed without the individual’s consent. However, this is modified when the evaluation is being conducted in response to a court-order.
a. Incorrect See explanation for response b.

b. CORRECT This issue is addressed by G. B. Melton et al. in their discussion of court-ordered evaluations (Psychological evaluations for the court, New York, Guilford, 1987). They note that the therapist-patient privilege does not apply when the therapist-patient relationship “is the creation of the court” (p. 46), which is the case in a court-ordered evaluation. However, from an ethical perspective, you should always warn an individual of the limits of confidentiality.
c. Incorrect See explanation for response b.
d. Incorrect See explanation for response b.

The correct answer is: discuss the limits of confidentiality with the defendant even though a release of information is not legally required in this situation.

157
Q

Studies comparing the age at which gays and lesbians first disclose their sexual orientation to a friend, family member, or other person have found that:
Select one:

A.
the average age of disclosure for gay men is 2 to 3 years earlier than the average age of disclosure for lesbians.

B.
the average age of disclosure for gay men is 5 to 6 years earlier than the average age of disclosure for lesbians.

C.
the average age of disclosure for lesbians is 3 to 4 years earlier than the average age of disclosure for gay men.

D.
the average ages of disclosure for gay men and lesbians are not significantly different.

A

This is a difficult question because this issue has been investigated in only a few empirical studies.
a. Incorrect See explanation for response d.

b. Incorrect See explanation for response d.
c. Incorrect See explanation for response d.
d. CORRECT In a recent study, R. C. Savin-Williams and L. M. Diamond compared the sexual identity milestones for gay males and lesbians and found that, in terms of disclosure, the average age was not substantially different. They also report the results of previous research, which is generally consistent with their findings. Note that Savin-Williams and Diamond did find significant differences for initial same-sex sexual contact and first self-labeling as gay or lesbian, with the average age for both milestones being lower for gay men than for lesbians [Sexual identity trajectories among sexual-minority youths: Gender comparisons, Archives of Sexual Behavior, 29, 607-627, 2000].

The correct answer is: the average ages of disclosure for gay men and lesbians are not significantly different.

158
Q

Berscheid’s (1989) “emotion-in-relationships” model predicts that strong emotions in close relationships are:
Select one:

A.
related to the degree to which prepotent needs are mutually satisfied.

B.
a function of each partners level of differentiation.

C.
moderated by personal beliefs about the acceptability of emotional expression.

D.
elicited by unexpected interruptions in routine interactions.

A

According to E. Berscheid (1989), a partner is most likely to experience a high degree of emotion in an intimate relationship when his/her partner’s behavior disrupts an organized sequence of behavior.
a. Incorrect See explanation for response d.

b. Incorrect See explanation for response d.
c. Incorrect See explanation for response d.
d. CORRECT Berscheid’s emotion-in-relationships model predicts that disruptions in organized sequences or patterns of behavior (established interactions between the partners) produce strong emotions in close relationships, with the nature of the emotion (positive or negative) depending on the consequence of the disruption.

The correct answer is: elicited by unexpected interruptions in routine interactions.

159
Q

The split-plot ANOVA is used when:
Select one:

A.
one of the independent variables is an extraneous variable.

B.
all independent variables are quantitative.

C.
the researcher has used a “matched-pair” design.

D.
the researcher has used a “mixed” design.

A

The split-plot ANOVA is one of several forms of the ANOVA that you want to be familiar with for the exam.
a. Incorrect See explanation for response d.

b. Incorrect See explanation for response d.
c. Incorrect See explanation for response d.
d. CORRECT Knowing that the split-plot ANOVA is also referred to as the mixed ANOVA would have helped you identify this as the correct answer. For additional information on the split-plot ANOVA, see the Statistics and Research Design chapter of the written study materials.

The correct answer is: the researcher has used a “mixed” design.

160
Q

To determine how well an examinee did on a test compared to other examinees, you would use:
Select one:

A.
criterion-referenced interpretation.

B.
domain-referenced interpretation.

C.
objectives-referenced interpretation.

D.
norm-referenced interpretation.

A

An individual examinee’s score on a test can be interpreted in several ways.
a. Incorrect When using criterion-referenced interpretation, an examinee’s test performance is interpreted in terms of an external criterion (i.e., in terms of a specific standard of performance).

b. Incorrect Domain-referenced interpretation is a type of criterion-referenced interpretation. It is used to determine how much of a specific knowledge domain the examinee has mastered.
c. Incorrect Objectives-referenced interpretation is another type of criterion-referenced interpretation. It involves interpreting an examinee’s performance in terms of achievement of instructional objectives.
d. CORRECT When using norm-referenced interpretation, an examinee’s test performance is compared to the performance of members of the norm group (other people who have taken the test).

The correct answer is: norm-referenced interpretation.

161
Q

Bell and Ainsworth (1972) conducted one of the first longitudinal studies on the impact of maternal responsiveness on infant crying. The results of their research indicated that a mother’s prompt and consistent response to her infant’s crying during the first few months of the infant’s life:
Select one:

A.
has no effect on the amount of infant crying in subsequent months.

B.
is associated with an increase in the amount of infant crying in subsequent months.

C.
is associated with a decrease in the amount of infant crying in subsequent months.

D.
is associated with a decrease in the amount of infant crying in subsequent months only for infants high in irritability.

A

Research on the impact of caregiver responsiveness and infant crying has, like research on a number of other issues, produced inconsistent results. However, this question is asking about the results of a specific study rather than the conclusion that can be drawn from the existing studies.
a. Incorrect See explanation for response c.

b. Incorrect See explanation for response c.
c. CORRECT Bell and Ainsworth (1972) found that consistent and prompt maternal response to infant crying during the first three months of the infant’s life was associated with a reduced frequency and shorter duration of crying and fussing in subsequent months. Note, however, that their study was criticized on methodological grounds and that subsequent studies have provided both confirming and disconfirming results. See~ e.g.~ A. S. Regine & S. Maarit~ Natural parenting - back to basics in infant care~ Evolutionary Psychology~ 5(1)~ 102-183~ 2007.
d. Incorrect Research by Bell and Ainsworth and other investigators has not found that level of infant irritability mediates the relationship between caregiver responsiveness and infant crying.

The correct answer is: is associated with a decrease in the amount of infant crying in subsequent months.

162
Q

Adults with ADHD are most likely to:
Select one:

A.
be excessively perfectionistic in their work.

B.
prefer jobs that involve routine, repetitive tasks.

C.
avoid relationships, especially sexual relationships.

D.
frequently change jobs and intimate partners.

A

Knowing that ADHD in adulthood is associated with prominent symptoms of inattention and impaired social and occupational functioning would have helped you identify the correct answer to this question.

a. Incorrect Because of their impulsivity (e.g., tendency to make quick decisions) and attention deficits, adults with ADHD are not likely to be perfectionistic.
b. Incorrect This is the opposite of what is true. A person with ADHD gets bored easily and would not prefer routine, repetitive tasks.
c. Incorrect Adults with ADHD often have trouble maintaining relationships but do not avoid them. With regard to sexual relationships, adults with ADHD tend to have more sexual partners than adults without ADHD and may engage in more impulsive or risky sexual behaviors.
d. CORRECT The research has found that adults with ADHD change jobs and intimate partners more often than do individuals without the disorder.

The correct answer is: frequently change jobs and intimate partners.

163
Q

Which of the following individuals is most likely to have hypertension?
Select one:

A.
a 52-year-old White man

B.
a 43-year-old Hispanic woman

C.
a 35-year-old Asian man

D.
a 65-year-old African-American woman

A

Research has confirmed that prevalence rates for hypertension are related to age, gender, and race/ethnicity.
a. Incorrect See explanation for response d.

b. Incorrect See explanation for response d.
c. Incorrect See explanation for response d.
d. CORRECT Information on the prevalence rates of hypertension in the United States are provided by the National Health and Nutrition Examination Survey, which found the rates to be highest for African-Americans (non-Hispanic Blacks), older individuals, and women, with African-American women aged 60 and over having the highest rate. See K. L. Ong et al., Prevalence, awareness, treatment, and control of hypertension among United States adults, 1999-2004, Hypertension, 2007, 49(1), 69-75.

The correct answer is: a 65-year-old African-American woman

164
Q

To determine if cigarette smoking is better controlled through aversion therapy, self-hypnosis, or cognitive self-control, a researcher randomly assigns smokers of at least two packs of cigarettes a day to one of the treatment groups and determines the average number of cigarettes each participant smokes each day during the 30 days following administration of the appropriate intervention. The scale of measurement of the dependent variable in this study is which of the following?
Select one:

A.
ordinal

B.
ratio

C.
interval

D.
nominal

A

The dependent variable in this study is number of cigarettes smoked.
a. Incorrect See explanation for response b.

b. CORRECT A ratio scale has the properties of order, equal intervals, and an absolute zero point – and number of cigarettes has all three properties.
c. Incorrect See explanation for response b.
d. Incorrect See explanation for response b.

The correct answer is: ratio

165
Q

As described by Kobasa (1979), the three components of “hardiness” are:
Select one:

A.
attachment, caring, and intimacy.

B.
commitment, control, and challenge.

C.
intellect, emotion, and will.

D.
what we do, what we think, and what we feel.

A

Hardiness is a psychological style that has been linked to the ability to maintain good physical health under conditions of stress.
a. Incorrect See explanation for response b.

b. CORRECT According to S. C. Kobasa, hardy people (1) have a strong sense of commitment to their lives and work, (2) believe they can control or influence the events they encounter, and (3) view change as a positive challenge (Stressful life events, personality, and health: An inquiry into hardiness. Journal of Personality and Social Psychology, 37, 1-11, 1979).
c. Incorrect See explanation for response b.
d. Incorrect See explanation for response b.

The correct answer is: commitment, control, and challenge.

166
Q

The standard error of the mean is used to:
Select one:

A.
construct a confidence interval around an obtained test score.

B.
construct a confidence interval around a predicted criterion score.

C.
calculate the standard deviation of the sampling distribution.

D.
calculate an estimate of the standard deviation of test scores in a population.

A

There are three standard errors that you want to be familiar with for the exam - the standard error of the mean, the standard error of measurement, and the standard error of estimate.

a. Incorrect The standard error of measurement is used to construct a confidence interval around an obtained (measured) test score.
b. Incorrect The standard error of estimate is used to construct a confidence interval around a predicted (estimated) criterion score.
c. CORRECT The standard error of the mean is the standard deviation of the sampling distribution of the mean. It is calculated by dividing the population standard deviation by the square root of the sample size.
d. Incorrect The standard deviation of test scores for a sample can be used to estimate the standard deviation of test scores in the population by using (N-1) instead of N in the denominator of the formula for the standard deviation.

The correct answer is: calculate the standard deviation of the sampling distribution.

167
Q

The Buckley Amendment:
Select one:

A.
establishes the right of the school psychologist to determine who should have access to information included in school records.

B.
establishes the right of parents to inspect the school records of their children.

C.
establishes the right of patients or their legal representatives to have access to their hospital records.

D.
requires that parental consent be obtained before a student can be subjected to a psychological or psychiatric examination.

A

The Buckley Amendment is another name for the Family Education Rights and Privacy Act (FERPA).
a. Incorrect See explanation for response b.

b. CORRECT Among other things, the Buckley Amendment states that any school district may be denied federal funds if parents of students (or other legal guardians) or students who have reached the age of majority are not given access to their school records. It also prohibits disclosure of school records to unauthorized individuals without parental consent. See the chapter on ethics and professional issues in the written study materials for additional information on the Buckley Amendment.
c. Incorrect See explanation for response b.
d. Incorrect See explanation for response b.

The correct answer is: establishes the right of parents to inspect the school records of their children.

168
Q

The majority of synapses in the human nervous system are ________ synapses.
Select one:

A.
electrical

B.
chemical

C.
electrochemical

D.
mechanical

A

A synapse is the point where a nerve impulse travels from one neuron to another neuron.

a. Incorrect Electrical synapses are rare in the human nervous system and occur at “gap junctions.” Electrical synapses connect some neurons, glial cells, and cardiac and smooth muscle cells.
b. CORRECT Most synapses in the human nervous system are chemical synapses that involve the release of a neurotransmitter by a presynaptic neuron into the synaptic cleft. The neurotransmitter then binds to receptors on the postsynaptic neuron.
c. Incorrect See explanation above.
d. Incorrect See explanation above.

The correct answer is: chemical

169
Q

E. H. Schein (1992) proposed that organizational culture exists on which of the following three levels?
Select one:

A.
time orientation, space orientation, and activity orientation

B.
microsystem, mesosystem, and macrosystem

C.
observable artifacts, espoused values and beliefs, and underlying assumptions

D.
informal organizational elements, formal organizational elements, and individual characteristics of employees

A

According to Schein (1992), organizational culture provides stability and meaning. Although culture can reduce anxiety among organizational members that is caused by uncertainty about current and future events, it can also impede change.
a. Incorrect These are three of the six value orientations that Kluckhohn and Strodbeck (1961) used to describe differences among cultural groups. The other three dimensions are relation to nature, view of human nature, and view of relationships among people.

b. Incorrect These are three of the five systems or levels of Bronfenbrenner’s (2004) ecological model of developmental influences.
c. CORRECT These are the three levels of organizational culture identified by Schein.
d. Incorrect According to Nadler (1988), these are three of the four factors that must be addressed to accomplish planned change in organizations. The fourth factor is characteristics of the employees and managerial tasks.

The correct answer is: observable artifacts, espoused values and beliefs, and underlying assumptions

170
Q

Dr. Prattle is asked by a local radio station if she would be interested in doing a “talk show” on Saturday mornings. On the show, she will respond to people who call the station. To be consistent with the provisions of the Ethics Code, Dr. Prattle should:
Select one:

A.
refuse to appear on the show.

B.
avoid giving direct advice of a psychological nature to callers.

C.
be sure that callers understand that their conversations with her do not constitute a therapeutic relationship.

D.
be sure that she has adequate malpractice insurance.

A

As noted by M. B. Canter, et al. (1994), in their discussion of the Ethics Code, the ethical requirements with regard to media psychology are “still evolving.” However, since this question refers specifically to the Code, the best answer is the one that best resembles the Code’s language. a. Incorrect It’s not necessary to refuse to do the show, but certain precautions should be taken. b. Incorrect Discussions of media psychology in the literature have generally concluded that advice is actually better than therapy in this context. c. CORRECT Of the responses, this one comes closest to the language of Standard 5.04 (Media Presentations). Moreover, Canter, et al. note that on radio talk shows, the psychologist should be sure that he/she clearly distinguishes between “media psychology” and “therapy” (Ethics for psychologists: A commentary on the APA Ethics Code, Washington, DC, APA, 1994).d. Incorrect This is probably a good idea, but not the best response to this question!

The correct answer is: be sure that callers understand that their conversations with her do not constitute a therapeutic relationship.

171
Q

Kochanska’s (1997) research suggests that, during the toddler years, the optimal parental behavior for ensuring the development of conscience in a child depends on the child’s:
Select one:

A.
activity level.

B.
level of fearfulness.

C.
intelligence.

D.
attachment.

A

G. Kochanska proposes that “different socialization mechanisms may promote conscience development in children with different temperamental individualities” (p. 228); and she is most interested in the temperament trait of fearfulness. [Multiple pathways to conscience for children with different temperaments: From toddlerhood to age 5, Developmental Psychology, 33(2), 1997, 228-240.]
a. Incorrect See explanation for response b.

b. CORRECT A number of researchers have found that fearful children score higher on measures of conscience, and Kochanska extended this finding by showing that the relationship between fearfulness and conscience development is affected by caregivers’ socialization practices. Specifically, she found that, among fearful toddlers, conscience development is fostered when the mother uses “gentle discipline.” In contrast, among fearless toddlers, conscience development depends more on a secure mother-child attachment and maternal responsiveness.
c. Incorrect See explanation for response b.
d. Incorrect See explanation for response b.

The correct answer is: level of fearfulness.

172
Q

Generalized Anxiety Disorder (GAD) has been found to have a high comorbidity rate and, according to the results of the National Comorbidity Survey Replication, the most common co-diagnosis is:
Select one:

A.
Obsessive-Compulsive Disorder.

B.
Posttraumatic Stress Disorder.

C.
Social Phobia.

D.
Separation Anxiety Disorder.

A

Answer C is correct: The National Comorbidity Survey Replication (NCS-R) found that GAD frequently co-occurs with Mood Disorders, Substance-Related Disorders, and other Anxiety Disorders [Kessler, R. C. et al., Prevalence, severity, and comorbidity of 12-month DSM-IV disorders in the National Comorbidity Survey Replication, Archives of General Psychiatry, 62, 617-709, 2005]. Although reported comorbidity rates for GAD vary somewhat from study to study, in the NCS-R, the highest correlation (.47) was found between GAD and Social Phobia. (Note that the earlier National Comorbidity Survey found the highest correlation for GAD and Simple Phobia, but this question is asking about the NCS-R.)

Answer A: The NCS-R reports a correlation of .33 between GAD and OCD.

Answer B: The NCS-R reports a correlation of .44 between GAD and PTSD.

Answer D: The NCS-R reports a correlation of .36 between GAD and Separation Anxiety Disorder.
The correct answer is: Social Phobia.

173
Q

Identifying alternative behaviors to replace targeted undesirable behaviors is an integral part of which of the following behavioral techniques?
Select one:

A.
response cost

B.
implosive therapy

C.
contingency management

D.
negative practice

A

While some behavioral techniques are designed specifically to either decrease or increase a behavior, others attempt to accomplish both goals.
a. Incorrect Response cost is used to decrease an undesirable behavior.

b. Incorrect Implosive therapy is used to extinguish a conditioned response and does not attempt to deliberately substitute an alternative response (although this may, of course, occur).
c. CORRECT “Contingency” refers to the consequence of a behavior, and contingency management involves identifying and applying punishments for undesirable behaviors and identifying and applying reinforcements for desirable ones.
d. Incorrect When using negative practice, the person is required to deliberately repeat the undesirable behavior. It has been found to be an effective treatment for tics.

The correct answer is: contingency management

174
Q

Job evaluation is to wages as needs assessment is to:
Select one:

A.
benefits.

B.
selection.

C.
vocational choice.

D.
training.

A

For the exam, you want to be able to distinguish between job analysis, job evaluation, and needs analysis.
a. Incorrect See explanation for response d.

b. Incorrect See explanation for response d.
c. Incorrect See explanation for response d.
d. CORRECT A needs analysis (or assessment) is conducted in organizations to identify training needs and determine the appropriate content for training programs.

The correct answer is: training.

175
Q

Although there is currently no cure for shingles, the duration of an outbreak may be shortened by use of which of the following drugs?
Select one:

A.
diphenhydramine or other antihistamine

B.
acyclovir or other antiviral medication

C.
a beta blocker

D.
an antifungal cream

A

Knowing that shingles is due to reactivation of the herpes zoster (chicken pox) virus would have helped you identify the correct answer to this question.
a. Incorrect See explanation for response b

b. CORRECT Antiviral medication can help reduce the duration of shingles when the drug is taken within 72 hours after development of the rash. Other drugs may be taken to help alleviate the pain caused by shingles such as oxycodine, amitriptyline or other tricyclic antidepressant, and a topical corticosteroid may be used to decrease inflammation.
c. Incorrect See explanation for response b.
d. Incorrect See explanation for response b.

The correct answer is: acyclovir or other antiviral medication

176
Q

When using a ___________________, the rater is provided with a list of behavioral examples and asked to indicate whether the ratee’s performance is equal to, worse than, or better than the performance described in each example.
Select one:

A.
critical incident technique

B.
behavioral observation scale

C.
forced-choice method

D.
mixed standard scale

A

The mixed standard scale (Blanz and Ghiselli, 1971) was originally designed to reduce rater biases. Note that, if you’re unfamiliar with this technique, you may have been able to identify it as the correct answer to this question using the process of elimination.
a. Incorrect See explanation for response d. (Although the mixed standard scale uses critical incidents, it is not the same as the “critical incident technique.”)

b. Incorrect See explanation for response d.
c. Incorrect See explanation for response d.
d. CORRECT When using the mixed standard scale, three critical incidents representing good, average, and poor performance for several dimensions of job performance are identified. They are then listed in the rating scale in a random order, and the rater indicates, for each incident, whether the rater’s performance is equal to, worse than, or better than the described behavior.

The correct answer is: mixed standard scale

177
Q

Research on primacy and recency effects has shown that:
Select one:

A.
memory for the first item in a list decreases while memory for the last item increases as the delay between study and recall increases.

B.
memory for the first item in a list increases while memory for the last item decreases as the delay between study and recall increases.

C.
memory for the first and the last item decreases as the delay between study and recall increases.

D.
memory for the first and the last item increases as the delay between study and recall increases.

A

This question is asking about research on the serial position effect, which identified a “recency-primacy shift.”
a. Incorrect See explanation for response b.

b. CORRECT The recency-primacy shift contradicts, to some extent, the axiom that “memory tends to fade over time.” Specifically, the research has shown that, as the delay between study and recall increases, memory for the last item in a list decreases but memory for the first item increases – i.e., there is a shift from a recency effect to a primacy effect.
c. Incorrect See explanation for response b.
d. Incorrect See explanation for response b.

The correct answer is: memory for the first item in a list increases while memory for the last item decreases as the delay between study and recall increases.

178
Q

Studies suggest that the ______________ of Obsessive-Compulsive Disorder are similar across cultures.
Select one:

A.
prevalence rates

B.
prevalence rates and age of onset

C.
prevalence rates, age of onset, and gender distribution

D.
prevalence rates, age of onset, gender distribution, and comorbidity

A

The studies have generally found that OCD does not differ significantly across different cultures.

a. Incorrect See explanation for response d.
b. Incorrect See explanation for response d.
c. Incorrect See explanation for response d.
d. CORRECT The DSM-5 notes that prevalence rates, age of onset, gender distribution, comorbidity, and symptom structure of OCD are similar across cultures (APA, 2013, p. 140). However, the content of obsessions and compulsions may be affected by cultural factors.

The correct answer is: prevalence rates, age of onset, gender distribution, and comorbidity

179
Q

A psychologist is hired by a defendant’s attorney to evaluate the defendant for a possible insanity defense. Prior to conducting the evaluation, the psychologist:
Select one:

A.
should inform the defendant that all information provided during the evaluation is confidential and will be revealed only to his attorney.

B.
should inform the defendant that information will be provided only to his attorney unless the defendant and his attorney decide to use the results of the evaluation to support an insanity defense.

C.
should inform the defendant that information related to his mental state at the time he committed the crime is not confidential and will be provided to the court and the opposing attorney.

D.
should inform the defendant that any information he provides during the course of the evaluation will be revealed not only to his attorney but also to the court and the opposing attorney.

A

When hired by a defendant’s attorney to evaluate the defendant for a court case, the limits of confidentiality must be discussed prior to conducting the evaluation.
a. Incorrect See explanation for response b.

b. CORRECT In this situation, the client should be advised that information will be shared only with his attorney unless the client and his attorney decide to use it to support an insanity or similar defense. See, e.g., G. B. Melton et al., Psychological evaluation for the courts, New York, Guilford Press, 1997.
c. Incorrect See explanation for response b.
d. Incorrect See explanation for response b.

The correct answer is: should inform the defendant that information will be provided only to his attorney unless the defendant and his attorney decide to use the results of the evaluation to support an insanity defense.

180
Q
Congenital cytomegalovirus (CMV) is the second-most common cause of an intellectual disability after Down Syndrome. In additional to cognitive impairments, it is most likely to cause which of the following?
Select one:

A.
facial deformities

B.
heart and lung abnormalities

C.
genital, urinary, and intestinal abnormalities

D.
hearing and visual impairments

A

CMV is a type of herpes virus. Although infection with CMV during adulthood usually produces no or mild symptoms, infection during prenatal development or birth can be life-threatening.
a. Incorrect See explanation for response d.

b. Incorrect See explanation for response d.
c. Incorrect See explanation for response d.
d. CORRECT Approximately 10% of infants infected with CMV show symptoms at birth, and about 10 to 15% of infants who are asymptomatic at birth develop symptoms during the first few years of life. Common symptoms include some degree of intellectual disability and hearing and visual impairments.

The correct answer is: hearing and visual impairments

181
Q

On the WAIS-IV, a Full Scale IQ score of 85 is ________ below the mean.
Select one:

A.
one standard deviation

B.
one and one-half standard deviations

C.
two standard deviations

D.
three standard deviations

A

Answer A is correct: This is an easy question as long as you know that the WAIS-IV Full Scale IQ has a mean of 100 and standard deviation of 15. On the WAIS-IV, a score of 85 is one standard deviation below the mean (100 - 15 = 85).

The correct answer is: one standard deviation

182
Q

A diagnosis of Persistent Depressive Disorder requires the presence of a depressed mood on most days for at least ________ for adults and at least ________ for children and adolescents.
Select one:

A.
one year; six months

B.
18 months; one year

C.
two years; one year

D.
36 months; 18 months

A

Answer C is correct: The duration requirement is the same for Cyclothymic Disorder and Persistent Depressive Disorder (Dysthymia) - at least two years for adults and one year for children and adolescents.

Answers A, B, and D: See explanation for response C.
The correct answer is: two years; one year

183
Q

Papez’s circuit is involved in which of the following?
Select one:

A.
control of emotions

B.
control of circadian rhythms

C.
emergence of the secondary sex characteristics

D.
transfer of visual input to the visual cortex

A

Knowing that the Papez circuit is one of the major pathways of the limbic system and that the limbic system is referred to as the “emotional brain” would have helped you identify the correct answer to this question.
a. CORRECT Papez (1937) was among the first researchers to propose the existence of a neuroanatomical circuit that mediates emotion. As described by Papez, this circuit includes the hippocampus, mammillary bodies, cingulate gyrus, and anterior nuclei of the thalamus. In addition to controlling the experience and expression of emotions, it plays a role in the acquisition of new memories.

b. Incorrect See explanation for response a.
c. Incorrect See explanation for response a.
d. Incorrect See explanation for response a.

The correct answer is: control of emotions

184
Q

The Strong Interest Inventory would probably be LEAST useful:
Select one:

A.
as a job selection test to predict future performance.

B.
as a guide for choosing a college major.

C.
as a guide for choosing a career.

D.
as a tool for predicting job turnover.

A

Longitudinal studies on the original version of the Strong Interest Inventory (the SVIB) have consistently shown that interests are not good predictors of academic or occupational success.
a. CORRECT Research has found interest tests to be good predictors of academic and job choice and job persistence and satisfaction which are listed as the other alternatives. However, they are not accurate predictors of job performance.

b. Incorrect See explanation for response a.
c. Incorrect See explanation for response a.
d. Incorrect See explanation for response a.

The correct answer is: as a job selection test to predict future performance.

185
Q

Which of the following is LEAST likely to be true about a 55-year-old Asian client?
Select one:

A.
When she smiles, this is more likely to indicate confusion or embarrassment than pleasure.

B.
She will be uncomfortable discussing personal details about her family relationships.

C.
When she avoids eye contact, this will communicate respect rather than inattention or disrespect.

D.
She will respond best to an indirect, unstructured therapeutic approach.

A

It is very important not to overgeneralize the characteristics of members of culturally diverse groups. However, the licensing exam sometimes includes questions like this one that require you to do so.
a. Incorrect Among Asians and Asian Americans, a smile is more likely to signal embarrassment or confusion than pleasure.

b. Incorrect Asians are generally uncomfortable discussing personal details about family members.
c. Incorrect Avoidance of eye contact is a sign of respect in the Asian culture.
d. CORRECT Although not all Asian and Asian American clients will prefer a direct, structured approach (rather than an indirect, unstructured approach), this is often the preferred strategy.

The correct answer is: She will respond best to an indirect, unstructured therapeutic approach.

186
Q

Dr. Street, a clinical psychologist, has been treating Bob B. for about five months. Bob originally came to therapy to resolve the depression he had been experiencing since his divorce and, in the past few weeks, Dr. Street has noticed a marked improvement in Bob’s symptoms. During one session, Bob says that, for some time, he has been having trouble going anywhere that requires him to use an elevator since being confined in small spaces with other people makes him very anxious. He says he has had this problem for some time, but that it seems to have gotten worse recently. Dr. Street has not had much experience in treating phobic reactions and therefore decides to refer Bob to a behavioral therapist. However, even though Bob has started behavioral therapy with another therapist, Dr. Street continues to see Bob in order to continue working on Bob’s depression. In terms of ethical practices, this arrangement is:
Select one:

A.
unethical since Dr. Street should have offered Bob a choice of therapists.

B.
unethical since Dr. Street should terminate his relationship with Bob while Bob is receiving treatment from another therapist.

C.
ethical as long as the therapists cooperate with one another during the course of Bob’s treatment.

D.
ethical but therapeutically questionable since Bob’s phobia is probably related to his divorce.

A

Dr. Street’s actions are consistent with the provisions of the Ethics Code. He has recognized his professional limitations and has taken action to ensure that his client receives the best possible treatment.
a. Incorrect Referral to a specific professional is not prohibited although it would be advisable, when appropriate, to give the client a choice of alternatives. This, however, is not the best answer of those given.

b. Incorrect Seeing a client who is seeing another professional is only a problem when the client is receiving similar services from the other professional, which is not the case in this situation.
c. CORRECT As noted above, Dr. Street has acted in an ethical and responsible manner by referring Bob to another therapist and will continue to do so as long as he cooperates with the other therapist during the course of Bob’s treatment.
d. Incorrect There is insufficient information to support this conclusion.

The correct answer is: ethical as long as the therapists cooperate with one another during the course of Bob’s treatment.

187
Q

Following the death of his mother, Dr. Beasley, a clinical psychologist, finds he is having trouble sleeping, has lost his appetite, and is having some trouble concentrating when listening to his clients during therapy sessions. In this situation, to fulfill the requirements of the APA’s Ethics Code, Dr. Beasley should:
Select one:

A.
begin therapy to work through his bereavement.

B.
temporarily suspend his practice until he feels better and refer his clients to another therapist.

C.
temporarily reduce his caseload to a more manageable level.

D.
seek consultation to determine if he should temporarily suspend his practice or take some other course of action.

A

To be consistent with the provisions of the Ethics Code, Dr. Beasley should seek competent professional assistance to help him determine if he should suspend, terminate, or otherwise modify his clinical practice because of his current personal problems.
a. Incorrect This is a possible course of action for Dr. Beasley, but answer d is a better answer because it addresses the ethical issue of what Dr. Beasley should do to best protect the welfare of his clients.

b. Incorrect This course of action might also be acceptable, but it is not the best answer.
c. Incorrect While this might be an acceptable action, answer d is a better answer.
d. CORRECT Dr. Beasley’s first course of action should be to seek advice to help him decide what to do.

The correct answer is: seek consultation to determine if he should temporarily suspend his practice or take some other course of action.

188
Q

Your client is convinced that, for the past three months, the government has been tapping his phone, reading his emails, and having someone follow him. He states that he believes that, at one point, he was drugged by “an agent” and was “out for a day or two.” The client works as a computer programmer for a medical records company, has been married for three years, and has one child. He says that, despite the government’s interference in his life, everything else seems to be okay. What is the most likely DSM-5 diagnosis for this client?
Select one:

A.
Paranoid Personality Disorder

B.
Brief Psychotic Disorder

C.
Delusional Disorder

D.
Schizophrenia

A

Answer C is correct: The client’s symptoms are most suggestive of Delusional Disorder, persecutory type.

Answer A: The client has a single delusional belief that is limited to the government being out to get him rather than a pervasive tendency to behave, feel, and perceive the world as hostile and persecutory, which would be more indicative of a personality disorder.

Answer B: Despite his delusion about the government, the client appears to be functioning well at work and home. Brief Psychotic Disorder would cause greater impairment in functioning, and its diagnosis requires a duration of one day to less than one month.

Answer D: The nature of the client’s symptoms and their duration do not meet the diagnostic criteria for Schizophrenia.

The correct answer is: Delusional Disorder

189
Q

Which of the following contributes LEAST to depth perception for objects at great (versus short) distances?
Select one:

A.
interposition

B.
linear perspective

C.
retinal disparity

D.
motion parallax

A

Depth perception depends on a combination of binocular and monocular cues. Binocular cues are responsible for depth perception of objects at relatively short distances, while monocular cues contribute to depth perception for objects at greater distances.
a. Incorrect Interposition is a monocular cue and occurs when one object is blocked by another.

b. Incorrect Linear perspective is a monocular cue and refers to the tendency for parallel lines to appear to converge as they approach the horizon.
c. CORRECT Retinal disparity is a binocular cue and is also known as stereopsis. It refers to the fact that our two eyes see objects in the world from two different views, and, the closer an object, the greater the disparity of the two images.
d. Incorrect Motion parallax is a monocular cue and refers to the apparent relative motion of stationary objects against a background when the observer moves.

The correct answer is: retinal disparity

190
Q

A therapist can expect that an Asian American client will be LEAST likely to do which of the following?
Select one:

A.
use silence as a way of communicating respect

B.
use smiling to express disapproval or disagreement

C.
avoid expressing strong emotions

D.
avoid eye contact when listening to the therapist

A

Asian Americans are a heterogeneous group and caution must be taken in drawing general conclusions about their behaviors. However, several generalizations are made in the multicultural counseling literature. See, e.g., D. W. Sue and D. Sue, Counseling the culturally different, New York, John Wiley & Sons, 1999.
a. Incorrect Asian Americans tend to demonstrate respect for authorities by “not speaking until spoken to.”

b. CORRECT For some Asian Americans, smiling may demonstrate uneasiness or embarrassment, while, for others, smiling is avoided because it is considered a sign of weakness.
c. Incorrect This is characteristic of Asian Americans.
d. Incorrect This is also characteristic of Asian Americans.

The correct answer is: use smiling to express disapproval or disagreement

191
Q

A test’s convergent and divergent (discriminant) validity provide information about its:
Select one:

A.
incremental validity

B.
construct validity

C.
content validity

D.
face validity

A

Answer B is correct: One way to evaluate a test’s construct validity is to determine if it has high correlations with measures of the same or related traits (convergent validity) and low correlations with measures of unrelated traits (divergent validity).

The correct answer is: construct validity

192
Q

A Mexican American client tells his psychologist that he wants to consult a curandero about the problems he is having. To be consistent with the requirements of the APA’s Guidelines for Providers of Psychological Services to Ethnic, Linguistic, and Culturally Diverse Populations, the psychologist should:
Select one:

A.
explain any concerns she has about the incompatibility of her approach with the approach of a traditional folk healer.

B.
find out as much about curanderos as possible and incorporate what she learns into her treatment of the client.

C.
refer the client to a therapist who is more familiar with traditional Mexican culture.

D.
support the client in his desire to work with a curandero and express her willingness to collaborate.

A

This issue is addressed in Paragraph 5 of the Guidelines.
a. Incorrect See explanation for response d.

b. Incorrect See explanation for response d.
c. Incorrect See explanation for response d.
d. CORRECT Paragraph 5(b) states that “effective psychological intervention may be aided by consultation with and/or inclusion of religious/spiritual leaders/practitioners relevant to the client’s cultural and belief systems.” This response is most consistent with this provision.

The correct answer is: support the client in his desire to work with a curandero and express her willingness to collaborate.

193
Q

The phenomena known as retroactive and proactive interference are most useful for understanding which of the following?
Select one:

A.
habituation versus sensitization

B.
anterograde amnesia

C.
memory consolidation

D.
forgetting (loss of memory)

A

Retroactive and proactive interference provide an alternative explanation to trace decay theory for the inability to recall previously acquired information.
a. Incorrect See explanation for response d.

b. Incorrect See explanation for response d.
c. Incorrect See explanation for response d.
d. CORRECT Proactive interference occurs when previously learned information hinders the ability to recall information learned more recently; while retroactive interference occurs when information acquired more recently hinders the ability to recall previously acquired information.

The correct answer is: forgetting (loss of memory)

194
Q

Which of the following is true about gender differences in newborns?
Select one:

A.
Newborn girls have higher morbidity and mortality rates than do newborn boys.

B.
Newborn boys have higher morbidity and mortality rates than do newborn girls.

C.
Newborn girls and boys have similar morbidity and mortality rates.

D.
Newborn girls have higher mortality rates, but newborn boys have higher morbidity rates.

A

A consistent finding of the research is a gender difference in morbidity and mortality rates among newborns.
a. Incorrect See explanation for response b.

b. CORRECT Male newborns are not only less mature than female newborns but are also have higher mortality rates and are more vulnerable to a variety of illnesses. See, e.g., J. C. K. Wells, Natural selection and sex differences in morbidity and mortality in early life, Journal of Theoretical Biology, 202(1), 65-76, 2000.
c. Incorrect See explanation for response b.
d. Incorrect See explanation for response b.

The correct answer is: Newborn boys have higher morbidity and mortality rates than do newborn girls.

195
Q

During the fourth session, a therapy client says he “is already feeling much better.” Most likely:
Select one:

A.
this is simply a transitory placebo effect.

B.
this reflects the initial development of insight.

C.
the client is beginning to feel more hopeful about his situation.

D.
the client is starting to benefit from newly acquired coping skills.

A

This is a difficult question because you don’t know what research or theory is being asked about. However, responses b, c, and d correspond to the stages identified by Howard et al.’s (1996) phase model of therapeutic effectiveness. Additional information about this model is provided in the Clinical Psychology chapter of the written study materials.
a. Incorrect Although this is a possibility, response b is a better answer since the question specifically mentions that the client’s remark occurs during the fourth session of therapy. A placebo effect could occur at any time during therapy.

b. Incorrect According to the phase model, insight is not achieved until the third stage of therapy (rehabilitation), which usually does not begin until after the 16th therapy session.
c. CORRECT According to Howard and his colleagues, during the first few sessions of therapy, a client experiences an increase in hopefulness. This stage is referred to as remoralization.
d. Incorrect During the second phase of therapy (remediation), the client acquires new coping skills and experiences real symptom relief.

The correct answer is: the client is beginning to feel more hopeful about his situation.

196
Q

According to classical test theory, X = T + E, where E refers to:
Select one:

A.
random error

B.
equivalency

C.
predictive error

D.
estimated consistency

A

According to classical test theory, an examinees obtained test score (X) is composed of two components – T and E. “T” refers to true score variability, while “E” refers to random (measurement) error.

The correct answer is: random error

197
Q

An African American individual in the immersion substage of the immersion-emersion stage of Cross’s (1991) Black Racial Identity Development Model will exhibit which of the following?
Select one:

A.
a lack of interest in race or racial issues

B.
identification with the White (dominant) culture

C.
functional paranoia

D.
intense involvement in African American culture

A

Cross’s (1991) Black Racial Identity Development Model distinguishes between four stages: pre-encounter, encounter, immersion-emersion, and internalization.
a. Incorrect A lack of interest in race is characteristic of the pre-encounter stage.

b. Incorrect African American individuals in the assimilation substage of the pre-encounter stage have adopted a mainstream (White) identity.
c. Incorrect Ridley (1984) used the term functional paranoia to refer to an unhealthy condition that is characterized by mistrust and suspicion. A person in the immersion-emersion stage is more likely to exhibit cultural paranoia, which Ridley defined as a healthy reaction to racism.
d. CORRECT An African American individual in the immersion substage is immersed in African American culture, which is evident in his/her interests, speech, style of dress, actions, choice of friends, etc. This individual also feels a great deal of rage toward Whites. During the emersion substage, the person’s intense emotions subside but he/she continues to reject White culture and begins to develop a more sophisticated African American identity.

The correct answer is: intense involvement in African American culture

198
Q

In a positively skewed distribution of scores:
Select one:

A.
the mean is larger than the median which is larger than the mode.

B.
the median is larger than the mode which is larger than the mean.

C.
the mode is larger than the median which is larger than the mean.

D.
the mode is larger than the mean which is larger than the median.

A

In a positively skewed distribution, the distribution of scores is extended by a few outliers on the positive end of the distribution. Note that knowing that the median is always the middle score in a distribution would have enabled you to eliminate answers b and d.
a. CORRECT Because the mean is affected more than the other measures of central tendency by outliers, it will be the largest number in a positively skewed distribution. The median is always the middle score so it will be second in magnitude, and the mode will be the smallest in magnitude. A figure that illustrates the relationships between the mean, median, and mode is included in the Statistics and Research Design chapter of the written study materials.

b. Incorrect See explanation for response a.
c. Incorrect This describes a negatively skewed distribution.
d. Incorrect See explanation for response a.

The correct answer is: the mean is larger than the median which is larger than the mode.

199
Q

For most children, holophrastic speech begins between the ages of:
Select one:

A.
8 to 12 months.

B.
12 to 18 months.

C.
18 to 24 months.

D.
24 to 36 months.

A

A child is exhibiting holophrastic speech when he/she uses a single word to convey an entire idea - e.g., uses the word “juice” to mean “give me some juice.”
a. Incorrect See explanation for response b.

b. CORRECT Most children begin exhibiting holophrastic speech by 12 to 18 months.
c. Incorrect Children begin exhibiting telegraphic speech (the use of two words to convey an entire sentence) between the ages of 18 and 24 months.
d. Incorrect By about 24 to 36 months, most children use sentences of three or more words.

The correct answer is: 12 to 18 months.

200
Q

An examinee obtains a score of 70 on a test that has a mean of 80, a standard deviation of 15, and a standard error of measurement of 5. The 95% confidence interval for the examinee’s score is:
Select one:

A.
65 to 75

B.
60 to 80

C.
55 to 85

D.
50 to 90

A

Answer B is correct: As described in the Test Construction chapter of the written study materials, the 95% confidence interval for an obtained test score is constructed by multiplying the standard error of measurement by 1.96 and adding and subtracting the result from the examinee’s score. This interval is closest to the 95% confidence interval and was obtained by multiplying the standard error by 2.0 (instead of 1.96) and adding and subtracting the result from the examinee’s score of 70.

The correct answer is: 60 to 80

201
Q

Research on sensory compensation in deaf children has found that:
Select one:

A.
children who are deaf outperform peers without hearing deficits on most visual tasks and this difference is apparent by four to six years of age.

B.
children who are deaf outperform peers without hearing deficits on most visual tasks but this difference is not apparent until preadolescence.

C.
children who are deaf perform similarly to peers without hearing deficits on most visual tasks.

D.
children who are deaf do more poorly than peers without hearing deficits on most visual tasks.

A

Sensory compensation refers to an improvement in one sensory modality when another modality is compromised.
a. Incorrect See explanation for response c.

b. Incorrect See explanation for response c.
c. CORRECT Contrary to the predictions of the sensory compensation hypothesis, deafness is not generally associated with superior visual abilities. One exception is that some deaf children and adults who use sign language exhibit superior visual processing skills on mental rotation, face recognition, and visual attention tasks, but this advantage appears to be due to the use of sign language rather than to deafness itself. See, e.g., J. A. Albertine, Deafness and hearing loss, in I. B. Weiner and W. E. Craighead (Eds.), The Corsini encyclopedia of psychology, Volume 2 (pp. 461-462), Hoboken, NJ, John Wiley & Sons, 2010.
d. Incorrect See explanation for response c.

The correct answer is: children who are deaf perform similarly to peers without hearing deficits on most visual tasks.

202
Q

The APA’s Ethical Principles of Psychologists and Code of Conduct:
Select one:

A.
clearly prohibits a psychologist from having sexual relationships with students and supervisees under any circumstances.

B.
clearly prohibits a psychologist from having sexual relationships with a student or supervisee when he/she is in a position of authority over that person.

C.
prohibits a psychologist from having sexual relationships with students and supervisees indirectly through the “dual relationship” clause.

D.
does not address the issue of sexual relationships with students and supervisees.

A

For the exam, you want to be familiar with the provisions of the Ethics Code regarding sexual relations with supervisees and students so that you can answer questions like this. For additional information on this issue, see the Ethics and Professional Issues chapter of the written study materials.
a. Incorrect See explanation for response b.

b. CORRECT Standard 7.07 states that, “Psychologists do not engage in sexual relationships with students or supervisees who are in their department, agency, or training center or over whom psychologists have or are likely to have evaluative authority.”
c. Incorrect See explanation for response b.
d. Incorrect See explanation for response b.

The correct answer is: clearly prohibits a psychologist from having sexual relationships with a student or supervisee when he/she is in a position of authority over that person.

203
Q

An ongoing debate in the literature centers on a therapist’s responsibility when working with a client who is HIV-positive and is engaging in risky sexual behavior. A central issue in this debate is:
Select one:

A.
social responsibility.

B.
client abandonment.

C.
foreseeability of harm.

D.
vicarious liability.

A

As noted in the chapter on ethics and professional issues in the written study materials, this issue is often discussed within the framework of the Tarasoff decision.
a. Incorrect See explanation for response c.

b. Incorrect See explanation for response c.
c. CORRECT In discussing this topic, S. Knapp and L. VandeCreek, for example, note that three issues are involved in the duty to protect: a fiduciary relationship, an identifiable victim, and the foreseeability of harm. They note that the “issue of foreseeability creates the greatest problems with the duty to protect with HIV-positive patients because the research has not identified all the risk factors involved in HIV transmission” (Application of the duty to protect to HIV-positive patients, Professional Psychology: Research and Practice, 21(3), 161-166, 1990).
d. Incorrect See explanation for response c.

The correct answer is: foreseeability of harm.

204
Q

The ring and little fingers of the hand represent one dermatome and are innervated by which of the following?
Select one:

A.
second cervical nerve (C4)

B.
eighth cervical nerve (C8)

C.
second thoracic nerve (T2)

D.
tenth thoracic nerve (T10)

A

An area of the body that is innervated by the sensory fibers of single nerve root is referred to as a dermatome.
a. Incorrect The skin on the shoulder and clavicular areas is innervated by C4.

b. CORRECT The skin on the medial side of the arm and forearm and the ring and little fingers is innervated by C8.
c. Incorrect The skin on the medial side of the upper arm and the pectoral area of the chest is innervated by T2.
d. Incorrect The skin around the waist at the level of the umbilicus is innervated by T10.

The correct answer is: eighth cervical nerve (C8)

205
Q

Virginia Satir’s approach to family therapy emphasizes:
Select one:

A.
styles of communication.

B.
family projection processes.

C.
boundary disturbances.

D.
transferences between family members.

A

Virginia Satir’s emphasis on communication reflects her affiliation with the Mental Research Institute in Palo Alto.
a. CORRECT Satir distinguishes between five communication styles–placater, blamer, super-reasonable, irrelevant, and congruent.

b. Incorrect See explanation above.
c. Incorrect See explanation above.
d. Incorrect See explanation above.

The correct answer is: styles of communication.

206
Q

A pharmaceutical company asks a psychologist to help it design and conduct a research study on one of its new drugs. The company representative says the company will publish the results of the study only if they are favorable to the drug. The psychologist should:
Select one:

A.
agree to participate only if he is able to approve the final report.

B.
agree to participate only if the company gives him appropriate authorship credit for his work.

C.
agree to participate since it is up to the company to decide what to publish and who to list as an author.

D.
not agree to participate in designing or conducting the study.

A

This situation is not explicitly covered by the Ethics Code. However, several Standards and General Principles are applicable. Principle A, for example, states that “psychologists seek to safeguard the welfare and rights of those with whom they interact professionally and other affected persons.”
a. Incorrect See explanation for response d.

b. Incorrect See explanation for response d.
c. Incorrect See explanation for response d.
d. CORRECT Standard 5.01 also applies to this situation. It states that “psychologists do not knowingly make public statements that are false, deceptive, or fraudulent concerning their research, practice, or other work activities or those of persons or organizations with which they are affiliated.” Agreeing with the company’s decision to publish only supportive data would be supporting a form of deception (and would not be in the best interests of the potential users of the drug). Therefore, this response is most consistent with the “spirit” of the Ethics Code.

The correct answer is: not agree to participate in designing or conducting the study.

207
Q

Studies on information processing during the first two years of life indicate that infants first exhibit recognition memory for up to 24 hours following presentation of a stimulus at about ____ months of age.
Select one:

A.
3

B.
5

C.
7

D.
9

A

Researchers often use habituation to assess information processing skills in infants. Studies using this technique have found that recognition memory skills are apparent at a very early age.
a. CORRECT By three months of age, infants habituate to visual stimuli - i.e., show less response to the second presentation of a stimulus for periods up to 24 hours. This is interpreted as indicating that the infant recognizes the stimulus.

b. Incorrect See explanation above.
c. Incorrect By 7 months, infants begin to show signs of recall memory.
d. Incorrect See explanation above.

The correct answer is: 3

208
Q

“Criterion deficiency” is most likely to result in:

Select one:

A.
high validity.

B.
low validity.

C.
high reliability.

D.
low reliability.

A

Criterion deficiency refers to the degree to which a criterion measure fails to measure all aspects of the ultimate (conceptual) criterion.<p>a. Incorrect See explanation for response b.</p><p>b. CORRECT A criterion measure can have high reliability but low validity - i.e., it can give consistent results but measure only some aspects of the ultimate criterion.</p><p>c. Incorrect See explanation for response b.</p><p>d. Incorrect See explanation for response b.

The correct answer is: low validity.</p>

209
Q

It is generally believed that, of the lobes of the cerebral cortex, the __________ lobes are the last to fully develop and myelinate.
Select one:

A.
frontal

B.
temporal

C.
parietal

D.
occipital

A

Development of the cortex corresponds to the emergence of various capabilities.
a. CORRECT The frontal lobes mediate a number of functions including higher-order cognitive skills, which do not fully develop until early adulthood.

b. Incorrect See explanation for response a.
c. Incorrect See explanation for response a.
d. Incorrect See explanation for response a.

The correct answer is: frontal

210
Q

Of the various types of test validity, ________ validity has been described as the broadest category of validity because it overlaps and encompasses all other types.
Select one:

A.
content

B.
construct

C.
criterion-related

D.
contextual

A
The classical (tripartite) view of validity distinguishes between three major types -- content, criterion-related, and construct. From this perspective, construct validity refers to the degree to which a test measures the construct it was designed to measure.
a. Incorrect See explanation for response b.

b. CORRECT Due to changes in the conceptualization of validity, methods for evaluating validity, and methods of interpreting test scores, construct validity is now described by many experts as a unifying concept of validity. See, e.g., M. E. Strauss and G. T. Smith, Construct validity: Advances in theory and methodology, Annual Review of Clinical Psychology, 5, 1-25, 2009.
c. Incorrect See explanation for response b.
d. Incorrect See explanation for response b.

The correct answer is: construct

211
Q

When using the DSM-5, a diagnosis of Major Neurocognitive Disorder requires the presence of a significant decline in:
Select one:

A.
social and occupational functioning from premorbid functioning.

B.
cognitive functioning that interferes with independence in everyday activities.

C.
at least three areas of cognitive functioning.

D.
the ability to perform activities of daily living.

A

Answer B is correct: The DSM-5 distinguishes between Major and Mild Neurocognitive Disorders. The diagnosis of a Major Neurocognitive Disorder requires a significant decline from a previous level of cognitive functioning that interferes with independence in everyday activities; while the diagnosis of a Mild Neurocognitive Disorder requires a modest decline from a previous level of cognitive functioning that does not interfere with independence in everyday activities.

Answers A, C, and D: See explanation for answer B.

The correct answer is: cognitive functioning that interferes with independence in everyday activities.

212
Q

Research conducted in a number of countries has confirmed that, when men are shown pictures of women of various shapes and sizes, they tend to prefer women whose waist circumference is about ___ percent of their hip circumference.
Select one:

A.
30

B.
50

C.
70

D.
90

A

Evolutionary psychologists propose that a woman’s waist-to-hip ratio provides potential mates with information about her reproductive potential.
a. Incorrect See explanation for response c.

b. Incorrect See explanation for response c.
c. CORRECT Studies in the United States, Britain, and other countries have found that men generally prefer a low waist-to-hip ratio of around .7. According to D. Singh, this is because this ratio is associated with greater fertility and health (Adaptive significance of waist-to-hip ratio and female physical attractiveness, Journal of Personality and Social Psychology, 65, 293-307, 1993).
d. Incorrect See explanation for response c.

The correct answer is: 70

213
Q

Haloperidol (Haldol) exerts its effects primarily by blocking _________ receptors in certain areas of the brain.
Select one:

A.
GABA

B.
norepinephrine

C.
dopamine

D.
glutamate

A

Knowing that haloperidol is an antipsychotic drug would have helped you identify the correct answer to this question.
a. Incorrect See explanation for response c.

b. Incorrect See explanation for response c.
c. CORRECT Like other conventional antipsychotics, haloperidol is a dopamine antagonist and exerts its therapeutic effects primarily by blocking dopamine receptors. However, it also has some effect on acetylcholine and serotonin levels.
d. Incorrect See explanation for response c.

The correct answer is: dopamine

214
Q

You design a study to compare the effectiveness of three different treatments for stress. You decide to divide 30 EPPP candidates equally among three treatments and measure their stress level before the treatment begins and one week and six weeks after the treatment has been applied. The design of your study is best described as:
Select one:

A.
counterbalanced.

B.
mixed.

C.
natural groups.

D.
between-groups.

A

Answer B is correct: This study has two independent variables - treatment and time. This is a mixed design because one of the variables is a between-groups variable (treatment), while the other is a within-subjects variable (time).

Answer A: Counterbalancing involves presenting the different levels of an IV to different subjects or groups of subjects in different orders.

Answer C: The term “natural groups” is used to describe a study in which the IV is an organismic variable (e.g., intelligence, self-esteem).

Answer D: In a between groups design, comparisons of different levels of the IV are made between different groups.

The correct answer is: mixed.

215
Q

According to John D. Krumboltz (1996), career development and career choice are related primarily to:

Select one:

A.
needs and drives.

B.
early childhood experiences.

C.
ego identity development.

D.
social learning.

A

Krumboltz proposes that career decisions are based on what the individual has learned.<p>a. Incorrect See explanation for response d.</p><p>b. Incorrect See explanation for response d.</p><p>c. Incorrect See explanation for response d.</p><p>d. CORRECT Krumboltz’s emphasis is on social learning – i.e., modeling and other aspects of the social environment – and his theory is referred to as the Social Learning Theory of Career Decision Making (SLTCDM). Additional information about Krumboltz’s theory is provided in the Industrial-Organizational Psychology chapter of the written study materials.

The correct answer is: social learning.</p>

216
Q

To be consistent with the requirements of the Uniform Guidelines on Employee Procedures, when it is determined that a selection procedure is having an adverse impact, an employer must:
Select one:

A.
stop using the procedure for making selection decisions.

B.
use the procedure only in conjunction with other validated measures.

C.
obtain the consent of each applicant before using the procedure.

D.
validate that the procedure is job-related.

A

When it is determined that a selection procedure is having an adverse impact, the employer cannot continue using it unless he/she can demonstrate that doing so is a business necessity.
a. Incorrect See explanation for response d.

b. Incorrect See explanation for response d.
c. Incorrect See explanation for response d.
d. CORRECT An employer can show that continued use of the selection procedure is a business necessity by demonstrating that (a) the measure is valid by establishing its relationship to successful job performance and (b) there are no alternate methods that would have less adverse impact.

The correct answer is: validate that the procedure is job-related.

217
Q

The research has found that activity in which of the following areas of the brain shows the greatest increase in activity when White research subjects are shown pictures of African American (versus White) faces?
Select one:

A.
pons

B.
amygdala

C.
hippocampus

D.
caudate nucleus

A

As might be expected, the area of the brain most associated with emotions is the area that exhibits different levels of activity when subjects view pictures of African American and White faces
a. Incorrect See explanation for answer b.

b. CORRECT Studies using functional MRI imaging to investigate the neural correlates of race evaluation have consistently found that, when White subjects are shown pictures of White and African American faces, they tend to exhibit a greater amygdala response to Africa American faces and that, the darker the skin tone of the African American faces, the greater the amygdala response. See, e.g., J. Ronquillo et al., The effects of skin tone on race-related amygdala activity: An fMRI investigation, Social Cognitive and Affective Neuroscience, 2(1), 39-44, 2007. Note that there is also some evidence that the increased amygdala response to African American faces also occurs when African American and White subjects are shown pictures of African American faces.
c. Incorrect See explanation for answer b.
d. Incorrect See explanation for answer b.

The correct answer is: amygdala

218
Q

One difference between Generalized Anxiety Disorder and nonpathological anxiety is that Generalized Anxiety Disorder is more likely to be accompanied by physical symptoms. In addition, a person with Generalized Anxiety Disorder:
Select one:

A.
feels he or she caused the problems underlying current worries.

B.
makes repeated, but unsuccessful, attempts to avoid anxiety-arousing stimuli.

C.
experiences anxiety that is unrelated to specific events or actions.

D.
finds it difficult to control his or her worrying.

A

Generalized Anxiety Disorder (GAD) is characterized by excessive anxiety and worry about a number of events or activities.

a. Incorrect See explanation for response d.
b. Incorrect See explanation for response d.
c. Incorrect Although worry may occur without a precipitant, it is often related to specific events or activities.
d. CORRECT One of the diagnostic criteria for GAD is that the person finds it difficult to control his/her anxiety and worry.

The correct answer is: finds it difficult to control his or her worrying.

219
Q

A researcher compares the effectiveness of two different relaxation techniques for relieving test anxiety by randomly assigning 30 students who have high levels of anxiety to one of the treatments so that each group includes 15 students. If the researcher uses the t-test for independent (unrelated) samples to analyze the data she obtains, the degrees of freedom will be:
Select one:

A.
13

B.
14

C.
28

D.
29

A

The formula for the degrees of freedom for the t-test for independent samples is N-2, where N is the total number of participants.
a. Incorrect See explanation for response c.

b. Incorrect See explanation for response c.
c. CORRECT The total number of participants is 30: 30 - 2 = 28.
d. Incorrect See explanation for response c.

The correct answer is: 28

220
Q

One approach to distinguishing between depression and anxiety involves considering three dimensions of emotion – negative affect, positive affect, and autonomic arousal. From this perspective, compared to depression, anxiety involves:
Select one:

A.
a similar level of negative affect and autonomic arousal but a higher level of positive affect.

B.
a similar level of negative affect but a higher level of both autonomic arousal and positive affect.

C.
a lower level of negative affect, a similar level of autonomic arousal, and a higher level of positive affect.

D.
a similar level of both negative and positive affect but a higher level of autonomic arousal.

A

Considering the characteristics of depression and anxiety might have helped you choose the correct response to this question even if you are unfamiliar with the research.

a. Incorrect See explanation for response b.
b. CORRECT As described in the Abnormal Psychology chapter of the written study materials, this is the difference reported by L. A. Clark et al., Temperament, personality, and the mood and anxiety disorders, Journal of Abnormal Psychology, 103, 92-102, 1994.
c. Incorrect See explanation for response b.
d. Incorrect See explanation for response b.

The correct answer is: a similar level of negative affect but a higher level of both autonomic arousal and positive affect.

221
Q

In training, providing “identical elements” is most useful for:
Select one:

A.
maximizing overlearning.

B.
reducing the probability of proactive and retroactive interference.

C.
ensuring transfer of training.

D.
providing opportunities for active practice.

A

The term “identical elements” refers to similarities in the learning and performance environments.
a. Incorrect See explanation for response c.

b. Incorrect See explanation for response c.
c. CORRECT Transfer of training is maximized when learning conditions are similar to performance conditions (i.e., when they are similar in terms of task, equipment, environment, etc.).
d. Incorrect See explanation for response c.

The correct answer is: ensuring transfer of training.

222
Q

Bartlett Barmicide says he’s continued having hallucinations (“flashbacks”) several months after he stopped using a drug. He also states that he realizes that this is probably due to the drug. Bartlett most likely used which of the following?
Select one:

A.
an inhalant

B.
methamphetamine

C.
cocaine

D.
mescaline

A

Knowing that mescaline is an hallucinogen would have helped you identify the correct answer to this question.

a. Incorrect See explanation for response d.
b. Incorrect See explanation for response d.
c. Incorrect See explanation for response d.
d. CORRECT The DSM diagnosis of Hallucinogen Persisting Perception Disorder is characterized by re-experiencing perceptual disturbances reminiscent of those experienced during earlier Hallucinogen Intoxication. Of the substances listed in the answers, only mescaline is a type of hallucinogen.

The correct answer is: mescaline

223
Q

Research investigating the relationship between leader intelligence and leader effectiveness has found a __________ relationship between these two variables.
Select one:

A.
strong positive

B.
strong negative

C.
weak positive

D.
weak negative

A

Perhaps surprisingly, the research has found that scores on cognitive ability tests are poor predictors of leader effectiveness.
a. Incorrect See explanation for response c.

b. Incorrect See explanation for response c.
c. CORRECT F. Fiedler and T. Link note that prior reviews of the research report median correlations of .20 to .30 between various measures of intellectual ability and leadership and managerial performance [Leader intelligence, interpersonal stress, and task performance, in R. J. Sternberg & R. J. Wagner (Eds.), Mind in context: Interactionist perspectives on human intelligence (pp. 152-170), New York, Cambridge University Press, 1994]. Explanations for these low correlations focus on potential moderating variables. For example, one explanation is that it is the relative intelligence of the leader to the intelligence of his/her supervisees that is most important - i.e., that leaders are most effective when they are only slightly more intelligent than their followers.
d. Incorrect See explanation for response c.

The correct answer is: weak positive

224
Q

An African American client feels that the primary obstacle in his life is racism and that there’s nothing he can do to change the problems he is experiencing. As defined by Sue (1978), this individual has an:
Select one:

A.
internal locus of control and internal locus of responsibility.

B.
external locus of control and internal locus of responsibility.

C.
internal locus of control and external locus of responsibility.

D.
external locus of control and external locus of responsibility.

A

Even if you are unfamiliar with Sue’s conceptualization of worldview, you probably could have identified the correct response by considering the information provided in the question.
a. Incorrect See explanation for response d.

b. Incorrect See explanation for response d.
c. Incorrect See explanation for response d.
d. CORRECT This individual has an external locus of responsibility (he can’t do anything about his problems) and an external locus of control (he believes his problems are due to racism).

The correct answer is: external locus of control and external locus of responsibility.

225
Q

A client you have been seeing for two months suddenly informs you that this will be his last session. You believe this is not a good time for the client to stop therapy and are concerned about the potential negative consequences of doing so. You should:
Select one:

A.
allow the client to quit.

B.
provide the client with appropriate referrals.

C.
discuss with him the reasons you believe he should continue and recommend that he re-think his decision.

D.
inform him that you are not responsible for any negative consequence of terminating prematurely.

A

Obviously, a client has a right to discontinue treatment. This creates an ethical dilemma, however, when the therapist believes that termination is ill-advised.
a. Incorrect See explanation for response c.

b. Incorrect See explanation for response c.
c. CORRECT This is the best response of those given. In this situation, the therapist would want to discuss the possible risks of terminating therapy at this time and encourage the client to re-think his decision. Although the action described in answer b would also be important, it should follow a discussion about the possible consequences of leaving.
d. Incorrect See explanation for response c.

The correct answer is: discuss with him the reasons you believe he should continue and recommend that he re-think his decision.